Anda di halaman 1dari 190

aa

Buku Wajib
OLIMPIADE SMA

MATEMATIKA

AL JABAR
TEORI BILANGAN
KOMBINATORIKA
&
TANTANGAN OLIMPIADE

Ikatan Pemerhati Matematika Indonesia (IPMI)


Klinik Pendidikan Indonesia
aa

Buku Wajib
Olimpiade Matematika SMA
OSK
OSP
OSN
IMO

PEMERHATI MATEMATIKA INDONESIA


KLINIK PENDIDIKAN
http://klinikpendidikan.com

Penulis :
Umar Fauzi MZ
Youtube Chanel IZI SMART
aa

Prakata

Salam Olimpiade

Buku ini merupakan buku wajib yang perlu dipelajari oleh calon peserta
olimpiade Matematika SMA baik tingkat Kabupaten/Kotamadya ataupun
tingkat Propinsi dan juga perlu untuk refferensi para pemerhati matematika di
seluruh Indonesia.

Buku ini selain berisi teori dasar dan contoh-contoh problem solving yang
umum ada di dalam silabus materi Olimpiade Matematika SMA baik OSK,
OSP ataupun OSN. Oleh karena itu buku ini sangat cocok digunakan sebagai
buku wajib bagi pemula atau siswa peserta OSK dan OSP.

Soal-soal dan pembahasan didalam buku ini diharapkan dapat menambah


khazanah atau pengetahuan yang memberikan kontribusi bagi seluruh
pengguna, terutama siswa agar lebih semangat untuk meraih prestasi.

Semoga buku wajib ini bermanfaat sebagai bahan belajar. Masukan dan saran
yang konstruktif kami nantikan untuk penyempurnaan buku ini dimasa yang
akan datang.

Yogyakarta, April 2019


Penulis
aa

Daftar Isi

ALJABAR
Sistem Bilangan Real 2
Pertaksamaan AM GM HM 17
Pertaksamaan Chaucy Schwart 22
Pola Bilangan 24
Polinomial 28
Problem Solving 35
Soal-soal Latihan Isian Singkat 54
Soal-soal Latihan Uraian Singkat 61
Soal dan Pembahasan 62

TEORI BILANGAN
Teorema Keterbagian 69
FPB dan KPK 74
Kekongruenan 78
Persamaan Diophantine 82
Fungsi Tangga 85
Problem Solving 90
Soal-soal Latihan Isian Singkat 102
Soal-soal Latihan Isian Singkat 108
Soal dan Pembahasan 109

KOMBINATORIKA
Aturan Penempatan (Filling Slots) 115
Permutasi dan Kombinasi 116
Prinsip Sarang Merpati (Pigeonhole Principle) 123
Prinsip Eksklusi dan Inklusi 124
Segitiga Pascal dan Binomium Newton 126
Peluang Kejadian 128
Problem Solving 134
Soal-soal Latihan Isian Singkat 149
Soal-soal Latihan Uraian Singkat 156
Soal dan Pembahasan 157

TANTANGAN OLIMPIADE 165


Olimpiade Matematika SMA 2020

Bab 1

ALJABAR
• Sistem Bilangan Real
• Pertaksamaan AM GM HM
• Pertaksamaan Chaucy Schwart
• Pola Bilangan
• Polinomial

1|AL JABAR
Olimpiade Matematika SMA 2020

AL JABAR
Bab 1
PENGANTAR TEORI

• SISTEM BILANGAN REAL

Bilangan real merupakan gabungan dari bilangan rasional dengan bilangan


irrasional. Bilangan rasional dapat berupa bilangan bulat ataupun dapat
𝒂
dinyatakan sebagai pecahan dalam bentuk dengan a, b bulat dan b  0.
𝒃
Sedangkan bilangan irrasional bukan merupakan hasil bagi dua bilangan
𝒂
sehingga tidak dapat dinyatakan sebagai bentuk . Bilangan irrasional juga
𝒃
dapat membentuk bilangan desimal yang tidak mengandung perulangan.
Contoh bilangan irrasional :

Π = 3,14159265358 .......
√2 = 1,414213562 ......

Bilangan Real dapat dikelompokan sebagai berikut :

• Bilangan Asli : 1, 2, 3, …, berfungsi sebagai bilangan kardinal untuk


menghitung banyaknya objek suatu himpunan. Himpunan bilangan asli
dinotasikan dengan huruf N dengan N ={ 1, 2, 3, …}.

• Bilangan Cacah : 0, 1, 2, 3, … adalah bilangan asli beserta unsur nol,


bilangan cacah biasanya juga disebut “ blangan bulat non negatif”.

• Bilangan Bulat Negatif : ( lawan bilangan asli ) : ......, –3, –2, –1

2|AL JABAR
Olimpiade Matematika SMA 2020

• Bilangan Bulat : ….–3, –2, –1, 0, 1, 2, 3, … adalah bilangan bulat terdiri


atas bilangan genap dan bilangan ganjil. Untuk n bilangan bulat :
❖ Bilangan genap : …–4, –2, 0, 2, 4, 6 … adalah bilangan bulat
kelipatan dua yang dinotasikan 2n.
❖ Bilangan ganjil : …–3, –1, 1, 3, 5, 7, 9 …adalah bilangan bulat bukan
kelipatan dua, yang dinotasikan 2n+1 atau 2n–1.

• Bilangan Prima : 2, 3, 5, 7, 11, ........ adalah bilangan yang tidak dapat


menghasilkan bilangan bulat apabila dibagi dengan bilangan bulat
lainnya kecuali 1 dan dirinya sendiri.

𝑛
• Bilangan Pecahan : adalah bilangan berbentuk , dimana n dan m
𝑚
bilangan bulat dengan m  0.

• Bentuk akar rasional


𝑛
Jika n bilangan genap positif, akar ke-n dari bilangan positif a, ditulis √𝑎
didefinisikan sebagai bilangan positif x yang memenuhi 𝑥 𝑛 = 𝑎.
4
Contoh, √81 = 3, Karena 3 bilangan positif yang memenuhi 𝑥 4 = 81
Jika n bilangan ganjil positif, n > 1, akar ke-n dari bilangan real a, ditulis
𝑛
√𝑎 didefinisikan sebagai bilangan real x yang memenuhi 𝑥 𝑛 = 𝑎

Contoh, 5√(−32) = −2, Karena −2 bilngan real yang memenuhi 𝑥 5 = −32

Selanjutnya kita mengenal istilah :


Bilangan kuadrat, seperti 1, 4, 9, 16, ....... (𝑛2 dengan n bilangan Asli)
Bilangan kubik, seperti 1, 8, 27, 64, ........ (𝑛3 dengan n bilangan Asli)

Sistem bilangan Real


adalah himpunan bilangan real yang disertai dengan operasi penjumlahan dan
perkalian sehingga memenuhi tiga aksioma, yaitu aksioma lapangan, aksioma
urutan dan aksioma kelengkapan.

• Aksioma Lapangan adalah aksioma yang mengatur tentang ketertutupan


terhadap operasi penjumlahan dan perkalian, sifat kumulatif, asosiatif,
distributive, unsur invers dan beberapa unsur di dalam aljabar lain
terhadap penjumlahan dan perkalian.

3|AL JABAR
Olimpiade Matematika SMA 2020

• Aksioma Urutan adalah aksioma yang mengatur tentang pemunculan


bilangan positif dan negatif, sehingga setiap bilangan real dapat
diurutkan dari kecil sampai besar (dikenal notasi <, =, ≤ , > dan ≥) . Dari
aksioma ini pula dapat diturunkan berbagai sifat yang mendasari
penyelesaian suatu pertidaksamaan. Selanjutnya dirancang konsep nilai
mutlak sebagai ukuran jarak dua bilangan real dan suatu alat untuk
menyelesaikan pertidaksamaan yang berkaitan dengan limit.

• Aksioma Kelengkapan adalah aksioma yang mengatur tentang adanya


batas atas terkecil atau batas bawah terbesar bagi setiap himpunan bagian
R yang tidak kosong. Persamaan-persamaan dalam berbagai operasi
hitung, kemudian diperkenalkan konsep interval hingga dan interval tak
hingga, yang akan berperan dalam aljabar dan kalkulus.

Jadi dengan mempelajari sistem bilangan real ini, kita dapat dengan mudah
mengenal lebih jauh mengenai berbagai unsur material di dalam matematika
secara keseluruhan, karena pada dasarnya bilangan real adalah semesta
bilangan yang menyediakan solusi banyak permasalahan.

Soal-soal dan Pembahasan :

1. Berapa jumlah semua bilangan asli yang terdiri dari 3 digit ?


Pembahasan :
Menentukan jumlah semua bilangan Asli yang terdiri dari 3 digit.
𝑛 (𝑛+1)
Kita ingat bahwa 1 + 2 + 3 + 4 + 5 + ....... + n =
2
Misalkan :
A = 1 + 2 + 3 + 4 + 5 + ....... + 999
B = 1 + 2 + 3 + 4 + 5 + ......... + 99

999 (999+1) 999000


Jadi A = =
2 2

99 (99+1) 9900
Jadi B = =
2 2

Maka jumlah semua bilangan Asli yang terdiri dari 3 digit = A – B

4|AL JABAR
Olimpiade Matematika SMA 2020

2. Jumlah dua buah bilangan prima adalah 63. Tentukan kedua bilangan prima
tersebut !
Pembahasan :
Jumlah dua bilangan bulat menghasilkan bilangan ganjil, pastilah salah satu
bilangan bulat itu adalah bilangan genap.
Jika kedua bilangan bulat itu adalah bilangan prima, maka pasti salah satu
bilangan itu adalah 2. Karena 2 adalah satu-satunya bilangan prima yang
genap. Jadi kedua bilangan prima itu adalah 2 dan 61.

3. Ubahlah bilangan desimal 1,24242424...... menjadi bentuk pecahan !


Pembahasan :
Bilangan desimal 1,24242424...... adalah bilangan rasional karena ada blok
bilangan yang terurut, oleh karena itu dapat diubah menjadi pecahan.
24 99 24 123
1,24242424...... = 1,24 = 1 + 0,24 = 1 + = + =
99 99 99 99

Perhatikan beberapa lain contoh berikut ini :


347 2345
2,347347347 ..... = 2,347 = 2 + 0,347 = 2 + =
999 999
31 36 3069+36 3105
3,136363636 ...... = 3,136 = 3,1 + 0,036 = + = =
10 990 990 990

4. Apakah √90 + √90 + √90 + ⋯ … . . bilangan rasional ?

Pembahasan :

Untuk mengetahui √90 + √90 + √90 + ⋯ … . . bilangan rasional atau bukan,


maka kita harus mengubah bentuk bilangan tersebut.

Misalkan √90 + √90 + √90 + ⋯ … . . adalah x. Kemudian dikuadratkan.

90 + √90 + √90 + √90 + ⋯ … . . = 𝑥 2

90 + 𝑥 = 𝑥 2 , menghasilkan x = 10 melalui metoda pemfaktoran

Jadi √90 + √90 + √90 + ⋯ … . . = 10 , bilangan rasional bukan ?

5|AL JABAR
Olimpiade Matematika SMA 2020
1 1 1
5. Berapakah hasil dari + + ?
√2 + 1 √3+√2 √4 + √3

Pembahasan :
1 1 1
Untuk mendapatkan hasil dari + +
maka kita harus
√2 + 1 √3+√2 √4 + √3
merasionalkan dulu pecahan tersebut dengan perkalian sekawan.

1 1 √2 − 1
= .( ) = √2 − 1
√2 + 1 √2 + 1 √2 − 1

1 1 √3 − √2
= .( ) = √3 − √2
√3 + √2 √3 + √2 √3 − √2

Sehingga :
1 1 1
+ + = (√2 − 1) + (√3 − √2) + (√4 − √3)
√2 + 1 √3 + √2 √4 + √3
1 1 1
+ + = − 1 + √4
√2 + 1 √3 + √2 √4 + √3
=1

Operasi bilangan real diantaranya adalah operasi hitung aljabar pada bentuk
akar, sistem persamaan dan bentuk pangkat.

1. Rumus-rumus bentuk Akar (𝑎 𝑑𝑎𝑛 𝑏 real positif)

√𝑎 + 𝑏 ± 2√𝑎𝑏 = √𝑎 ± √𝑏

Contoh : √10 + 2√21 = √7 + √3

2. Rumus-rumus jumlah dan selisih kuadrat dua bilangan

𝑎2 + 𝑏 2 = (𝑎 + 𝑏)2 − 2𝑎𝑏

𝑎2 − 𝑏 2 = (𝑎 − 𝑏)(𝑎 + 𝑏)

6|AL JABAR
Olimpiade Matematika SMA 2020

Contoh soal 1
Andy baru boleh memiliki KTP pada tahun 𝑛2 dengan 𝑛 ∈ 𝐵. Sekarang masih
2
tahun (√2019 + √1 + 2(2019 − √20192 + 2019) ) , maka usia Andy

sekarang adalah ... (Asumsikan usia minimal memiliki KTP adalah 17 tahun)

Pembahasan :
2
Kita akan pecahkan dulu bentuk (√2019 + √1 + 2(2019 − √20192 + 2019) ) .

2
Disederhanakan menjadi (√2019 + √2020 + 2019 − 2√2019.2020 )

2 2
(√2019 + (√2020 − √2019) ) = (√2020) = 2020
2
Jadi sekarang tahun (√2019 + √1 + 2(2019 − √20192 + 2019) ) = 2020

Tahun 𝑛2 yang dimaksudkan pastilah bilangan kuadrat terdekat dengan 2020,


yaitu 𝑛2 = 2025 , karena 2025 = 452
Pada umumnya syarat untuk memiliki KTP adalah usia 17 tahun, maka saat ini
usia Andy adalah 17 – (2025 – 2020) = 12 tahun

Contoh soal 2
Tentukan hasil dari
(20202019)2
(20202018)2 + (20202020)2 − 2
Pembahasan :

(20202019)2 (20202019)2
= =
(20202018)2 + (20202020)2 − 2 (20202018)2 − 1 + (20202020)2 − 1

(20202019)2
=
(20202018 + 1)(20202018 − 1) + (20202020 + 1)(20202020 − 1)
(20202019) (20202019) 202019
= =
(20202019)(20202017) + (20202021)(20202019) 20202017 + 20202021
202019 202019 20202019 1
= = =
20202017 + 20202021 40404038 2 (20202019) 2

7|AL JABAR
Olimpiade Matematika SMA 2020

Contoh soal 3
𝑎 𝑏
Misalkan 𝑎 = 2√2 − √8 − 4√2 dan 𝑏 = 2√2 + √8 − 4√2. Jika + = 𝑥 + 𝑦√2
𝑏 𝑎
dengan 𝑥 , 𝑦 bulat, maka 𝑥 + 𝑦 = ..
Pembahasan :

𝑎 = 2√2 − √8 − 4√2

𝑏 = 2√2 + √8 − 4√2

𝑎 + 𝑏 = 2√2 − √8 − 4√2 + 2√2 + √8 − 4√2 = 4√2

𝑎. 𝑏 = (2√2 − √8 − 4√2) (2√2 + √8 − 4√2) = 4√2

Mengingat jumlah kuadrat akar-akar suatu bilangan : 𝑎2 + 𝑏 2 = (𝑎 + 𝑏)2 − 2𝑎𝑏


Maka :
2
𝑎2 + 𝑏 2 = (4√2) − 8√2

𝑎2 + 𝑏 2 = 32 − 8√2

𝑎 𝑏
Diketahui juga bahwa + = 𝑥 + 𝑦√2
𝑏 𝑎

𝑎2 + 𝑏 2 32 − 8√2
= = 𝑥 + 𝑦√2
𝑎𝑏 4√2

32 − 8√2 √2
( ) = 𝑥 + 𝑦√2
4√2 √2

32√2 − 16
= 𝑥 + 𝑦√2
8

𝑥 + 𝑦√2 = −2 + 4√2
Maka 𝑥 = −2 dan 𝑦 = 4 , jadi 𝑥 + 𝑦 = 2

Catatan :
Jika a, b, c real positif maka berlaku juga bentuk

• √𝑎 + 𝑏 + 𝑐 + 2(√𝑎𝑏 + √𝑏𝑐 + √𝑎𝑐) = √𝑎 + √𝑏 + √𝑐

8|AL JABAR
Olimpiade Matematika SMA 2020

Contoh Soal 4
Tentukan hasil dari

1 1 1 1 1
𝐴= + + + + ⋯….+
1 1+2 1+2+3 1+2+3+4 1 + 2 + 3 + 4 + ⋯ 2020

Pembahasan :

1 1 1 1 1
A = + + + +⋯… +
1 1+2 1+2+3 1+2+3+4 1+2+3+4+⋯2020

𝐴 1 1 1 1 1
= + + + + ⋯… +
2 2 6 12 20 2 (1+2+3+4+⋯2020)

𝐴 1 1 1 1 1 1
= + + + + + ⋯……….+
2 1 .2 2 .3 3 .4 4 .5 5 .6 2020 .2021

𝐴 1 1 1 1 1 1 1 1 1 1
= − + − + − + − + ⋯……….+ −
2 1 2 2 3 3 4 4 5 2020 2021

𝐴 1 1 2020
= − =
2 1 2021 2021

2 2 4040
maka A = − =
1 2021 2021

Contoh soal 5
Tentukan hasil dari
1 1 1 1 1
𝐴 = (1 − 2
) (1 − 2 ) (1 − 2 ) (1 − 2 ) … … . . … . (1 − )
2 3 4 5 20202

Pembahasan :
1 1 1 1 1
A = (1 − 22 ) (1 − 32 ) (1 − 42 ) (1 − 52 ) … … . . … . (1 − 20202 )

1 1 1 1 1 1 1
= (1 − 2) (1 + 2) (1 − 3) (1 + 3) (1 − 4) … … . . … . (1 − 2020) (1 + 2020)

1 3 2 4 3 5 2019 2021
= ( ) ( ) ( ) ( ) ( ) ( )……..….( )( )
2 2 3 3 4 4 2020 2020

1 2021
= ( )( )
2 2020

2021
=
4040

9|AL JABAR
Olimpiade Matematika SMA 2020

Penguraian dan Faktorisasi

Penguraian adalah suatu transformasi bentuk perkalian ke bentuk penjumlahan.


Sedangkan Faktorisasi adalah sebaliknya.

Contoh :
Penguraian :
(2𝑥 + 3)3 𝑥 = (8𝑥 3 + 27 + 18𝑥)𝑥 = 8𝑥 4 + 18𝑥 2 + 27𝑥

Faktorisasi :
𝑥 7 − 𝑥 = 𝑥 (𝑥 6 − 1) = 𝑥 (𝑥 3 − 1) (𝑥 3 + 1) = 𝑥 (𝑥 − 1)(𝑥 2 + 𝑥 + 1) (𝑥 3 + 1)

Untuk menguraikan dan memfaktorkan sebuah persamaan, beberapa rumus


dasar tentang jumlah dan selisih pangkat tiga dua bilangan berikut ini kita
perlukan :

𝑎3 + 𝑏 3 = (𝑎 + 𝑏)3 − 3𝑎𝑏(𝑎 + 𝑏)

𝑎3 − 𝑏 3 = (𝑎 − 𝑏)3 + 3𝑎𝑏(𝑎 − 𝑏)

𝑎3 + 𝑏 3 = (𝑎 + 𝑏)(𝑎2 − 𝑎𝑏 + 𝑏 2 )

𝑎3 − 𝑏 3 = (𝑎 − 𝑏)(𝑎2 + 𝑎𝑏 + 𝑏 2 )

Contoh soal 6
Tentukan nilai 𝑥 dan 𝑦, Real yang memenuhi (𝑥 + 𝑦)2 = (𝑥 + 1)(𝑦 − 1)

Pembahasan :
(𝑥 + 𝑦)2 = (𝑥 + 1)(𝑦 − 1)
((𝑥 + 1) + (𝑦 − 1))2 = (𝑥 + 1)(𝑦 − 1)
(𝑥 + 1)2 + (𝑦 − 1)2 + 2(𝑥 + 1)(𝑦 − 1) = (𝑥 + 1)(𝑦 − 1)
(𝑥 + 1)2 + (𝑦 − 1)2 + (𝑥 + 1)(𝑦 − 1) = 0
(𝑥 + 1)(𝑥 + 1 + 𝑦 − 1) + (𝑦 − 1)(𝑦 − 1 + 𝑥 + 1) = 0
(𝑥 + 1)(𝑥 + 𝑦) + (𝑦 − 1)(𝑥 + 𝑦) = 0
(𝑥 + 𝑦) (𝑥 + 𝑦) = 0
(𝑥 + 𝑦)2 = 0 , akibatnya (𝑥 + 1)(𝑦 − 1) = 0

Nilai yang memenuhi adalah 𝑥 = −1 dan 𝑦 = 1

10 | A L J A B A R
Olimpiade Matematika SMA 2020

Contoh soal 7
Jika 𝑎 adalah bilangan real positif dan 𝑎2 + 𝑎−2 = 23, maka 𝑎3 + 𝑎−3 = ⋯

Pembahasan :

𝑎2 + 𝑎 −2 = (𝑎1 + 𝑎 −1 )2 − 2 𝑎1 𝑎−1 = 23
(𝑎 + 𝑎−1 )2 − 2 𝑎0 = 23
(𝑎 + 𝑎−1 )2 = 25 , maka kita ambil 𝑎 + 𝑎−1 = 5
Jadi 𝑎3 + 𝑎−3 = (𝑎 + 𝑎−1 )3 − 3𝑎𝑎−1 (𝑎 + 𝑎−1 )
𝑎3 + 𝑎 −3 = (5)3 − 3(5) = 110

Contoh soal 8
Jika 𝑎 adalah salah satu solusi dari 𝑥 2 − 2𝑥 + 3 = 0 , tentukan 𝑎3 − 𝑎 !
Pembahasan :
Karena 𝑥 = 𝑎 maka berlaku 𝑎2 − 2𝑎 + 3 = 0. Kemudian kita akan melakukan
manipulasi aljabar untuk mendapatkan persamaan yang diminta.
𝑎2 − 2𝑎 + 3 = 0, atau 𝑎2 − 2𝑎 = −3
𝑎2 − 1 − 2𝑎 + 4 = 0
𝑎2 − 1 = 2𝑎 − 4
𝑎(𝑎2 − 1) = 𝑎(2𝑎 − 4)
𝑎3 − 𝑎 = 2(𝑎2 − 2𝑎)
𝑎3 − 𝑎 = 2(−3) = −6

Identitas Aljabar
𝑎4 + 𝑏 4 = (𝑎2 + 𝑏 2 )2 − 2𝑎2 𝑏 2
𝑎5 + 𝑏 5 = (𝑎3 + 𝑏 3 )(𝑎2 + 𝑏 2 ) − (𝑎𝑏)2 (𝑎 + 𝑏)
𝑎7 + 𝑏 7 = (𝑎4 + 𝑏 4 )(𝑎3 + 𝑏 3 ) − (𝑎𝑏)3 (𝑎 + 𝑏)

𝑎4 − 𝑏 4 = (𝑎 − 𝑏)(𝑎3 + 𝑎2 𝑏 + 𝑎𝑏 2 + 𝑏 3 )
𝑎5 − 𝑏 5 = (𝑎 − 𝑏)(𝑎4 + 𝑎3 𝑏 + 𝑎2 𝑏 2 + 𝑎𝑏 3 + 𝑏 4 )

𝑎2 + 𝑏 2 = (𝑎 + 𝑏)2 − 2𝑎𝑏
𝑎2 + 𝑏 2 + 𝑐 2 = (𝑎 + 𝑏 + 𝑐)2 − 2 (𝑎𝑏 + 𝑏𝑐 + 𝑎𝑐)
𝑎2 + 𝑏 2 + 𝑐 2 + 𝑑2 = (𝑎 + 𝑏 + 𝑐 + 𝑑)2 − 2 (𝑎𝑏 + 𝑏𝑐 + 𝑐𝑑 + 𝑎𝑐 + 𝑎𝑑 + 𝑏𝑑)

𝑎3 + 𝑏 3 + 𝑐 3 = (𝑎 + 𝑏 + 𝑐)(𝑎2 + 𝑏 2 + 𝑐 2 − 𝑎𝑏 − 𝑏𝑐 − 𝑎𝑐) + 3𝑎𝑏𝑐


𝑎3 + 𝑏 3 + 𝑐 3 = (𝑎 + 𝑏 + 𝑐)3 − 3(𝑎𝑏 + 𝑎𝑐 + 𝑏𝑐)(𝑎 + 𝑏 + 𝑐) + 3𝑎𝑏𝑐
𝑎3 + 𝑏 3 + 𝑐 3 = (𝑎 + 𝑏 + 𝑐)3 − 3(𝑎 + 𝑏)(𝑏 + 𝑐)(𝑐 + 𝑎)
𝑎5 + 𝑏 5 + 𝑐 5 = (𝑎 + 𝑏 + 𝑐)5 − 5(𝑎 + 𝑏)(𝑏 + 𝑐)(𝑐 + 𝑎)(𝑎2 + 𝑏 2 + 𝑐 2 + 𝑎𝑏 + 𝑏𝑐 + 𝑐𝑎)

11 | A L J A B A R
Olimpiade Matematika SMA 2020

Contoh soal 9
Banyaknya pasangan terurut bilangan bulat (𝑎 , 𝑏) sehingga 𝑎2 + 𝑏 2 = 𝑎 + 𝑏
adalah ....
Pembahasan
𝑎2 + 𝑏 2 − (𝑎 + 𝑏) = 0
(𝑎 + 𝑏)2 − 2𝑎𝑏 − (𝑎 + 𝑏) = 0
(𝑎 + 𝑏)((𝑎 + 𝑏) − 1) = 2𝑎𝑏
Untuk (𝑎 , 𝑏) bulat maka haruslah 2𝑎𝑏 = 0
(𝑎 + 𝑏)((𝑎 + 𝑏) − 1) = 0
𝑎 + 𝑏 = 0 , 𝑎 = −𝑏
(𝑎 + 𝑏) − 1 = 0 , 𝑎 + 𝑏 = 1
Sehingga nilai yang mungkin a dan b adalah 0 dan 1.
Untuk a = − 1 menyebabkan nilai b tidak real, 𝑏 2 − 𝑏 + 2 = 0
jadi banyaknya pasangan (𝑎 , 𝑏) ada 4, yaitu (0 , 0), (1 , 0), (0 , 1) dan (1 , 1)

Contoh soal 10
Jika 𝑎 + 𝑏 + 𝑐 = 0 dan 𝑎3 + 𝑏 3 + 𝑐 3 = 𝑎5 + 𝑏 5 + 𝑐 5 , tentukan nilai 𝑎2 + 𝑏 2 + 𝑐 2 !

Pembahasan :
𝑎3 + 𝑏 3 + 𝑐 3 = (𝑎 + 𝑏 + 𝑐)3 − 3(𝑎 + 𝑏)(𝑏 + 𝑐)(𝑐 + 𝑎)
𝑎3 + 𝑏 3 + 𝑐 3 = 0 − 3(−𝑐)(−𝑎)(−𝑏)
𝑎3 + 𝑏 3 + 𝑐 3 = 3𝑎𝑏𝑐

𝑎5 + 𝑏 5 + 𝑐 5 = (𝑎 + 𝑏 + 𝑐)5 − 5(𝑎 + 𝑏)(𝑏 + 𝑐)(𝑐 + 𝑎)(𝑎2 + 𝑏 2 + 𝑐 2 + 𝑎𝑏 + 𝑏𝑐 + 𝑐𝑎)


𝑎5 + 𝑏 5 + 𝑐 5 = 0 − 5(−𝑐)(−𝑎)(−𝑏)(𝑎2 + 𝑏 2 + 𝑐 2 + 𝑎𝑏 + 𝑏𝑐 + 𝑐𝑎)
𝑎5 + 𝑏 5 + 𝑐 5 = 5𝑎𝑏𝑐(𝑎2 + 𝑏 2 + 𝑐 2 + 𝑎𝑏 + 𝑏𝑐 + 𝑐𝑎)

Karena diketahui bahwa 𝑎3 + 𝑏 3 + 𝑐 3 = 𝑎5 + 𝑏 5 + 𝑐 5 maka,


3𝑎𝑏𝑐 = 5𝑎𝑏𝑐(𝑎2 + 𝑏 2 + 𝑐 2 + 𝑎𝑏 + 𝑏𝑐 + 𝑐𝑎)
3
𝑎2 + 𝑏 2 + 𝑐 2 + 𝑎𝑏 + 𝑏𝑐 + 𝑐𝑎 =
5
Mengingat
𝑎2 + 𝑏 2 + 𝑐 2 = (𝑎 + 𝑏 + 𝑐)2 − 2 (𝑎𝑏 + 𝑏𝑐 + 𝑎𝑐)
1
𝑎2 + 𝑏 2 + 𝑐 2 = 0 − 2 (𝑎𝑏 + 𝑏𝑐 + 𝑎𝑐) , maka 𝑎𝑏 + 𝑏𝑐 + 𝑎𝑐 = − 2 (𝑎2 + 𝑏 2 + 𝑐 2 )
Jadi
1 2 3
𝑎2 + 𝑏 2 + 𝑐 2 − (𝑎 + 𝑏 2 + 𝑐 2 ) =
2 5
1 3 6
(𝑎2 + 𝑏 2 + 𝑐 2 ) = dan akhirnya 𝑎2 + 𝑏 2 + 𝑐 2 =
2 5 5

12 | A L J A B A R
Olimpiade Matematika SMA 2020

Daerah Penyelesaian
Setiap bilangan real dapat digambarkan sebagai titik pada garis, dan setiap titik
pada garis dapat dinyatakan sebagai representasi bilangan real. Hal ini berarti
terdapat “korespondensi satu-satu” diantara bilangan real dan titik pada garis.
Diantara dua bilangan real terdapat tak hingga banyaknya bilangan rasional dan
irrasional.

Oleh karena itu, kita harus mengenal interval titik-titik sebagai pernyataan solusi
atau kondisi seperti contoh sebagai berikut :

{𝑥|𝑎 ≤ 𝑥 ≤ 𝑏, 𝑥 ∈ 𝑅} boleh dituliskan [𝑎 , 𝑏]


{𝑥|𝑎 < 𝑥 ≤ 𝑏, 𝑥 ∈ 𝑅} boleh dituliskan (𝑎 , 𝑏]
{𝑥|𝑎 < 𝑥 < 𝑏, 𝑥 ∈ 𝑅} boleh dituliskan (𝑎 , 𝑏)
{𝑥|𝑎 ≤ 𝑥 < 𝑏 𝑎𝑡𝑎𝑢 𝑥 > 𝑏, 𝑥 ∈ 𝑅} boleh dituliskan [𝑎 , 𝑏) (𝑏 , )
{𝑥|𝑥 < 𝑎 𝑎𝑡𝑎𝑢 𝑥 ≥ 𝑏, 𝑥 ∈ 𝑅} boleh dituliskan (− , 𝑎) [𝑏 , )

Bentuk kuadrat 𝑎𝑥 2 + 𝑏𝑥 + 𝑐 dengan 𝑎 ≠ 0, dikatakan definit positif jika selalu


bernilai positif untuk setiap 𝑥 ∈ 𝑅.

Dengan diskriminan 𝐷 = 𝑏 2 − 4𝑎𝑐


𝑎𝑥 2 + 𝑏𝑥 + 𝑐 bernilai positif jika D < 0 dan 𝑎 > 0
𝑎𝑥 2 + 𝑏𝑥 + 𝑐 bernilai negatif jika D < 0 dan 𝑎 < 0

Contoh Soal 11
Tentukan himpunan penyelesaian sistem pertidaksamaan berikut :

𝑥3 + 8
√ >𝑥−2
𝑥
Pembahasan :
𝑥 3 +8
Anggap bahwa adalah suatu pecahan
𝑥

Kemungkinan pertama :
Pembilang pada pecahan : 𝑥 3 + 8 ≥ 0 , 𝑥 ≥ −2
Penyebut pada pecahan : 𝑥 > 0
Solusi kemungkinan pertama ini adalah 𝑥 > 0

13 | A L J A B A R
Olimpiade Matematika SMA 2020

Kemungkinan kedua :
Pembilang pada pecahan : 𝑥 3 + 8 ≤ 0 , 𝑥 ≤ −2
Penyebut pada pecahan : 𝑥 < 0
Solusi kemungkinan kedua ini adalah 𝑥 ≤ −2

Solusi dari kemungkinan pertama dan kedua adalah 𝑥 ≤ −2 atau 𝑥 > 0

Pada ruas kanan 𝑥 − 2 < 0 atau 𝑥<2 sangat mungkin ada nilai 𝑥 yang
memenuhi penyelesaian.

Untuk 𝑥 − 2 ≥ 0 persamaan ruas kiri dan kanan sudah ekuivalen sehingga kita
lakukan proses lebih lanjut :
𝑥3 + 8
> (𝑥 − 2)2
𝑥

Untuk 𝑥 > 0 , 𝑥 3 + 8 > 𝑥(𝑥 − 2)2

𝑥 3 + 8 > 𝑥 3 − 4𝑥 2 + 4𝑥
4𝑥 2 − 4𝑥 + 8 > 0
𝑥2 − 𝑥 + 2 > 0

𝑥 2 − 𝑥 + 2 selalu bernilai positif (definit positif) untuk setiap nilai x real, sehingga
𝑥3 + 8
> (𝑥 − 2)2 dipenuhi oleh 𝑥 > 0 , 𝑥 ∈ 𝑅
𝑥

Sehingga himpunan penyelesaian dari sistem pertidaksamaan ini adalah


𝑥 ≤ −2 atau 𝑥 > 0 atau (− . −𝟐]  (0 , +)

–2 0

Hp : {𝒙 | 𝑥 ≤ −2 𝑎𝑡𝑎𝑢 𝑥 > 0, 𝑥 ∈ 𝑅}
Hp : (− . −𝟐]  (0 , +)

14 | A L J A B A R
Olimpiade Matematika SMA 2020

Pertidaksamaan Nilai Mutlak


Nilai mutlak biasanya ditulis |𝑥| dan didefinisikan sebagai :

𝑥 , 𝑗𝑖𝑘𝑎 𝑥 ≥ 0
|𝑥| = {
−𝑥 , 𝑗𝑖𝑘𝑎 𝑥 < 0

|𝑥| = √𝑥 2 karena 𝑥 2 bernilai nol atau positif untuk nilai 𝑥 real


2
|𝑥|  (√𝑥) karena √𝑥 bernilai tidak real untuk 𝑥 negatif

Contoh penggunaan harga mutlak :


Misalnya, |𝑥 − 7| akan berubah tanda di 𝑥 = 7

• Jika 𝑥 ≥ 7 , maka 𝑥 − 7 ≥ 0, sehingga |𝑥 − 7| = 𝑥 − 7


• Jika 𝑥 < 7 , maka 𝑥 − 7 < 0, sehingga |𝑥 − 7| = 7 − 𝑥

Sehingga |𝑥 − 7| ditulis sebagai

𝑥 − 7 , 𝑗𝑖𝑘𝑎 𝑥 ≥ 7
|𝑥 − 7| = {
7 − 𝑥 , 𝑗𝑖𝑘𝑎 𝑥 < 7

Dengan demikian, jika kita mendapatkan masalah aljabar atau kalkulus yang
menggunakan tanda mutlak, terkadang kita harus mengubahnya terlebih dahulu
sesuai dengan syarat pertidaksamaan.

Contoh Soal 12
(a). Jika 𝑓(𝑥) = |3𝑥 − 8| , tentukan nilai dari 𝑓(𝑓(3))
4
(b). Tuliskan dalam bentuk integral hasil dari ∫−3|𝑥 2 − 4| 𝑑𝑥

Pembahasan :
8
(a). 𝑓(𝑥) = |3𝑥 − 8| akan berubah tanda di 𝑥 = 3
8
3𝑥 − 8 , 𝑥≥
𝑓(𝑥) = { 3
8
−3𝑥 + 8 , 𝑥<
3
Maka 𝑓(𝑓(3)) = 𝑓(𝑓(3(3) − 8)) = 𝑓(1) = −3(1) + 8 = 5

(b). Pada pemfaktoran 𝑥 2 − 4 = (𝑥 − 2)(𝑥 + 2) sesuai syarat harga mutlak


𝑥 2 − 4 ≥ 0 dan 𝑥 2 − 4 < 0 dituliskan sebagai berikut :

𝑥2 − 4 , 𝑥 ≤ −2  𝑥 ≥ 2
|𝑥 2 − 4| = {
−𝑥 2 + 4 , −2<𝑥 <2

15 | A L J A B A R
Olimpiade Matematika SMA 2020

Dengan demikian , integral tentu yang mau kita hitung hasilnya berbentuk
sebagai berikut :
4 −2 2 4

∫|𝑥 − 4| 𝑑𝑥 = ∫ (𝑥 − 4) 𝑑𝑥 + ∫(−𝑥 + 4) 𝑑𝑥 + ∫(𝑥 2 − 4) 𝑑𝑥


2 2 2

−3 −3 −2 2

Contoh soal 13
Jika S adalah himpunan semua bilangan bulat yang memenuhi |𝑥 2 − 5| ≤ 4 , dan
N adalah himpunan bagian dari S yang beranggotakan sedikitnya 2 bilangan,
maka banyaknya N adalah ....

Pembahasan :
Ada dua bentuk pertidaksamaan nilai mutlak sederhana sebagai berikut :
Bentuk |𝑓(𝑥)| < 𝑎 diubah menjadi −𝑎 < 𝑓(𝑥) < 𝑎
Bentuk |𝑓(𝑥)| > 𝑎 diubah menjadi 𝑓(𝑥) > 𝑎 atau 𝑓(𝑥) < −𝑎
Maka proses solusi dimulai dengan mencari himpunan penyelesaian.

|𝑥 2 − 5| ≤ 4
−4 ≤ 𝑥 2 − 5 ≤ 4
1 ≤ 𝑥2 ≤ 9

𝑥2 − 1 ≥ 0 𝑥2 − 9 ≤ 0
(𝑥 − 1)(𝑥 + 1) ≥ 0 (𝑥 − 3)(𝑥 + 3) ≤ 0
𝑥 ≤ −1 𝑎𝑡𝑎𝑢 𝑥 ≥ 1 −3 ≤ 𝑥 ≤ 3
Menuliskan himpunan penyelesaian
𝑆 = {𝑥 | − 3 ≤ 𝑥 ≤ −1 𝑎𝑡𝑎𝑢 1 ≤ 𝑥 ≤ 3, 𝑥 ∈ 𝐵}
𝑆 = {−3, −2, −1, 0, 1, 2, 3}

Jika N adalah himpunan bagian dari S yang beranggotakan sedikitnya 2


bilangan, maka himpunan N bisa terdiri dari 2, 3, 4, 5, 6 dan 7 anggota. Jadi
banyaknya himpunan bagian dari N dapat ditentukan dengan terlebih dahulu
menentukan jumlah himpunan bagian dari S.
Himpunan S beranggotakan 7 elemen, jumlah himpunan bagian S = 2 7 = 128.
Himpunan bukan N adalah himpunan kosong dan himpunan bagian dari S yang
mengandung satu elemen.
Himpunan kosong ada 1 dan himpunan S yang mengandung 1 elemen ada 7.
Sehingga banyaknya N = 128 – (1 + 7) = 120

16 | A L J A B A R
Olimpiade Matematika SMA 2020

• PERTAKSAMAAN AM – GM – HM

Misalkan 𝑎 dan 𝑏 adalah bilangan real positif, maka √𝑎 dan √𝑏 terdefinisi untuk
𝑎, 𝑏 ≥ 0. Maka √𝑎 − √𝑏 bisa berharga negatif, nol atau positif.

2 2
Tetapi jika (√𝑎 − √𝑏) tidak mungkin bernilai negatif, (√𝑎 − √𝑏) ≥ 0

2
(√𝑎 − √𝑏) ≥ 0

𝑎 + 𝑏 − 2√𝑎𝑏 ≥ 0

𝑎+𝑏
≥ √𝑎𝑏
2
𝑎+𝑏
disebut aritmetik mean (AM) dan √𝑎𝑏 disebut geometrik mean (GM), jadi
2
𝐴𝑀 ≥ 𝐺𝑀

Kita sudah mengetahui bahwa :


𝑎+𝑏
≥ √𝑎𝑏
2
Kalau kedua ruas dibagi 𝑎𝑏 , maka yang terjadi adalah

𝑎+𝑏 1

2𝑎𝑏 √𝑎𝑏

Atau jika dibalik :


2𝑎𝑏
≤ √𝑎𝑏
𝑎+𝑏
2
√𝑎𝑏 ≥
1 1
𝑎+𝑏

2
1 1 disebut harmonik mean (HM), jadi 𝐺𝑀 ≥ 𝐻𝑀 , Jadi kesimpulannya adalah
+
𝑎 𝑏
𝐴𝑀 ≥ 𝐺𝑀 ≥ 𝐻𝑀

𝑎+𝑏 2
≥ √𝑎𝑏 ≥
2 1 1
𝑎+𝑏

17 | A L J A B A R
Olimpiade Matematika SMA 2020

Maka jika 𝑎 , 𝑏 dan 𝑐 real positif, berlaku 𝐴𝑀 ≥ 𝐺𝑀 ≥ 𝐻𝑀 sebagai berikut :

𝑎+𝑏+𝑐 3 3
≥ √𝑎𝑏𝑐 ≥
3 1 1 1
𝑎+𝑏+𝑐
Secara umum untuk n bilangan real :

𝑥1 + 𝑥2 + 𝑥3 + ⋯ + 𝑥𝑛 𝑛
≥ 𝑛√𝑥1 𝑥2 𝑥3 … 𝑥𝑛 ≥
𝑛 1 1 1 1
+ + + ⋯+
𝑥1 𝑥2 𝑥3 𝑥𝑛

Dari bentuk persamaan diatas, untuk 𝑎 , 𝑏 dan 𝑐 real positif dapat kita lihat
hubungan-hubungan berikut ini :

3
𝐴𝑀 ≥ 𝐺𝑀 , 𝑎 + 𝑏 + 𝑐 ≥ 3 √𝑎𝑏𝑐

1 1 1
𝐴𝑀 ≥ 𝐻𝑀 , (𝑎 + 𝑏 + 𝑐) ( + + ) ≥ 9
𝑎 𝑏 𝑐

3 3
𝐺𝑀 ≥ 𝐻𝑀 , √𝑎𝑏𝑐 ≥
1 1 1
𝑎+𝑏+𝑐

Contoh Penggunaan 𝐴𝑀 ≥ 𝐺𝑀 ≥ 𝐻𝑀 untuk 𝑎, 𝑏 dan 𝑐 real positif :

(𝑎+𝑏) (𝑏+𝑐)(𝑎+𝑐)
(a). Menentukan nilai minimum dari dengan 𝐴𝑀 ≥ 𝐺𝑀
𝑎𝑏𝑐
𝑎 + 𝑏 ≥ 2√𝑎𝑏
𝑏 + 𝑐 ≥ 2√𝑏𝑐
𝑎 + 𝑐 ≥ 2√𝑎𝑐
Jika kedua ruas dikalikan, menghasilkan (𝑎 + 𝑏) (𝑏 + 𝑐)(𝑎 + 𝑐) ≥ 8√𝑎𝑏. 𝑏𝑐. 𝑎𝑐

(𝑎+𝑏) (𝑏+𝑐)(𝑎+𝑐)
≥ 8, jadi nilai minimumnya 8
𝑎𝑏𝑐
𝑎 𝑏 𝑐
(b). Menentukan nilai minimum + + dengan 𝐴𝑀 ≥ 𝐺𝑀
𝑏 𝑐 𝑎

𝑎 𝑏 𝑐 𝑎 𝑏 𝑐
+ +
𝑏 𝑐 𝑎 3 𝑎 𝑏 𝑐 + +
𝑏 𝑐 𝑎 3
≥ √ akan menjadi ≥ √1
3 𝑏 𝑐 𝑎 3
𝑎 𝑏 𝑐
+ + ≥ 3, jadi nilai minimumnya 3
𝑏 𝑐 𝑎

18 | A L J A B A R
Olimpiade Matematika SMA 2020
1 1 1 𝑎 𝑏 𝑐
(c). Pembuktian (𝑎 + 𝑏 + 𝑐) ( + + ) ( + + ) ≥ 27
𝑎 𝑏 𝑐 𝑏 𝑐 𝑎

𝑎, 𝑏, 𝑐  𝑅 + maka menurut 𝐴𝑀 ≥ 𝐻𝑀 berlaku bahwa :

𝑎+𝑏+𝑐 3 1 1 1
≥ 1 1 1 atau (𝑎 + 𝑏 + 𝑐) (𝑎 + 𝑏 + 𝑐 ) ≥ 9 ......... (1)
3 + +
𝑎 𝑏 𝑐

𝑎, 𝑏, 𝑐  𝑅 + maka menurut 𝐴𝑀 ≥ 𝐺𝑀 berlaku :

𝑎 𝑏 𝑐 3 𝑎 𝑏 𝑐 𝑎 𝑏 𝑐
+ + ≥ 3√ atau + + ≥ 3 ....................... (2)
𝑏 𝑐 𝑎 𝑏 𝑐 𝑎 𝑏 𝑐 𝑎

Jika (1) dan (2) dikalikan, maka akan kita peroleh :

1 1 1 𝑎 𝑏 𝑐
(𝑎 + 𝑏 + 𝑐) ( + + ) ( + + ) ≥ 9 . 3
𝑎 𝑏 𝑐 𝑏 𝑐 𝑎

1 1 1 𝑎 𝑏 𝑐
(𝑎 + 𝑏 + 𝑐) ( + + ) ( + + ) ≥ 27
𝑎 𝑏 𝑐 𝑏 𝑐 𝑎

1 1 1 9
(d). Pembuktian 2 ( + + ) ≥ 𝑎+𝑏+𝑐
𝑎+𝑏 𝑏+𝑐 𝑎+𝑐

Menurut 𝐴𝑀 ≥ 𝐻𝑀 berlaku :

1 1 1
+ + 3
𝑎+𝑏 𝑏+𝑐 𝑎+𝑐 ≥
3 1 1 1
1 + 1 + 1
𝑎+𝑏 𝑏+𝑐 𝑎+𝑐

1 1 1 9
+ + ≥
𝑎+𝑏 𝑏+𝑐 𝑎+𝑐 𝑎+𝑏+𝑏+𝑐+𝑎+𝑐

1 1 1 9
+ + ≥
𝑎 + 𝑏 𝑏 + 𝑐 𝑎 + 𝑐 2 (𝑎 + 𝑏 + 𝑐)

1 1 1 9
2( + + ) ≥
𝑎+𝑏 𝑏+𝑐 𝑎+𝑐 𝑎+𝑏+𝑐

19 | A L J A B A R
Olimpiade Matematika SMA 2020

Contoh soal 14
1 1 1
Misalkan 𝑎, 𝑏, 𝑐 bilangan real positif dengan + + =3
𝑎 𝑏 𝑐
4
Buktikan bahwa 𝑎 + 𝑏 + 𝑐 + 2 ≥ 5
1+ (𝑎𝑏𝑐)3
Pembahasan :
Berdasarkan 𝐴𝑀 ≥ 𝐺𝑀

1 1 1 3 1
+ + ≥ 3√
𝑎 𝑏 𝑐 𝑎𝑏𝑐

1 1 1 3 1
+ + = 3 , maka √ ≤ 1 , sehingga 𝑎𝑏𝑐 ≥ 1
𝑎 𝑏 𝑐 𝑎𝑏𝑐

3
Berdasarkan 𝐴𝑀 ≥ 𝐺𝑀 , 𝑎 + 𝑏 + 𝑐 ≥ 3√𝑎𝑏𝑐 sehingga 𝑎 + 𝑏 + 𝑐 ≥ 3

4 4
𝑎+𝑏+𝑐+ 2 ≥ (3) + 2 , dengan mengambil 𝑎𝑏𝑐 = 1
1+ (𝑎𝑏𝑐)3 1 + (1)3

4
𝑎+𝑏+𝑐+ 2 ≥ 5 (Terbukti)
1+ (𝑎𝑏𝑐)3

Contoh soal 15
Bilangan real terbesar M sehingga untuk setiap x positif berlaku
(𝑥 + 1)(𝑥 + 3)(𝑥 + 5)(𝑥 + 11) ≥ 𝑀𝑥
adalah ....

Pembahasan :
(𝑥 + 1)(𝑥 + 3)(𝑥 + 5)(𝑥 + 11) = 𝑥 4 + 20𝑥 3 + 122𝑥 2 + 268𝑥 + 165
Berdasarkan 𝐴𝑀 ≥ 𝐺𝑀

𝑥 4 + 20𝑥 3 + 122𝑥 2 + 268𝑥 + 165 𝟏+𝟐𝟎+𝟏𝟐𝟐+𝟐𝟔𝟖+𝟏𝟔𝟓


≥ √𝑥 4 (𝑥 3 )20 (𝑥 3 )20 (𝑥 2 )122 . 𝑥 20 . 1165
1 + 20 + 122 + 268 + 165

𝑥 4 + 20𝑥 3 + 122𝑥 2 + 268𝑥 + 165 576


≥ √𝑥 576
576

𝑥 4 + 20𝑥 3 + 122𝑥 2 + 268𝑥 + 165 ≥ 576𝑥


Jadi 𝑀 = 576

20 | A L J A B A R
Olimpiade Matematika SMA 2020

Contoh soal 16
Bilangan real tak negatif 𝑎, 𝑏, 𝑐, 𝑑, 𝑒 dengan 𝑎𝑏 + 𝑏𝑐 + 𝑐𝑑 + 𝑑𝑒 = 2020. Nilai
minimum dari 𝑎 + 𝑏 + 𝑐 + 𝑑 + 𝑒 adalah ...
Pembahasan :
Berdasarkan AM – GM diperoleh

𝑎 + 𝑏 + 𝑐 + 𝑑 + 𝑒 ≥ 2 √(𝑎 + 𝑐 + 𝑒)(𝑏 + 𝑑)

𝑎 + 𝑏 + 𝑐 + 𝑑 + 𝑒 ≥ 2 √𝑎𝑏 + 𝑎𝑑 + 𝑏𝑐 + 𝑐𝑑 + 𝑏𝑒 + 𝑑𝑒

𝑎 + 𝑏 + 𝑐 + 𝑑 + 𝑒 ≥ 2 √(𝑎𝑏 + 𝑏𝑐 + 𝑐𝑑 + 𝑑𝑒) + 𝑎𝑑 + 𝑑𝑒

𝑎 + 𝑏 + 𝑐 + 𝑑 + 𝑒 ≥ 2 √2020 + 𝑎𝑑 + 𝑑𝑒

Karena yang diminta adalah nilai minimum dimana 𝑎, 𝑏, 𝑐, 𝑑, 𝑒 bilangan real


tak negatif, maka pada bentuk 2020 + 𝑎𝑑 + 𝑑𝑒 dapat dianggap bahwa 𝑎𝑑 +
𝑑𝑒 = 0 dengan mengambil asumsi 𝑑 bernilai 0.

Sehingga nilai minimum 𝑎 + 𝑏 + 𝑐 + 𝑑 + 𝑒 adalah 2 √2020

Contoh soal 17
2𝑎 2𝑏 2𝑐
Jika 𝑎, 𝑏, 𝑐  𝑅 + , buktikan + + ≥3
𝑏+𝑐 𝑎+𝑐 𝑎+𝑏
Pembahasan :
Menurut 𝐴𝑀 ≥ 𝐺𝑀 berlaku :

2𝑎 2𝑏 2𝑐 3 2𝑎 2𝑏 2𝑐
+ + ≥3√
𝑏+𝑐 𝑎+𝑐 𝑎+𝑏 𝑏+𝑐 𝑎+𝑐 𝑎+𝑏

2𝑎 2𝑏 2𝑐 3 8𝑎𝑏𝑐
+ + ≥3√
𝑏+𝑐 𝑎+𝑐 𝑎+𝑏 (𝑎 + 𝑏)(𝑏 + 𝑐)(𝑎 + 𝑐)

𝑎𝑏𝑐 1
Kita dapat mengambil bahwa
(𝑎+𝑏)(𝑏+𝑐)(𝑎+𝑐)
=8

2𝑎 2𝑏 2𝑐 3 1
+ + ≥ 3 √8 ( )
𝑏+𝑐 𝑎+𝑐 𝑎+𝑏 8

2𝑎 2𝑏 2𝑐
+ + ≥3
𝑏+𝑐 𝑎+𝑐 𝑎+𝑏

21 | A L J A B A R
Olimpiade Matematika SMA 2020

• PERTAKSAMAAN CHAUCY SCHWART

Untuk sembarang bilangan real 𝑥1 , 𝑥2 , 𝑥3 … . 𝑥𝑛 dan 𝑦1 , 𝑦2 , 𝑦3 … . . , 𝑦𝑛 maka


menurut Chaucy Schwarz berlaku :

(𝑥12 + 𝑥22 + ⋯ + 𝑥𝑛2 ) (𝑦12 + 𝑦22 + ⋯ + 𝑦𝑛2 ) ≥ (𝑥1 𝑦1 + 𝑥2 𝑦2 + ⋯ + 𝑥𝑛 𝑦𝑛 )2

dan Persamaan Chaucy Schwarz Engel (CS-Engel)

𝑥12 𝑥22 𝑥32 𝑥𝑛2


( + + + ⋯ . + ) (𝑦1 + 𝑦2 + 𝑦3 + ⋯ . . + 𝑦𝑛 ) ≥ (𝑥1 + 𝑥2 + 𝑥3 + ⋯ . +𝑥𝑛 )2
𝑦1 𝑦2 𝑦3 𝑦𝑛

Atau ditulis dalam bentuk :

𝑥12 𝑥22 𝑥32 𝑥𝑛2 (𝑥1 + 𝑥2 + 𝑥3 + ⋯ . +𝑥𝑛 )2


+ + +⋯.+ ≥
𝑦1 𝑦2 𝑦3 𝑦𝑛 𝑦1 + 𝑦2 + 𝑦3 + ⋯ . . + 𝑦𝑛

𝑥1 𝑥2 𝑥3 𝑥𝑛
Kesamaan terjadi jika = = =⋯=
𝑦1 𝑦2 𝑦3 𝑦𝑛

Contoh Soal 18
Dikethui a, b, x dan y adalah bilangan real positif.
Jika 𝑎2 + 𝑏 2 = 1 dan 𝑥 2 + 𝑦 2 = 1 , tentukan nilai maksimum dari 𝑎𝑥 + 𝑏𝑦
dengan cara pertaksamaan AM – GM dan pertaksamaan Chaucy Schwarz.

Pembahasan :
Dengan pertaksamaan AM – GM
Untuk mendapatkan 𝑎𝑥 + 𝑏𝑦 maka
(𝑎 − 𝑥)2 ≥ 0 dan (𝑏 − 𝑦)2 ≥ 0

𝑎2 +𝑥 2
≥ 𝑎𝑥 atau 𝑎2 + 𝑥 2 ≥ 2𝑎𝑥 ........... (1)
2

𝑏2 +𝑦 2
≥ 𝑏𝑦 atau 𝑏2 + 𝑦 2 ≥ 2𝑏𝑦 ........... (2)
2

Dari penjumlahan (1) dan (2), diperoleh :


𝑎2 + 𝑥 2 + 𝑏 2 + 𝑦 2 ≥ 2𝑎𝑥 + 2𝑏𝑦
𝑎2 + 𝑏 2 + 𝑥 2 + 𝑦 2 ≥ 2(𝑎𝑥 + 𝑏𝑦)
1 + 1 ≥ 2(𝑎𝑥 + 𝑏𝑦)
𝑎𝑥 + 𝑏𝑦 ≤ 1 , nilai maksimum 𝑎𝑥 + 𝑏𝑦 adalah 1

22 | A L J A B A R
Olimpiade Matematika SMA 2020

Dengan pertaksamaan Chaucy Schwarz

(𝑥12 + 𝑥22 ) (𝑦12 + 𝑦22 ) ≥ (𝑥1 𝑦1 + 𝑥2 𝑦2 )2

(𝑎2 + 𝑏 2 ) (𝑥 2 + 𝑦 2 ) ≥ (𝑎𝑥 + 𝑏𝑦)2


1 . 1 ≥ (𝑎𝑥 + 𝑏𝑦)2

(𝑎𝑥 + 𝑏𝑦)2 ≤ 1
𝑎𝑥 + 𝑏𝑦 ≤ 1 , nilai maksimum 𝑎𝑥 + 𝑏𝑦 adalah 1

Contoh Soal 19
Buktikan bahwa dengan pertaksamaan CS-Engel untuk 𝑎, 𝑏, 𝑐 real berlaku :

𝑎3 + 𝑏 3 + 𝑐 3 𝑎 + 𝑏 + 𝑐

𝑎2 + 𝑏 2 + 𝑐 2 3

Pembahasan :
𝑥12 𝑥22 𝑥32
( + + ) (𝑦1 + 𝑦2 + 𝑦3 ) ≥ (𝑥1 + 𝑥2 + 𝑥3 )2
𝑦1 𝑦2 𝑦3

𝑎4 𝑏 4 𝑐 4 (𝑎2 )2 (𝑏 2 )2 (𝑐 2 )2
𝑎3 + 𝑏 3 + 𝑐 3 = + + = + +
𝑎 𝑏 𝑐 𝑎 𝑏 𝑐
(𝑎2 )2 (𝑏 2 )2 (𝑐 2 )2
( + + ) (𝑎 + 𝑏 + 𝑐) ≥ (𝑎2 + 𝑏 2 + 𝑐 2 )2
𝑎 𝑏 𝑐

Ketika bentuk persamaan dikembalikan semula akan kita lihat :

(𝑎3 + 𝑏 3 + 𝑐 3 )(𝑎 + 𝑏 + 𝑐) ≥ (𝑎2 + 𝑏 2 + 𝑐 2 )2

𝑎3 + 𝑏 3 + 𝑐 3 𝑎2 + 𝑏 2 + 𝑐 2

𝑎2 + 𝑏 2 + 𝑐 2 𝑎+𝑏+𝑐
(𝑎2 +𝑏2 +𝑐 2 )
Langkah berikutnya mengubah bentuk dengan cara yang sama :
(𝑎+𝑏+𝑐)

𝑎2 𝑏 2 𝑐 2
( + + ) (1 + 1 + 1) ≥ (𝑎 + 𝑏 + 𝑐)2
1 1 1

𝑎2 + 𝑏 2 + 𝑐 2 𝑎 + 𝑏 + 𝑐

𝑎+𝑏+𝑐 3

𝑎2 +𝑏2 +𝑐 2 𝑎+𝑏+𝑐 𝑎3 +𝑏3 +𝑐 3 𝑎+𝑏+𝑐


Karena ≥ maka terbukti bahwa ≥
𝑎+𝑏+𝑐 3 𝑎2 +𝑏2 +𝑐 2 3

23 | A L J A B A R
Olimpiade Matematika SMA 2020

• POLA BILANGAN

Pola bilangan adalah sederetan bilangan baik dalam susunan baris atau kolom
yang memiliki kesamaan khas atau pola khusus yang dapat diamati. Misalkan
kita memiliki bilangan-bilangan Asli dibentuk suatu pola piramid seperti
ilustrasi di bawah ini.

Jika suatu bilangan yang terletak pada baris ke n dan ada pada urutan ke m
𝑛
dinyatakan sebagai ( ) dengan 𝑚 ≤ 2𝑛 − 1.
𝑚
31 26 22
maka kita akan menentukan bilangan berapakah pada ( ) + ( ) + ( ) ?
20 15 19

Perhatikan pola tersebut bilangan pada soal diatas.


Baris ke 1 12
Baris ke 2 12 + 1 12 + 2 22
Baris ke 3 22 + 1 22 + 2 22 + 3 22 + 4 32
Baris ke 4 32 + 1 32 + 2 32 + 3 32 + 4 32 + 5 .....
................
................
................
Baris ke n (n – 1)2 + m
𝑛
Jadi ( ) = (𝑛 − 1)2 + 𝑚
𝑚

Jadi bilangan pada baris ke n dan urutan ke m dari kiri, adalah (𝑛 − 1)2 + 𝑚
31
Jadi notasi ( ) adalah posisi bilangan pada baris ke 31 dan urutan ke 20.
20
31 26 22
( ) + ( ) + ( ) = ((31 − 1)2 + 20) + ((26 − 1)2 + 15) + ((22 − 1)2 + 19)
20 15 19
31 26 22
( ) + ( ) + ( ) = (920) + (640) + (460)
20 15 19
31 26 22
( ) + ( ) + ( ) = 2020
20 15 19

24 | A L J A B A R
Olimpiade Matematika SMA 2020

Barisan dan Deret


Pada umumnya telah dikenal pola bilangan yang disebut barisan aritmatika dan
Geometri.

Pada barisan aritmatika dengan suku pertama 𝑎 dan beda antar sukunya 𝑏, maka
suku ke n dalam barisan dinyatakan dengan 𝑈𝑛

𝑏 = 𝑈𝑛 − 𝑈𝑛−1

𝑈𝑛 = 𝑈𝑛−1 + 𝑏

𝑈𝑛+1 = 𝑈𝑛 + 𝑏

𝑎, 𝑎 + 𝑏, 𝑎 + 2𝑏, 𝑎 + 3𝑏, … …. Sehingga 𝑈𝑛 = 𝑎 + (𝑛 − 1)𝑏


Rumus seperti ini disebut sebagai rumus rekursif, yaitu nilai dari suku ke n
diperoleh dari suku sebelumnya.

Begitu pula pada barisan geometri, suku ke n dapat diperoleh dari suku
sebelumnya dengan adanya pembanding (rasio = r) , suku pertama a, maka Un
dapat diturunkan dengan teknik yang sama, yaitu 𝑈𝑛 = 𝑎 𝑟 𝑛−1 .

Sebagai contoh menentukan pembanding atau rasio (r) pada barisan geometri,
misalnya ada tiga suku berurutan dalam barisan geometri yaitu :
𝑎2 + 𝑏 , 3𝑎2 − 2𝑏 , 7𝑎2 − 8𝑏
Untuk menentukan rasio dari barisan tersebut, terlebih dahulu kita cari selisih
antar suku yang berdekatan.

𝑈3 − 𝑈2 = (7𝑎2 − 8𝑏) − (3𝑎2 − 2𝑏) = 4𝑎2 − 6𝑏

𝑈2 − 𝑈1 = (3𝑎2 − 2𝑏) − (𝑎2 + 𝑏) = 2𝑎2 − 3𝑏

4𝑎2 −6𝑏
Maka rasio (r) = =2
2𝑎2 −3𝑏

Atau bisa dengan cara yang lebih umum , 𝑈22 = 𝑈1 . 𝑈3

(3𝑎2 − 2𝑏)2 = (𝑎2 + 𝑏 )(7𝑎2 − 8𝑏)


9𝑎4 − 12𝑎2 𝑏 + 4𝑏 2 = 7𝑎2 − 𝑎2 𝑏 − 8𝑏 2
2𝑎4 − 11𝑎2 𝑏 + 12𝑏 2 = 0
(2𝑎2 − 3𝑏)(𝑎2 − 4𝑏) = 0

Jika kita ambil salah satu nilai 𝑎2 = 4𝑏 maka deret itu menjadi 5b, 10b , 20b
sehingga memiliki rasio 2.

25 | A L J A B A R
Olimpiade Matematika SMA 2020

Barisan aritmatika dengan beda berpola atau bertingkat sering kita jumpai dalam
permasalahan yang tidak umum. Perhatikan soal berikut dalam menentukan
rumus umum 𝑈𝑛 dan 𝑆𝑛 pada pola bilangan barisan aritmatika dengan beda
berpola.

Contoh Soal 20
Berikut adalah pola bilangan secara berurutan
2 , 15 , 34 , 61 , 98 , … ..
Jika suku ke-𝑛 dan jumlah 𝑛 suku pertama pola bilangan itu masing-masing 𝑈𝑛
𝑛 𝑈𝑛 −𝑆𝑛 1
dan 𝑆𝑛 sehingga = (𝑛2 + 𝑏𝑛 + 𝑐 ), maka nilai 𝑎 + 𝑏 + 𝑐 adalah ....
𝑛 (𝑛−1) 𝑎

Pembahasan :
Perhatikan pola bilangan pertama dari atas ke bawah :

2 , 15 , 34 , 61 , 98 , … ..

13 19 27 37 ................

6 8 10 ..............

2 2 ..............

Bilangan pertama setiap pola deret ke bawah adalah 2, 13, 6 dan 2 digunakan
untuk menentukan rumus suku ke-n dan rumus jumlah n suku pertama.

2 13 (𝑛 − 1) 6 (𝑛 − 1)(𝑛 − 2) 2 (𝑛 − 1)(𝑛 − 2)(𝑛 − 3)


𝑈𝑛 = + + +
0! 1! 2! 3!

1
𝑈𝑛 = 2 + 13 (𝑛 − 1) + 3(𝑛 − 1)(𝑛 − 2) + (𝑛 − 1)(𝑛 − 2)(𝑛 − 3)
3

Untuk menentukan rumus jumlah n suku pertama, kita turunkan berdasarkan


pola diatas.

2𝑛 13𝑛 (𝑛 − 1) 6𝑛 (𝑛 − 1)(𝑛 − 2) 2𝑛 (𝑛 − 1)(𝑛 − 2)(𝑛 − 3)


𝑆𝑛 = + + +
1! 2! 3! 4!
13 1
𝑆𝑛 = 2𝑛 + 𝑛(𝑛 − 1) + 𝑛(𝑛 − 1)(𝑛 − 2) + 𝑛(𝑛 − 1)(𝑛 − 2)(𝑛 − 3)
2 12

26 | A L J A B A R
Olimpiade Matematika SMA 2020

𝑛𝑈𝑛 − 𝑆𝑛 dapat diperoleh dengan cara berikut :

1
𝑛𝑈𝑛 = 2𝑛 + 13𝑛 (𝑛 − 1) + 3𝑛(𝑛 − 1)(𝑛 − 2) + 𝑛(𝑛 − 1)(𝑛 − 2)(𝑛 − 3)
3

13 1
𝑆𝑛 = 2𝑛 + 𝑛(𝑛 − 1) + 𝑛(𝑛 − 1)(𝑛 − 2) + 𝑛(𝑛 − 1)(𝑛 − 2)(𝑛 − 3)
2 12

13 1
𝑛𝑈𝑛 − 𝑆𝑛 = 𝑛(𝑛 − 1) + 2𝑛(𝑛 − 1)(𝑛 − 2) + 𝑛(𝑛 − 1)(𝑛 − 2)(𝑛 − 3)
2 4

13 1
𝑛𝑈𝑛 − 𝑆𝑛 𝑛(𝑛 − 1) + 2𝑛(𝑛 − 1)(𝑛 − 2) + 4 𝑛(𝑛 − 1)(𝑛 − 2)(𝑛 − 3)
= 2
𝑛 (𝑛 − 1) 𝑛 (𝑛 − 1)

Bentuk persamaan terakhir menjadi lebih sederhana :

𝑛𝑈𝑛 − 𝑆𝑛 13 1
= + 2(𝑛 − 2) + (𝑛 − 2)(𝑛 − 3)
𝑛 (𝑛 − 1) 2 4

𝑛𝑈𝑛 − 𝑆𝑛 26 8(𝑛 − 2) (𝑛 − 2)(𝑛 − 3)


= + +
𝑛 (𝑛 − 1) 4 4 4

𝑛𝑈𝑛 − 𝑆𝑛 1
= (26 + 8(𝑛 − 2) + (𝑛 − 2)(𝑛 − 3))
𝑛 (𝑛 − 1) 4

𝑛𝑈𝑛 − 𝑆𝑛 1
= (26 + 8𝑛 − 16 + 𝑛2 − 5𝑛 + 6)
𝑛 (𝑛 − 1) 4

𝑛𝑈𝑛 − 𝑆𝑛 1 2
= (𝑛 + 3𝑛 + 16)
𝑛 (𝑛 − 1) 4

𝑛𝑈𝑛 −𝑆𝑛 1
Berdasarkan = (𝑛2 + 𝑏𝑛 + 𝑐)
𝑛 (𝑛−1) 𝑎

Maka 𝑎 = 4 , 𝑏 = 3 dan 𝑐 = 16
Nilai 𝑎 + 𝑏 + 𝑐 = 23

27 | A L J A B A R
Olimpiade Matematika SMA 2020

• POLINOMIAL

Polinomial atau suku banyak berderajat n memiliki bentuk umum :

𝑓(𝑥) = 𝑎𝑛 𝑥 𝑛 + 𝑎𝑛−1 𝑥 𝑛−1 + 𝑎𝑛−2 𝑥 𝑛−2 + ⋯ + 𝑎2 𝑥 2 + 𝑎1 𝑥 + 𝑎𝑜

Jika 𝑥1 , 𝑥2 , 𝑥3 , 𝑥4 , … … . 𝑥𝑛 adalah akar-akar dari suku banyak 𝑓(𝑥).

Algoritma Pembagian

Misalkan f(x) dan g(x) dengan g(x) bukan suku banyak nol. Maka terdapat suku
banyak q(x) dan r(x) yang memenuhi :
f(x) = q(x)g(x) + r(x)
dengan r(x) merupakan suku bayak nol atau r(x) bukan suku banyak nol yang
berderajat kurang dari derajat g(x).

Dalam hal tersebut, q(x) disebut hasil bagi dan r(x) disebut sisa pembagian.
Selanjutnya jika r(x) merupakan suku banyak nol maka dikatakan f(x) habis
dibagi oleh g(x).

Jika suku banyak f(x) dibagi oleh (x−a) maka sisanya adalah f(a). Bilangan a di f
disebut akar dari suku banyak f(x) jika f(a) = 0.
Suku banyak f(x) habis dibagi oleh (x − a) jika dan hanya jika a merupakan akar
dari f(x)

Contoh Soal 21
Tentukan semua bilangan Asli n sehingga suku banyak 𝑥 2 + 𝑥 + 1 membagi
habis suku banyak 𝑥 2𝑛 + 𝑥 𝑛 + 1.

Pembahasan :
Mengingat 𝑥 3 − 1 = (𝑥 − 1)(𝑥 2 + 𝑥 + 1) dan 𝑥 3𝑚 − 1 habis dibagi 𝑥 3 − 1.
Kita akan menggunakan metoda induksi.

Untuk n = 3k
𝑥 2𝑛 + 𝑥 𝑛 + 1 = 𝑥 6𝑘 + 𝑥 3𝑘 + 1
= (𝑥 6𝑘 − 1) + (𝑥 3𝑘 − 1) + 3
= (𝑥 2 + 𝑥 + 1) 𝐴(𝑥) + 3
Untuk n kelipatan 3 tersebut, ternyata ada sisa pembagian.

28 | A L J A B A R
Olimpiade Matematika SMA 2020

Untuk n = 3k+1
𝑥 2𝑛 + 𝑥 𝑛 + 1 = 𝑥 6𝑘+2 + 𝑥 3𝑘+1 + 1
= 𝑥 2 (𝑥 6𝑘 − 1) + 𝑥(𝑥 3𝑘 − 1) + 𝑥 2 + 𝑥 + 1
= (𝑥 2 + 𝑥 + 1) 𝐵(𝑥)

Untuk n = 3k+2
𝑥 2𝑛 + 𝑥 𝑛 + 1 = 𝑥 6𝑘+4 + 𝑥 3𝑘+2 + 1
= 𝑥 4 (𝑥 6𝑘 − 1) + 𝑥 2 (𝑥 3𝑘 − 1) + 𝑥 4 + 𝑥 2 + 1
= 𝑥 4 (𝑥 6𝑘 − 1) + 𝑥 2 (𝑥 3𝑘 − 1) + 𝑥(𝑥 3 − 1) + 𝑥 2 + 𝑥 + 1
= (𝑥 2 + 𝑥 + 1) 𝐶(𝑥)

Jadi 𝑥 2 + 𝑥 + 1 membagi 𝑥 2𝑛 + 𝑥 𝑛 + 1 jika dan hanya jika n bukan kelipatan 3.

Teorema Vieta
Sifat yang lain dari suku banyak yang sering digunakan adalah sifat simetri akar,
yang lebih dikenal dengan nama Teorema Vieta, yaitu hasil tambah dan hasil kali
akar-akar suatu suku banyak.

𝑓(𝑥) = 𝑎𝑛 𝑥 𝑛 + 𝑎𝑛−1 𝑥 𝑛−1 + 𝑎𝑛−2 𝑥 𝑛−2 + ⋯ + 𝑎2 𝑥 2 + 𝑎1 𝑥 + 𝑎𝑜

𝑎𝑛−1
𝑥1 + 𝑥2 + 𝑥3 + 𝑥4 + … … . + 𝑥𝑛 = −
𝑎𝑛
𝑎𝑛−2
(𝑥1 𝑥2 + ⋯ + 𝑥1 𝑥𝑛 ) + (𝑥2 𝑥3 + ⋯ + 𝑥2 𝑥𝑛 ) + (𝑥3 𝑥4 + ⋯ + 𝑥3 𝑥𝑛 ) + 𝑥𝑛−1 𝑥𝑛 =
𝑎𝑛
𝑎𝑜
𝑥1 , 𝑥2 , 𝑥3 , 𝑥4 , … … . 𝑥𝑛 = (−1)𝑛
𝑎𝑛

Jika suku banyak memiliki akar-akar 𝛼, 𝛽,  maka hubungan akar-akar itu


berdasarkan teorema Vieta adalah :

𝑓(𝑥) = 𝑎𝑥 3 + 𝑏𝑥 2 + 𝑐𝑥 + 𝑑
𝑏
𝛼+𝛽+=−
𝑎
𝑐
𝛼𝛽 + 𝛽 + 𝛼 =
𝑎
𝑑
𝛼𝛽 = −
𝑎

29 | A L J A B A R
Olimpiade Matematika SMA 2020

Contoh Soal 22
Misalkan 𝛼, 𝛽,  adalah akar-akar dari 𝑥 3 − 2𝑥 + 7 = 0.
Tentukan nilai dari :

(a). 𝛼 3 + 𝛽 3 + 3
1 1 1
(b). + +
𝛼2 𝛽2 2

1 1 1
(c). + +
𝛼3 𝛽3 3

Pembahasan :
𝑥 3 − 2𝑥 + 7 = 0 akar-akarnya 𝛼, 𝛽, . Menurut teorema Vieta
0
𝛼+ 𝛽+ =− =0
1
(a). Menentukan nilai 𝛼 3 + 𝛽 3 + 3
𝑥 3 − 2𝑥 + 7 = 0 , dapat ditulis menjadi 𝑥 3 = 2𝑥 − 7
maka :
𝛼 3 = 2𝛼 − 7
𝛽 3 = 2𝛽 − 7
3 = 2 − 7

𝛼 3 + 𝛽 3 + 3 = 2𝛼 − 7 + 2𝛽 − 7 + 2 − 7
𝛼 3 + 𝛽 3 + 3 = 2(𝛼 + 𝛽 + ) − 21
𝛼 3 + 𝛽 3 + 3 = 2(0) − 21 = −21

1 1 1
(b). Menentukan nilai + +
𝛼2 𝛽2 2

1 1 1 𝛼𝛽 + 𝛽 + 𝛼 −2 2
+ + = = =
𝛼 𝛽  𝛼𝛽 −7 7

1 1
𝑥 3 − 2𝑥 + 7 = 0 , dapat ditulis menjadi 7 (𝑥 2 ) = 2 (𝑥) − 𝑥
maka :
1 1
7 (𝛼2) = 2 (𝛼) − 𝛼

1 1
7 ( 2 ) = 2 (𝛽) − 𝛽
𝛽

1 1
7 (2) = 2 (  ) − 

30 | A L J A B A R
Olimpiade Matematika SMA 2020
1 1 1 1 1 1
7( + + 2) = 2 ( ) − 𝛼 + 2 ( ) − 𝛽 + 2 ( ) − 
𝛼 2 𝛽2  𝛼 𝛽 

1 1 1 1 1 1
7( + + 2 ) = 2 ( + + ) − (𝛼 + 𝛽 + )
𝛼2 𝛽2  𝛼 𝛽 

1 1 1 2
7( + + 2 ) = 2 ( ) − (0)
𝛼2 𝛽2  7

1 1 1 4
+ + =
𝛼2 𝛽2 2 49

1 1 1
(c). Menentukan nilai + +
𝛼3 𝛽3 3

1 1
𝑥 3 − 2𝑥 + 7 = 0 , dapat ditulis menjadi 7 (𝑥 3 ) = 2 (𝑥 2 ) − 1

1 1 1 1 1 1
7( + + 3 ) = 2 ( 2) − 1 + 2 ( 2) − 1 + 2 ( 2) − 1
𝛼 3 𝛽3  𝛼 𝛽 

1 1 1 1 1 1
7( + + 3) = 2 ( 2 + 2 + 2) − 3
𝛼3 𝛽3  𝛼 𝛽 

1 1 1 4
7( + + 3) = 2 ( ) − 3
𝛼3 𝛽3  49

1 1 1 1 8 147 139
+ + = ( − ) = − 343
𝛼3 𝛽3 3 7 49 49

Hubugan akar-akar persamaan kubik menurut teorema Vieta :

𝛼 2 + 𝛽 2 = (𝛼 + 𝛽)2 − 2𝛼𝛽

𝛼 2 + 𝛽 2 + 2 = (𝛼 + 𝛽 + )2 − 2 (𝛼𝛽 + 𝛽 + 𝛼)

𝛼 3 + 𝛽 3 = (𝛼 + 𝛽)3 − 3𝛼𝛽(𝛼 + 𝛽)

𝛼 3 + 𝛽 3 + 3 = (𝛼 + 𝛽 + )3 − 3(𝛼𝛽 + 𝛼 + 𝛽)(𝛼 + 𝛽 + ) + 3𝛼𝛽

31 | A L J A B A R
Olimpiade Matematika SMA 2020

Contoh Soal 23
Untuk setiap pasangan bilangan real (𝑥 , 𝑦) memenuhi persamaan :
2
log(2𝑥 + 𝑦) = 4log(𝑥 2 + 𝑥𝑦 + 7𝑦 2 )
Terdapat konstanta real 𝑘 sedemikian sehingga
3
log(3𝑥 + 𝑦) = 9log(3𝑥 2 + 4𝑥𝑦 + 𝑘𝑦 2 )
Tentukan hasil kali semua nilai 𝑘 yang mungkin !

Pembahasan :
𝑛
Dengan menggunakan sifat logaritma 𝑎 log 𝑏 𝑛 = 𝑎log 𝑏 maka dapat kita ubah
2
log(2𝑥 + 𝑦) = 4log(𝑥 2 + 𝑥𝑦 + 7𝑦 2 )
menjadi
2 1
log(2𝑥 + 𝑦) = 2log(𝑥 2 + 𝑥𝑦 + 7𝑦 2 )
2

(2𝑥 + 𝑦)2 = 𝑥 2 + 𝑥𝑦 + 7𝑦 2
4𝑥 2 + 4𝑥𝑦 + 𝑦 2 = 𝑥 2 + 𝑥𝑦 + 7𝑦 2
3𝑥 2 + 3𝑥𝑦 − 6𝑦 2 = 0
𝑥 2 + 𝑥𝑦 − 2𝑦 2 = 0
(𝑥 − 𝑦)(𝑥 + 2𝑦) = 0
𝑥 = 𝑦 dan 𝑥 = −2𝑦

Dari persamaan kedua


3
log(3𝑥 + 𝑦) = 9log(3𝑥 2 + 4𝑥𝑦 + 𝑘𝑦 2 )
3 1
log(3𝑥 + 𝑦) = 2log(3𝑥 2 + 4𝑥𝑦 + 𝑘𝑦 2 )
2
(3𝑥 + 𝑦)2 = 3𝑥 2 + 4𝑥𝑦 + 𝑘𝑦 2
9𝑥 2 + 6𝑥𝑦 + 𝑦 2 = 3𝑥 2 + 4𝑥𝑦 + 𝑘𝑦 2
6𝑥 2 + 2𝑥𝑦 + (1 − 𝑘)𝑦 2 = 0

Jika diambil 𝑥 = 𝑦 maka


8𝑥 2 + (1 − 𝑘)𝑥 2 = 0 , diperoleh 𝑘 = 9
Jika diambil 𝑥 = −2𝑦 maka
6(−2𝑦)2 + 2(−2𝑦)𝑦 + (1 − 𝑘)𝑦 2 = 0
24𝑦 2 − 4𝑦 2 + (1 − 𝑘)𝑦 2 = 0
20𝑦 2 + (1 − 𝑘)𝑦 2 = 0 , diperoleh 𝑘 = 21
Maka hasil kali semua nilai 𝑘 adalah 9 x 21 = 189

32 | A L J A B A R
Olimpiade Matematika SMA 2020

Contoh Soal 24
Misalkan 𝑝(𝑥) polinomial bukan nol sedemikian sehingga
(𝑥 − 1)𝑝(𝑥 + 1) = (𝑥 + 2)𝑝(𝑥)
7 𝑚
Untuk setiap nilai 𝑥 real. Dan (𝑝(2))2 = 𝑝(3) dan 𝑝 (2) = dimana 𝑚 dan 𝑛
𝑛
relatif prima. Tentukan 𝑚 + 𝑛 !

Pembahasan :

Untuk 𝑥 = 1 , maka (1 − 1)𝑝(1 + 1) = (1 + 2)𝑝(1) , diperoleh 𝑝(1) = 0


Untuk 𝑥 = 0 , maka (0 − 1)𝑝(0 + 1) = (0 + 2)𝑝(0) , diperoleh 𝑝(0) = 0
Untuk 𝑥 = −1 , maka (−1 − 1)𝑝(−1 + 1) = (−1 + 2)𝑝(−1) , diperoleh 𝑝(−1) = 0
Jadi 𝑝(𝑥) = 𝑥 (𝑥 − 1)(𝑥 + 1)𝑄(𝑥) untuk polinom tertentu 𝑄(𝑥)

Maka dengan menggunakan persamaan semula


(𝑥 − 1)𝑝(𝑥 + 1) = (𝑥 + 2)𝑝(𝑥)
(𝑥 − 1)[(𝑥 + 1)(𝑥 + 1 − 1)(𝑥 + 1 + 1)𝑄(𝑥 + 1)] = (𝑥 + 2)[𝑥(𝑥 − 1)(𝑥 + 1)𝑄(𝑥)]
(𝑥 − 1)(𝑥 + 1)𝑥 (𝑥 + 2)𝑄(𝑥 + 1) = (𝑥 + 2) (𝑥)(𝑥 − 1)(𝑥 + 1)𝑄(𝑥)
𝑄(𝑥 + 1) = 𝑄(𝑥), menyebabkan 𝑄(𝑥 + 1) = 𝑄(𝑥) = 𝐶 dengan C suatu konstanta

Sehingga : 𝑝(𝑥) = 𝐶 𝑥 (𝑥 − 1)(𝑥 + 2)


Ketika (𝑝(2))2 = 𝑝(3) maka

(𝐶. 2. (2 − 1)(2 + 1))2 = 𝐶 . 3. (3 − 1)(3 + 1)


(6𝐶)2 = 24 𝐶
2
𝐶 = 0 atau 𝐶 = 3

Dengan demikian
2
𝑝(𝑥) = 𝑥(𝑥 − 1)(𝑥 + 1)
3
7 𝑚
𝑝( ) =
2 𝑛

2 7 7 7 𝑚
( ) ( − 1) ( + 1) =
3 2 2 2 𝑛

𝑚 105
Diperoleh = , jadi 𝑚 + 𝑛 = 109
𝑛 4

33 | A L J A B A R
Olimpiade Matematika SMA 2020

Contoh Soal 25
Misalnya 𝑓(𝑥) adalah polinom berderajat tiga dengan koefisien bilangan real.
Jika diketahui |𝑓(1)| = |𝑓(2)| = |𝑓(3)| = |𝑓(5)| = |𝑓(6)| = |𝑓(7)| = 12
Tentukan nilai dari |𝑓(0)| !

Pembahasan :
Misalkan 𝑓(𝑥) = 𝑎𝑥 3 + 𝑏𝑥 2 + 𝑐𝑥 + 𝑑
Karena 𝑓(𝑥) polinom berderajar tiga, maka dapat kita nyatakan |𝑓(𝑥)| = 12
dapat turun naik sehingga nilai 𝑓(𝑥) = 12 dan 𝑓(𝑥) = −12
Oleh karena itu 𝑓(1) = 𝑓(5) = 𝑓(6) = 12 dan 𝑓(2) = 𝑓(3) = 𝑓(7) = −12

Untuk 𝑓(1) = 12 maka 𝑎 + 𝑏 + 𝑐 + 𝑑 = 12


Untuk 𝑓(2) = −12 maka 8𝑎 + 4𝑏 + 2𝑐 + 𝑑 = −12
Untuk 𝑓(3) = −12 maka 27𝑎 + 9𝑏 + 3𝑐 + 𝑑 = −12
Untuk 𝑓(5) = 12 maka 125𝑎 + 25𝑏 + 5𝑐 + 𝑑 = 12
Untuk 𝑓(6) = 12 maka 216𝑎 + 36𝑏 + 6𝑐 + 𝑑 = 12
Untuk 𝑓(7) = −12 maka 343𝑎 + 49𝑏 + 9𝑐 + 𝑑 = −12

Dengan sistem persamaan diatas kita akan mendapatkan nilai 𝑎, 𝑏, 𝑐 dan 𝑑


dengan cara eliminasi atau substitusi

𝑎 = −2
𝑏 = 24
𝑐 = −82
𝑑 = 72

Maka 𝑓(𝑥) = −2𝑥 3 + 24𝑥 2 − 82𝑥 + 72


dan 𝑓(0) = 72
Sehingga diperoleh |𝑓(0)| = 72

34 | A L J A B A R
Olimpiade Matematika SMA 2020

Problem Solving
AL JABAR

Problem 01
Diketahui 𝑎 + 2𝑏 = 1, 𝑏 + 3𝑐 = 3 , dan 𝑏 ≠ 0. Jika 𝑎 + 𝑛𝑏 + 2019𝑐 = 2020, maka
nilai 𝑛 adalah ......

Pembahasan :
𝑏 + 3𝑐 = 3, dikali 673 akan menjadi 673b + 2019c = 2019
673b + 2019c = 2019 dijumlahkan dengan 𝑎 + 2𝑏 = 1, akan diperoleh

𝑎 + 675𝑏 + 2019𝑐 = 2020

Analog dengan 𝑎 + 𝑛𝑏 + 2019𝑐 = 2020 , maka n adalah 675

Problem 02
Jika sebuah jam sekarang menunjukan jam 13.00 maka 2019 menit yang lalu, jam
menunjukan pukul .....

Pembahasan :
1 hari = 24 x 60 menit = 1.440 menit

2019 1440 + 579 579


2019 menit = hari = hari = (1 + 1440) hari = 1 hari + 579 menit
1440 1440

9 (24)+39
1 hari + 579 menit = 1 hari + jam = 1 hari + 9 jam + 39 menit
24

Jika sekarang jam 13.00 maka 2019 menit yang lalu sama dengan 9 jam 39 menit
yang lalu, yaitu pukul 13.00 – (9 jam 39 menit) = pukul 03.21 menit

35 | A L J A B A R
Olimpiade Matematika SMA 2020

Problem 03
(𝑎−𝑏) (𝑐−𝑑) 4 (𝑎−𝑐) (𝑏−𝑑)
Jika =− maka nilai dari adalah ....
(𝑏−𝑐) (𝑑−𝑎) 7 (𝑎−𝑏) (𝑐−𝑑)

Pembahasan :
7 (𝑎 − 𝑏) (𝑐 − 𝑑) = −4 (𝑏 − 𝑐) (𝑑 − 𝑎)
7 (𝑎𝑐 + 𝑏𝑑 − 𝑏𝑐 − 𝑎𝑑) = −4 (𝑏𝑑 + 𝑎𝑐 − 𝑎𝑏 − 𝑐𝑑)
7𝑎𝑐 + 7𝑏𝑑 − 7𝑏𝑐 − 7𝑎𝑑 = −4𝑏𝑑 − 4𝑎𝑐 + 4𝑎𝑏 + 4𝑐𝑑
11𝑎𝑐 + 11𝑏𝑑 − 7𝑏𝑐 − 7𝑎𝑑 = 4𝑎𝑏 + 4𝑐𝑑
11𝑎𝑐 + 11𝑏𝑑 − 7𝑏𝑐 − 7𝑎𝑑 + (−4𝑏𝑐 − 4𝑎𝑑) = 4𝑎𝑏 + 4𝑐𝑑 + (−4𝑏𝑐 − 4𝑎𝑑)
11𝑎𝑐 + 11𝑏𝑑 − 11𝑏𝑐 − 11𝑎𝑑 = 4𝑎𝑏 + 4𝑐𝑑 − 4𝑏𝑐 − 4𝑎𝑑
11 (𝑎𝑐 + 𝑏𝑑 − 𝑏𝑐 − 𝑎𝑑) = 4 (𝑎𝑏 + 𝑐𝑑 − 𝑏𝑐 − 𝑎𝑑)
11 (𝑎 − 𝑏) (𝑐 − 𝑑) = 4 (𝑎 − 𝑐)(𝑏 − 𝑑)
maka :
(𝑎−𝑐) (𝑏−𝑑) 11
(𝑎−𝑏) (𝑐−𝑑)
=
4

Problem 04
Nilai 𝑥 pada bentuk persamaan berikut,

√3 + 2√5 + √7 3
= (𝑥 − ) √10 − 2√21
√35 + √21 + √15 + 5 2
adalah .....
Pembahasan :
Kita selesaikan terlebih dahulu persamaan ruas kiri

√3 + 2√5 + √7 √3 + 2√5 + √7 (√7 + √5)+(√3 + √5)


= =
√35 + √21 + √15 + 5 √7 √5+√7 √3+√5 √3+√5 √5 (√7 + √5) (√3 + √5)

1 1 √7− √3
= + =
√7 + √5 √5 + √3 2

Maka dari persamaan semula :

3 3 √7− √3
(𝑥 − 2) √10 − 2√21 = (𝑥 − 2) √7 + 3 − 2√7 . 3 = 2

1 3
(√7 − √3) = (𝑥 − 2) (√7 − √3)
2

1 3
=𝑥− diperoleh 𝑥 = 2
2 2

36 | A L J A B A R
Olimpiade Matematika SMA 2020

Problem 05
Tentukan hasil dari :
1 1 1 1 1
𝐴= + 2 + 2 + 2 + ⋯……….+
12 +1 2 +2 3 +3 4 +4 2
2020 + 2020
Pembahasan :
1 1 1 1 1
𝐴 = 2 + 2 + 2 + 2 + ⋯……….+ 2
1 +1 2 +2 3 +3 4 +4 2020 +2020

1 1 1 1 1
= + + + + ⋯……….+
1(1+1) 2(2+1) 3(3+1) 4(4+1) 2020(2020+1)

1 1 1 1 1
= + + + + ⋯……….+
1 .2 2 .3 3 .4 4 .5 2020 .2021

1 1 1 1 1 1 1 1 1 1
= − + − + − + − + ⋯……….+ −
1 2 2 3 3 4 4 5 2020 2021

1 1 2020
= − =
1 2021 2021

Problem 06
Nyatakan √2 sebagai bentuk pembagian bersambung !
Pembahasan
𝑥 = √2
𝑥2 = 2
𝑥2 − 1 = 1
(𝑥 − 1)(𝑥 + 1) = 1
1
𝑥 = 1+
1+𝑥
1
𝑥 =1+
1
1+1+1+𝑥

1
𝑥 = 1+
1
2+ 1
1+1+1+𝑥
Maka
1
𝑥 = √2 = 1 +
1
2+ 1
2+ 1
2 + 2 + ⋯..

37 | A L J A B A R
Olimpiade Matematika SMA 2020

Problem 07
Tentukan hasil dari :
1 1 1 1 1
𝑃= + + + + ⋯….+
√3 + 2√2 √5 + 2√6 √7 + 2√12 √9 + 2√20 √4039 + 2√2019 𝑥 2020

Pembahasan :
1 1 1 1 1
𝑃= + + + + ⋯….+
√3 + 2√2 √5 + 2√6 √7 + 2√12 √9 + 2√20 √4039 + 2√2019 𝑥 2020

Perlu diuraikan dahulu ,

√3 + 2√2 = √2 + 1 + 2√2 𝑥 1 = √2 + √1

√5 + 2√6 = √3 + 2 + 2√3 𝑥 2 = √3 + √2
......... ......... ......... ..........

√4039 + 2√2019 𝑥 2020 = √2020 + 2019 + 2 + 2√2020 𝑥 2019 = √2020 + √2019

Sehingga P diubah menjadi :


1 1 1 1 1 1
𝑃= + + + + ⋯+ +
√2 + √1 √3 + √2 √4 + √3 √5 + √4 √2019 + √2018 √2020 + √2019

1 √2−√1
Dengan perkalian sekawan, seperti ; ( ) = √2 − √1 maka hasil
√2+√1 √2−√1
perkalian sekawan tiap suku-sukunya, menyebabkan P menjadi :

𝑃 = (√2 − 1) + (√3 − √2) + (√4 − √3) + ⋯ + (√2019 − √2018) + (√2020 − √2019)

𝑃 = √2020 − 1

38 | A L J A B A R
Olimpiade Matematika SMA 2020

Problem 08
Persamaan kuadrat 𝑥 2 − 𝑎𝑥 + 𝑏 = 0 memiliki paling sedikit satu akar bilangan
prima. Jika 𝑎 + 𝑏 = 41, banyaknya pasangan bilangan bulat (𝑎 , 𝑏) yang memenuhi
adalah ...

Pembahasan :
Pada persamaan kuadrat 𝑥 2 − 𝑎𝑥 + 𝑏 = 0 terdapat akar-akar 𝑥1 dan 𝑥2 yang salah
satu atau kedua-duanya bilangan prima.

Jumlah dan hasil kali akar-akar tersebut adalah 𝑥1 + 𝑥2 = 𝑎 dan 𝑥1 . 𝑥2 = 𝑏


maka :
𝑥1 . 𝑥2 + (𝑥1 + 𝑥2 ) = 𝑎 + 𝑏
𝑥1 . 𝑥2 + (𝑥1 + 𝑥2 ) = 41

Untuk mendapatkan solusi persamaan ini, kita tambah 1 di kedua ruas


persamaan
𝑥1 . 𝑥2 + (𝑥1 + 𝑥2 ) + 1 = 41 + 1
𝑥1 . 𝑥2 + (𝑥1 + 𝑥2 ) + 1 = 42
(𝑥1 + 1) (𝑥2 + 1) = 42

42 dapat merupakan hasil perkalian 1 x 42 , 2 x 21 , 3 x 14 atau 6 x 7


(𝑥1 + 1) (𝑥2 + 1) = 1 . 42 menghasilkan 𝑥1 = 0 dan 𝑥2 = 42 , (tidak memenuhi)
(𝑥1 + 1) (𝑥2 + 1) = 2 . 21 menghasilkan 𝑥1 = 1 dan 𝑥2 = 20 , (tidak memenuhi)
(𝑥1 + 1) (𝑥2 + 1) = 3 . 14 menghasilkan 𝑥1 = 2 dan 𝑥2 = 13 , (memenuhi)
(𝑥1 + 1) (𝑥2 + 1) = 6 . 7 menghasilkan 𝑥1 = 5 dan 𝑥2 = 6 , (memenuhi)

Kita dapat menyimpulkan bahwa :

Untuk 𝑥1 = 2 dan 𝑥2 = 13, maka :


𝑎 = 𝑥1 + 𝑥2 = 2 + 13 = 15
𝑏 = 𝑥1 . 𝑥2 = 2 x 13 = 26
Sehingga (𝑎 , 𝑏) = (15 , 26)

Solusi Untuk 𝑥1 = 5 dan 𝑥2 = 6, maka :


𝑎 = 𝑥1 + 𝑥2 = 5 + 6 = 11
𝑏 = 𝑥1 . 𝑥2 = 5 x 6 = 30
Sehingga (𝑎 , 𝑏) = (11 , 30)

Jadi ada 2 pasang nilai 𝑎 dan 𝑏 yang memenuhi.

39 | A L J A B A R
Olimpiade Matematika SMA 2020

Problem 09
Jika a = 1,3636363636 .... dan b = 1,2383838.......
𝑛
Nyatakan penjumlahan a dan b dalam bentuk dimana n  Bulat dan m  Asli.
𝑚
Pembahasan :
36 99 36 135
a = 1,3636363636 ....= 1 + = + =
99 99 99 99
38 1188 38 1226
b = 1,2383838383.... = 1,2 + = + =
990 990 990 990

135 1226 1350 1226 2576 1288


a+b= + = + = =
99 990 990 990 990 495

Problem 10
Parabola 𝑦 = 𝑎𝑥 2 − 4 dan 𝑦 = 8 − 𝑏𝑥 2 memotong sumbu koordinat pada
tepat empat titik. Keempat titik tersebut merupakan titik-titik sudut layang-
layang dengan luas 24. Nilai 𝑎 + 𝑏 adalah ....
Pembahasan :
Kurva 𝑦 = 𝑎𝑥 2 − 4 memotong sumbu-Y di
titik C(0, −4) dan kurva 𝑦 = 8 − 𝑏𝑥 2
memotong sumbu-Y di titik A(0, 8). Agar
kedua kurva tersebut memotong sumbu
koordinat pada empat titik maka kurva y1
dan y2 berpotongan tepat di sumbu- X ,
misalkan di titik B dan D. B

Luas ABC D adalah 24 dan panjang AB = 12 maka panjang BD = 4. Jadi


diperoleh B (2, 0) dan D (−2, 0). Dengan mensubstitusikan titik B (2, 0) ke
kurva y1 dan y2 berturut-turut diperoleh 𝑎 = 1 dan 𝑏 = 2.
Jadi, 𝑎 + 𝑏 = 1 + 2 = 3

40 | A L J A B A R
Olimpiade Matematika SMA 2020

Problem 11
Jika 458 merupakan jumlah kuadrat dari dua bilangan prima, juga merupakan
sisi terpanjang pada segitiga siku-siku, tentukan panjang sisi-sisi lainnya !

Pembahasan :
Kuadrat bilangan prima adalah 22, 32, 52, 72, 112, 132, 172, 192, 232, .......
458 merupakan jumlah dari 132 dan 172.
Sisi terpanjang = 172 + 132 = 458
Sisi yang lain = 172 – 132 = 120

Maka sisi yang ketiga adalah √4582 − 1202 = 442

A CATATAN :
Jika x dan y bilangan real positif, 𝑥 ≠ 𝑦 ,
maka :
AC = c = 𝑥 2 + 𝑦 2
458
442 AB = b = 2xy
BC= a = |𝑥 2 − 𝑦 2 |
Berlaku pada teorema pita goras untuk
B C segitiga siku-siku sembarang.
120

Problem 12
Kedua akar persamaan kuadrat 𝑥 2 − 111𝑥 + 𝑘 2 − 7 = 0 adalah bilangan prima.
Nilai k positif adalah ....

Pembahasan :
Jumlah akar-akar persamaan kuadrat 𝑥 2 − 111𝑥 + 𝑘 2 − 7 = 0
adalah 𝑥1 + 𝑥2 = 111
111 adalah bilangan ganjil. Bilangan ganjil diperoleh dari penjumlahan dua
bilangan bulat yang salah satunya bilangan genap.

Satu-satunya bilangan prima genap adalah 2. Maka salah satu akarnya adalah 2.
Karena jumlah kedua akarnya 111, maka akar yang lain adalah 109.

Maka 𝑥1 = 2 dan 𝑥2 = 109 , jadi 𝑘 2 − 7 = 𝑥1 . 𝑥2 = 2 . 109 = 218


𝑘 2 = 225 , maka nilai k positif adalah 15.

41 | A L J A B A R
Olimpiade Matematika SMA 2020

Problem 13
Tentukan himpunan penyelesaian pertaksamaan
5 1
a). Tentukan himpunan penyelesaian pertaksamaan 
2x − 1 x − 2
b). Jika 𝑎 dan 𝑏 bilangan asli, tentukan daerah penyelesaian a x  x − 1 + b
Pembahasan
2 2
5 1  5   1 
a).     
2x − 1 x − 2  2x − 1   x −2
 25( x − 2) − (2 x − 1)  0
25 1 2
 
2

(2 x − 1) (x − 2)
2 2

 21x 2 − 54 x + 24  0
 3(7 x − 4)(x − 2)  0
4   4 
Himpunan penyelesaiannya adalah  ,2  =  x  R;  x  2
7   7 
b). a x  x −1 + b  a x − x −1  b
Tuliskan persamaannya tanpa bentuk nilai mutlak dengan menggunakan
sifat :
 x, bila x  0  x − 1, bila x  1
x = dan x −1 = 
− x, bila x  0 1 − x, bila x  1
Titik 0 dan 1 membagi garis bilangan atas tiga daerah penyelesaian
Yaitu 𝑥 < 0 , 0 ≤ 𝑥 < 1 dan 𝑥 ≥ 1

Problem 14
Solusi (𝑥 , 𝑦 , 𝑧) ≠ 0 yang memenuhi 𝑥 2 + 𝑦 2 + 𝑧 2 = 𝑥𝑦 + 𝑥𝑧 + 𝑦𝑧 = 𝑥𝑦𝑧 adalah ...
Pembahasan :
𝑥 2 + 𝑦 2 + 𝑧 2 − 𝑥𝑦 − 𝑥𝑧 − 𝑦𝑧 = 0
2 (𝑥 2 + 𝑦 2 + 𝑧 2 − 𝑥𝑦 − 𝑥𝑧 − 𝑦𝑧) = 0
𝑥 2 − 2𝑥𝑦 + 𝑦 2 + 𝑦 2 − 2𝑦𝑧 + 𝑧 2 + 𝑥 2 − 2𝑥𝑧 + 𝑧 2 = 0
(𝑥 − 𝑦)2 + (𝑦 − 𝑧)2 + (𝑥 − 𝑧)2 = 0
Hanya ada satu kemungkinan untuk nilai 𝑥 − 𝑦 , 𝑦 − 𝑧 dan 𝑥 − 𝑧 yaitu 0.
Maka 𝑥 = 𝑦 = 𝑧
𝑥 2 + 𝑦 2 + 𝑧 2 = 𝑥𝑦𝑧
3𝑥 2 = 𝑥 3 berlaku untuk 𝑥 = 3 dan 𝑥 ≠ 0
Maka solusi (𝑥 , 𝑦 , 𝑧) adalah (3 , 3 , 3)

42 | A L J A B A R
Olimpiade Matematika SMA 2020

Problem 15
Misalkan 𝑎, 𝑏 dan 𝑐 adalah tiga bilangan berbeda. Jika ketiga bilangan
tersebut merupakan bilangan asli satu digit maka jumlah terbesar akar-akar
persamaan
(𝑥 − 𝑎)(𝑥 − 𝑏) + (𝑥 − 𝑏)(𝑥 − 𝑐) = 0
yang mungkin adalah ...

Pembahasan :

(𝑥 − 𝑎)(𝑥 − 𝑏) + (𝑥 − 𝑏)(𝑥 − 𝑐) = 0
(𝑥 − 𝑏)((𝑥 − 𝑎) + (𝑥 − 𝑐)) = 0
(𝑥 − 𝑏)(2𝑥 − 𝑎 − 𝑐) = 0
1
Akar-akar persamaan kuadrat tersebut, 𝑥1 = 𝑏 dan 𝑥2 = 2 (𝑎 + 𝑐)
Jumlah akar-akar persamaan 𝑥1 + 𝑥2 akan terjadi untuk 0 ≤ (𝑎, 𝑏, 𝑐) ≤ 9
1
𝑥1 + 𝑥2 terbesar = 𝑏 + 2 (𝑎 + 𝑐) terjadi jika b = 9, a = 8 dan c = 7
1 1
𝑥1 + 𝑥2 terbesar = 9 + 2 (8 + 7) = 16 2

Problem 16
Diketahui x dan y bilangan prima dengan x < y, dan

𝑥 3 + 𝑦 3 + 2020 = 30𝑦 2 − 300𝑦 + 3020

Nilai x yang memenuhi adalah ...

Pembahasan :
Diketahui x dan y bilangan prima dengan x < y

𝑥 3 + 𝑦 3 + 2020 = 30𝑦 2 − 300𝑦 + 3020


𝑥 3 + 𝑦 3 − 30𝑦 2 + 300𝑦 − 1000 = 0
𝑥 3 + (𝑦 − 10)3 = 0

berdasarkan pola 𝑎3 + 𝑏 3 = (𝑎 + 𝑏)(𝑎2 − 𝑎𝑏 + 𝑏 2 ) diperoleh persamaan

(𝑥 + 𝑦 − 10)(𝑥 2 − 𝑥(𝑦 − 10) + (𝑦 − 10)2 ) = 0

Diperoleh 𝑥 + 𝑦 − 10 = 0 atau 𝑥 + 𝑦 = 10
Karena x dan y bilangan prima dengan x < y , maka x = 3

43 | A L J A B A R
Olimpiade Matematika SMA 2020

Problem 17
Misalkan 𝑎, 𝑏 dan 𝑐 adalah bilangan bulat positif sehingga

𝑏 1
𝑐=𝑎+ −
𝑎 𝑏
Buktikan bahwa c adalah kuadrat dari suatu bilangan bulat.
Pembahasan :
𝑏 1
Pada pengubahan bentuk persamaan 𝑐 = 𝑎 + 𝑎 − 𝑏 diperoleh :

𝑏2 − 𝑎
𝑐=𝑎+
𝑎𝑏

𝑏2 −𝑎
Karena 𝑎 dan 𝑐 bilangan bulat, maka pastilah bilangan bulat juga.
𝑎𝑏

2
Olehkarena itu jika 𝑏|𝑏 − 𝑎 maka 𝑏|𝑎.
Misalkan 𝑎 = 𝑘 𝑏 dengan k bilangan bulat positif, maka :

𝑏2 − 𝑎 𝑏2 − 𝑘 𝑏 𝑏−𝑘
𝑐=𝑎+ =𝑎+ 2
=𝑎+
𝑎𝑏 𝑘𝑏 𝑘𝑏

𝑏 dan 𝑘 adalah bilangan bulat positif dimana 𝑏|𝑘 dan 𝑘|𝑏 , maka 𝑏 = 𝑘.

𝑏2 −𝑎
Akibatnya 𝑐 = 𝑎 dan = 0. Maka tentulah 𝑎 = 𝑏 2 atau 𝑐 = 𝑏 2
𝑎𝑏

Jadi 𝑐 = 𝑏 2 terbukti bahwa 𝑐 adalah kuadrat suatu bilangan.

Problem 18
Jika a dan b adalah bilangan real, dalam bentuk persamaan :
𝑎3 − 3𝑎𝑏 2 = 44
𝑏 3 − 3𝑎2 𝑏 = 8
Maka nilai dari 𝑎2 + 𝑏 2 adalah ....
Pembahasan :
Kedua persamaan ini tidak dapat dieliminasi langsung karena sebenarnya
memiliki empat variabel yang berbeda.
𝑎3 − 3𝑎𝑏 2 = 44
𝑏 3 − 3𝑎2 𝑏 = 8
𝑎3 − 3𝑎𝑏 2 = 44 dikuadratkan 𝑎6 − 6𝑎4 𝑏 2 + 9𝑎2 𝑏 4 = 1936

𝑏 3 − 3𝑎2 𝑏 = 8 dikuadratkan 𝑏 6 − 6𝑎2 𝑏 4 + 9𝑎4 𝑏 2 = 64

44 | A L J A B A R
Olimpiade Matematika SMA 2020

Penjumlahan
(𝑎6 − 6𝑎4 𝑏 2 + 9𝑎2 𝑏 4 ) + (𝑏 6 − 6𝑎2 𝑏 4 + 9𝑎4 𝑏 2 ) = 1936 + 64
𝑎6 + 3𝑎4 𝑏 2 + 3𝑎2 𝑏 4 + 𝑏 6 = 2000
(𝑎2 + 𝑏 2 )3 = 2000
3
𝑎2 + 𝑏 2 = 10 √2

Problem 19
Untuk x , y bilangan real tak nol, jumlah nilai maksimum dan nilai minimum

𝑥𝑦 − 4𝑦 2
𝑥 2 + 4𝑦 2
adalah .....
Pembahasan :

𝑥𝑦−4𝑦2
Misalkan = 𝑠 dan 𝑥 = 𝑘𝑦 maka diperoleh :
𝑥 2 +4𝑦 2
𝑘𝑦 2 − 4𝑦 2
=𝑠
(𝑘𝑦)2 + 4𝑦 2

𝑘−4
=𝑠
𝑘2 + 4

𝑠𝑘 2 + 4𝑠 = 𝑘 − 4

𝑠𝑘 2 − 𝑘 + 4𝑠 + 4 = 0

Karena k bilangan real, maka diskriminannya 𝐷 ≥ 0

(−1)2 − 4𝑠(4𝑠 + 4) ≥ 0
1 − 16𝑠 2 − 16𝑠 ≥ 0
−16𝑠 2 − 16𝑠 + 1 ≥ 0
𝑥𝑦−4𝑦 2
Nilai minimum dan maksimum adalah 𝑠1 dan 𝑠2 dimana s adalah
𝑥 2 +4𝑦 2
Kareana −16𝑠 2 − 16𝑠 + 1 = 0 maka 𝑠1 + 𝑠2 = − 1

45 | A L J A B A R
Olimpiade Matematika SMA 2020

Problem 20
90
Misalkan 𝑓(𝑥) = 1 + , nilai terbesar x yang memenuhi :
𝑥
𝑓(𝑓(… . (𝑓(𝑥)) … . )) = 𝑥

2019 𝑘𝑎𝑙𝑖
adalah ....

Pembahasan :
Bentuk 𝑓(𝑓(… . (𝑓(𝑥)) … . )) pada soal ini kita kenal sebagai fungsi turun yang
berkelanjutan, Berikut ini adalah persamaan 𝑓(𝑥1 ) = 𝑓(𝑥) , 𝑓(𝑥2 ) dan 𝑓(𝑥3 ).
Jika 𝑓(𝑥2 ) = 𝑓(𝑓(𝑥)) dan 𝑓(𝑥3 ) = 𝑓(𝑓(𝑥2 )) dan seterusnya.

90
𝑓(𝑥1 ) = 𝑓(𝑥) = 1 +
𝑥
90
𝑓(𝑥2 ) = 𝑓(𝑓(𝑥)) = 1 +
90
1+ 𝑥

dan seterusnya sampai 2019 kali.

2019 memiliki faktor 1, 3, 673 dan 2019, maka apabila 𝑓(𝑥𝑛 ) merupakan
komposisi n kali dari 𝑓(𝑥) maka sifat fungsi ini konstan untuk setiap faktor dari
2019. Sehingga :
𝑓(𝑥1 ) = 𝑓(𝑥3 ) = 𝑓(𝑥673 ) = 𝑓(𝑥2019 ) = 𝑥

Dimana 𝑓(𝑥1 ) = 𝑓(𝑥) = 𝑥


90
𝑓(𝑥) = 1 + =𝑥
𝑥
90
1+ =𝑥
𝑥
Kedua ruas di kali dengan 𝑥 sehingga terbentuk persamaan kuadrat

𝑥 2 − 𝑥 − 90 = 0
(𝑥 − 10)(𝑥 + 9) = 0

𝑥1 = 10 dan 𝑥2 = −9
90
Jika kita ingin membuktikan 𝑓(𝑥3 ) 𝑚𝑎𝑘𝑎 𝑓 (𝑓(𝑓(𝑥))) = 1 + 90 =𝑥
1+ 90
1+
𝑥
Maka kita akan mendapatkan solusi yang sama, 𝑥 = 10

46 | A L J A B A R
Olimpiade Matematika SMA 2020

Problem 21
𝑎+𝑏 𝑏+𝑐 𝑎+𝑐 1 1 1
Jika 𝑎, 𝑏, 𝑐  𝑅 + , buktikan + + ≥2 ( + + )
𝑐2 𝑎2 𝑏2 𝑎 𝑏 𝑐
Pembahasan :

(𝑎+𝑏)(𝑏+𝑐)(𝑎+𝑐)
Menurut 𝐴𝑀 ≥ 𝐺𝑀 telah kita ketahui, ≥8
𝑎𝑏𝑐

3 3 27
Menurut 𝐺𝑀 ≥ 𝐻𝑀 telah kita ketahui, √𝑎𝑏𝑐 ≥ 1 1 1 atau 𝑎𝑏𝑐 ≥ 1 1 1 3
+ + ( + + )
𝑎 𝑏 𝑐 𝑎 𝑏 𝑐

𝑎+𝑏 𝑏+𝑐 𝑎+𝑐 3 𝑎 +𝑏 𝑏 +𝑐 𝑎 +𝑐


+ + ≥ 3 √ 2 . 2 . 2
𝑐2 𝑎2 𝑏2 𝑐 𝑎 𝑏

𝑎+𝑏 𝑏+𝑐 𝑎+𝑐 3 (𝑎 + 𝑏)(𝑏 + 𝑐)(𝑎 + 𝑐) 1


+ + ≥ 3 √
𝑐 2 𝑎 2 𝑏 2 𝑎𝑏𝑐 𝑎𝑏𝑐

𝑎+𝑏 𝑏+𝑐 𝑎+𝑐 3 1


+ + ≥ 3 √ 8 .
𝑐2 𝑎2 𝑏2 𝑎𝑏𝑐

1 1 1 3
3
𝑎+𝑏 𝑏+𝑐 𝑎+𝑐 √ (𝑎 + 𝑏 + 𝑐 )
+ + ≥ 3 8.
𝑐2 𝑎2 𝑏2 27

𝑎+𝑏 𝑏+𝑐 𝑎+𝑐 1 1 1


2
+ 2 + 2 ≥2 ( + + )
𝑐 𝑎 𝑏 𝑎 𝑏 𝑐

Problem 22
Jika ABC suatu segitiga sembarang, nilai minimum tan 𝐴 . tan 𝐵 . tan 𝐶 adalah ....
Pembahasan :
Pada segitiga ABC berlaku jumlah sudut A + B + C = 180o
A + B = 180o – C
tan(𝐴 + 𝐵) = tan (180𝑜 − 𝐶)
tan 𝐴 + tan 𝐵
= − tan 𝐶
1 − tan 𝐴 . tan 𝐵

tan 𝐴 + tan 𝐵 = − tan 𝐶 + tan 𝐴 . tan 𝐵 . tan 𝐶


tan 𝐴 + tan 𝐵 + tan 𝐶 = tan 𝐴 . tan 𝐵 . tan 𝐶

47 | A L J A B A R
Olimpiade Matematika SMA 2020

Berdasarkan 𝐴𝑀 ≥ 𝐺𝑀

tan 𝐴 + tan 𝐵 + tan 𝐶 3


≥ √tan 𝐴 . tan 𝐵 . tan 𝐶
3
3
tan 𝐴 + tan 𝐵 + tan 𝐶 ≥ 3 √tan 𝐴 . tan 𝐵 . tan 𝐶
3
tan 𝐴. tan 𝐵. tan 𝐶 ≥ 3 √tan 𝐴 . tan 𝐵 . tan 𝐶

(tan 𝐴. tan 𝐵. tan 𝐶)3 ≥ 27 (tan 𝐴. tan 𝐵. tan 𝐶)

Karena tan A  0, Karena tan B  0 dan Karena tan C  0, maka :

(tan 𝐴. tan 𝐵. tan 𝐶)2 ≥ 27


tan 𝐴. tan 𝐵. tan 𝐶 ≥ 3√2

Jadi nilai minimum tan 𝐴. tan 𝐵. tan 𝐶 adalah 3√2

Problem 23
Buktikan bahwa hasil kali empat bilangan berurutan yang membentuk deret
aritmatika ditambah pangkat empat dari beda suku-sukunya, merupakan
bilangan kuadrat.

Pembahasan :
Misalkan a, a+b, a + 2b dan a + 3b adalah empat suku deret aritmatika yang
berurutan dengan beda suku-sukunya b.
Hasil kali keempat bilangan ditambah pangkat empat beda suku-sukunya
merupakan bilangan kuadrat, maka dapat dituliskan menjadi :

𝑎 (𝑎 + 𝑏)(𝑎 + 2𝑏)(𝑎 + 3𝑏) + 𝑏 4 = 𝑁 2

𝑎(𝑎 + 3𝑏) (𝑎 + 𝑏)(𝑎 + 2𝑏) + 𝑏 4 = 𝑁 2


(𝑎3 + 3𝑎𝑏)(𝑎2 + 3𝑎𝑏 + 2𝑏 2 ) + 𝑏 4 = 𝑁 2

𝑎4 + 6𝑎3 𝑏 + 11𝑎2 𝑏 2 + 6𝑎𝑏 3 + 𝑏 4 = 𝑁 2


(𝑎2 + 3𝑎𝑏 + 𝑏 2 )2 = 𝑁 2 (terbukti)

48 | A L J A B A R
Olimpiade Matematika SMA 2020

Problem 24
Buktikan bahwa jumlah kuadrat dari n suku pertatama bilangan asli adalah
𝑛 (2𝑛 + 1)(𝑛 + 1)
6
Pembahasan :
jumlah kuadrat dari n suku pertatama bilangan asli 12 + 22 + 32 + ⋯ … . . + 𝑛2
membentuk pola bilangan seperti di bawah ini :

1 , 4 , 9 , 16 , 25 , … ..

3 5 7 9 ................

2 2 2 ..............

Bilangan pertama setiap pola deret ke bawah adalah 1, 3 dan 2 digunakan untuk
menentukan rumus jumlah n suku pertama.

1 𝑛 3𝑛 (𝑛 − 1) 2𝑛 (𝑛 − 1)(𝑛 − 2)
𝑆𝑛 = + +
1! 2! 3!
3 1
𝑆𝑛 = 𝑛 + 𝑛(𝑛 − 1) + 𝑛(𝑛 − 1)(𝑛 − 2)
2 6

6𝑛 + 𝑛(𝑛 − 1)(2𝑛 + 5) 𝑛(2𝑛2 + 3𝑛 + 1)


𝑆𝑛 = =
6 6

𝑛 (2𝑛 + 1)(𝑛 + 1)
𝑆𝑛 =
6

Problem 25
Misalkan a > 0 dan 0 < r1 < r2 < 1
sehingga 𝑎 + 𝑎𝑟1 + 𝑎𝑟12 + ⋯ dan 𝑎 + 𝑎𝑟2 + 𝑎𝑟22 + ⋯. adalah deret geometri
tak hingga dengan jumlah berturut-turut 𝑟1 dan 𝑟2 . Nilai dari 𝑟1 + 𝑟2
adalah....

Pembahasan :
𝑎 𝑎
Berdasarkan rumus deret geometri tak hingga, 𝑟1 = dan 𝑟2 =
1−𝑟1 1−𝑟2
Atau dalam bentuk 𝑟12 − 𝑟1 + 1 = 0 dan 𝑟22 − 𝑟2 + 1 = 0.
maka 𝑟1 dan 𝑟2 adalah akar-akar dari persamaan kuadrat 𝑟 2 − 𝑟 + 1 = 0
sehingga 𝑟1 + 𝑟2 = 1

49 | A L J A B A R
Olimpiade Matematika SMA 2020

Problem 26
1 1 1
Diberikan , , adalah barisan aritmatika. Tunjukkan bahwa
𝑎+𝑏 𝑏+𝑐 𝑐+𝑎
𝑏 2 , 𝑎2 , 𝑐 2 juga barisan aritmatika.

Pembahasan :
Pada barisan aritmatika mempunyai beda yang sama maka berlaku :

1 1 1 1
− = −
𝑏+𝑐 𝑎+𝑏 𝑐+𝑎 𝑏+𝑐

𝑎+𝑏−𝑏−𝑐 𝑏+𝑐−𝑐−𝑎
=
(𝑏 + 𝑐)(𝑎 + 𝑏) (𝑐 + 𝑎) (𝑏 + 𝑐)

𝑎−𝑐 𝑏−𝑎
=
(𝑎 + 𝑏) (𝑐 + 𝑎)

(𝑎 − 𝑐) (𝑎 + 𝑐) = (𝑏 − 𝑎)(𝑏 + 𝑎)

𝑎2 − 𝑐 2 = 𝑏 2 − 𝑎2

Pada barisan 𝑏 2 , 𝑎2 , 𝑐 2 memiliki beda suku 𝑎2 − 𝑐 2 dan 𝑏 2 − 𝑎2 yang sama


Maka 𝑏 2 , 𝑎2 , 𝑐 2 merupakan barisan aritmatika

Problem 27
Jika 𝑎, 𝑏 , 𝑐 adalah solusi dari persamaan 𝑥 3 − 64𝑥 − 14 = 0. Nilai 𝑎3 + 𝑏 3 + 𝑐 3
adalah ...

Pembahasan :
𝑥 3 − 64𝑥 − 14 = 0
Menurut teorema Vieta :
0 𝑏
𝑎 + 𝑏 + 𝑐 = − 1 = 0 , mengingatkan kembali 𝑥1 + 𝑥2 + 𝑥3 = −
𝑎

𝑥 3 = 64𝑥 + 14

𝑎3 = 64𝑎 + 14 , 𝑏 3 = 64𝑏 + 14 , 𝑐 3 = 64𝑐 + 14


𝑎3 + 𝑏3 + 𝑐3 = 64𝑎 + 14 + 64𝑏 + 14 + 64𝑐 + 14
𝑎3 + 𝑏 3 + 𝑐 3 = 64 (𝑎 + 𝑏 + 𝑐) + 42
𝑎3 + 𝑏 3 + 𝑐 3 = 64 (0) + 42 = 42

50 | A L J A B A R
Olimpiade Matematika SMA 2020

Problem 28
Semua bilangan bulan n sehingga 𝑛4 + 16𝑛3 + 71𝑛2 + 56𝑛 merupakan bilangan
kuadrat tak nol adalah .....

Pembahasan :
Misalkan 𝑛4 + 16𝑛3 + 71𝑛2 + 56𝑛 = 𝐴 , merupakan bilangan kuadrat tak nol.

𝐴 = 𝑛4 + 16𝑛3 + 71𝑛2 + 56𝑛𝑐

𝐴 = 𝑛 (𝑛3 + 16𝑛2 + 71𝑛 + 56)

𝐴 = 𝑛 (𝑛 + 1)(𝑛2 + 15𝑛 + 56)

𝐴 = (𝑛 + 8) 𝑛 (𝑛 + 1)(𝑛 + 7)

𝐴 = (𝑛2 + 8𝑛) (𝑛2 + 8𝑛 + 7) , misalkan 𝑛2 + 8𝑛 = 𝑦, maka :

𝐴 = 𝑦 (𝑦 + 7) = 𝑦 2 + 7𝑦

𝑦 2 + 7𝑦 harus bilangan kuadrat, maka 𝑦 2 + 7𝑦 = 𝑚2 atau 𝑦 2 + 7𝑦 − 𝑚2 = 0

Diskriminannya haruslah bilangan kuadrat, sehingga 𝐷 = 72 − 4 . 1 (−𝑚2 ) = 𝑝2

49 + 4 𝑚2 = 𝑝2
𝑝2 − 4 𝑚2 = 49
(𝑝 − 2𝑚)(𝑝 + 2𝑚) = 49

Maka nilai-nilai yang mungkin ( 1 , 49) :


𝑝 − 2𝑚 = 1 dan 𝑝 + 2𝑚 = 49
Dengan substitusi, kita mendapatkan nilai 𝑚 = 12 dan 𝑝 = 25

Untuk m = 12, persamaan 𝑦 2 + 7𝑦 = 𝑚2 menjadi 𝑦 2 + 7𝑦 − 144 = 0


𝑦 2 + 7𝑦 − 144 = 0
(𝑦 + 16)(𝑦 − 9) = 0
𝑦 + 16 = 0 dan 𝑦 − 9 = 0
𝑦 + 16 = 0 , dimana 𝑦 = 𝑛2 + 8𝑛 ,
maka 𝑛2 + 8𝑛 + 16 = 0 atau menjadi (𝑛 + 4)2 = 0 , 𝑛 = −4
𝑦 − 9 = 0 , dimana 𝑦 = 𝑛2 + 8𝑛 ,
maka 𝑛2 + 8𝑛 − 9 = 0 atau menjadi (𝑛 + 9) (𝑛 − 1) = 0, 𝑛 = −9 dan 𝑛 = 1
Jadi nilai 𝑛 yang memenuhi agar 𝑛4 + 16𝑛3 + 71𝑛2 + 56𝑛 merupakan bilangan
kuadrat tak nol ada tiga, yaitu {−9, −4, 1}

51 | A L J A B A R
Olimpiade Matematika SMA 2020

Problem 29
Jika 𝑥1 , 𝑥2 , 𝑥3 , 𝑥4 , … . . 𝑥2020 merupakan akar-akar dari 𝑥 2020 − 2020𝑥 + 1 = 0,
Nilai dari
2020

∑ 𝑥𝑖 2020
𝑖=1
adalah .....
Pembahasan :

𝑥 2020 − 2020𝑥 + 1 = 0

0
𝑥1 + 𝑥2 + 𝑥3 + ⋯ 𝑥2020 = − =0
1

𝑥 2020 = 2020𝑥 − 1

𝑥12020 = 2020𝑥1 − 1

𝑥22020 = 2020𝑥2 − 1

𝑥32020 = 2020𝑥3 − 1
...................................
..................................
..................................
..................................
..................................

2020
𝑥2020 = 2020𝑥2020 − 1

Menurut teorema Vieta :

2020

∑ 𝑥𝑖 2020 = 2020(𝑥1 + 𝑥2 + 𝑥3 + ⋯ 𝑥2020 ) − 2020 (1)


𝑖=1

2020

∑ 𝑥𝑖 2020 = 2020(0) − 2020


𝑖=1

2020

∑ 𝑥𝑖 2020 = − 2020
𝑖=1

52 | A L J A B A R
Olimpiade Matematika SMA 2020

Problem 30
Misalkan 𝑓(𝑥) = 𝑥 + 3𝑥 2 + 5𝑥 3 + 7𝑥 4 + ….
Dipenuhi oleh 𝑥1 dan 𝑥2 sehingga 𝑓(𝑥1 ) = 𝑓(𝑥2 ) = 10, maka nilai 𝑓(⌊𝑥1 + 𝑥2 ⌋)
adalah .....

Pembahasan :

𝑓(𝑥) = 𝑥 + 3𝑥 2 + 5𝑥 3 + 7𝑥 4 + …. (1)
𝑥𝑓(𝑥) = 𝑥 2 + 3𝑥 3 + 5𝑥 4 + 7𝑥 5 + …. (2)

(2) dikurangi (1) akan menghasilkan :


𝑓(𝑥) − 𝑥𝑓(𝑥) = 𝑥 + 2𝑥 2 + 2𝑥 3 + 2𝑥 4 + ⋯
2𝑥 2
(1 − 𝑥)𝑓(𝑥) = 𝑥 +
1−𝑥
2
𝑥 +𝑥
𝑓(𝑥) =
(1 − 𝑥)2
𝑓(𝑥) = 10
𝑥2 + 𝑥
= 10
(1 − 𝑥)2

𝑥 2 + 𝑥 = 10 (1 − 2𝑥 + 𝑥 2 )
𝑥 2 + 𝑥 = 10 − 20𝑥 + 10𝑥 2
9𝑥 2 − 21𝑥 + 10 = 0
(3𝑥 − 2)(3𝑥 − 5) = 0
2 5
𝑥1 = 𝑑𝑎𝑛 𝑥2 =
3 3
2 5 7
⌊𝑥1 + 𝑥2 ⌋ = ⌊ + ⌋ = ⌊ ⌋ = 2
3 3 3
22 + 2
𝑓(2) = =6
(1 − 2)2

53 | A L J A B A R
Olimpiade Matematika SMA 2020

Soal-soal Latihan Isian Singkat

1
Angka satuan yang dihasilkan pada operasi hitung berikut ini
6887344 . 39921037 + 32 . 42757 + 411233 . 109687 adalah ....
Kunci Jawab : 3

2
Bilangan real 2,13636363636 .... adalah bilangan rasional, sehingga dapat ditulis
𝑛
dalam bentuk , dimana n, m bilangan-bilangan bulat, m ≠ 0. Nilai terkecil dari
𝑚
m + n =....
Kunci Jawab : 3105

3
Perhatikan persegi ajaib berikut :

A B C

D E F

G H I

Huruf-huruf dalam persegi ajaib diisi dengan bilangan bulat satu digid sehingga
jumlah setiap 3 huruf yang tersusun secara vertikal, horisontal dan diagonal
selalu sama. Jika C = 6, H = 7 dan I = 2.
maka A + B + C + D + E + F + G + H + I = .....
Kunci Jawab : 26

4
In a group of dogs and people, the number of legs was 28 more than twice the
number of heads. How many dogs were there?
Kunci Jawab : 114

54 | A L J A B A R
Olimpiade Matematika SMA 2020

5
Hasil dari 20203 − 2019 . 2020 . 2021 = .....
Kunci Jawab : 2020

Jika diketahui suatu data :


4–5+6=1
5–6+7=2
6–7+8=3
7–8+4=4
8–4+5=5
Maka, (6 + 7 + 8) – ( 4 x 5) = ......
Kunci Jawab : 9

7
Hasil dari 153 + 163 + 173 +1 83 +1 93 + 203 adalah ...
Kunci Jawab : 33075

8
√7+√3 √7−√3
Diketahui 𝑥 = dan 𝑦 =
√7−√3 √7+√3

Maka nilai dari 𝑥 5 + 𝑦 5 adalah ....


Kunci Jawab : 2525

9
Peusahaan jasa Penjahit memiliki beberapa tenaga pria dan wanita yang terampil
membuat gaun pesta. Tenaga pria mampu membuat sebuah gaun rata-rata
18 jam tanpa istirahat, sedangkan tenaga wanita 36 jam. Suatu ketika pimpinan
perusahaan memilih 5 orang yang terdiri dari pria dan wanita, untuk diberi
tugas membuat 50 gaun yang sama.
Dengan mengabaikan waktu istirahat dan waktu tidak bekerja, ke 5 orang itu
dapat menyelesaikan pekerjaan tersebut paling lama selama ....... jam

Kunci Jawab : 300

55 | A L J A B A R
Olimpiade Matematika SMA 2020

10
Jika 𝑎2 − 3𝑎 + 1 = 0, dan 𝑤, 𝑥, 𝑦 dan 𝑧 dinyatakan dalam bentuk di bawah ini :
1 1 1 1
𝑤 =𝑎+𝑎 𝑥 = 𝑎2 + y= 𝑎3 + 𝑧 = 𝑎4 +
𝑎2 𝑎3 𝑎4
𝑥+𝑦+𝑧
maka nilai = ....
𝑤2
Kunci Jawab : 8

11
Seorang petualang pergi dari rumah melalui sebuah jalan rata, kemudian menaik
sampai puncak sebuah bukit. Sesampai di puncak bukit ia langsung turun
kembali sampai di rumah dengan jalan yang sama. Petualang berangkat jam 6
pagi dan kembali sampai rumah jam 12 siang. Jika jarak jalan mendatar sama
dengan jarak jalan menaik, kecepatannya ialah 4 km/jam di jalan rata, 3 km/jam
di jalan menaik dan 6 km/jam di jalan menurun. Jarak yang ditempuh pergi
pulang oleh petualang tersebut adalah .... km
Kunci Jawab : 24

12
3 3 3
Solusi dari persamaan √1 + √𝑥 + √1 − √𝑥 = √5 adalah .....
𝟓
Kunci Jawab : 𝟒

13
Jika 𝑓 adalah fungsi terdefinisi pada himpunan bilangan real dan berlaku :
3𝑓(𝑥) − 2 𝑓(2 − 𝑥) = 𝑥 2 + 8𝑥 − 9
Nilai dari 𝑓(10) adalah ...
Kunci Jawab : 99

14
Jika 𝑎, 𝑏, 𝑐 bilangan real, 𝑎 + 𝑏 + 𝑐 = 0, dan (𝑎 − 1)(𝑏 − 2)(𝑐 − 3) = 30, maka nilai
dari (𝑎 − 1)3 + (𝑏 − 2)3 + (𝑐 − 3)3 adalah ....
Kunci Jawab : 90

56 | A L J A B A R
Olimpiade Matematika SMA 2020

15
Persamaan kuadrat 𝑥 2 + 𝑎𝑥 + 𝑏 = 0 memiliki akar-akar bilangan bulat positif 𝑥1 <
𝑥2 , bukan prima. Jika 𝑎 + 𝑏 = 14, maka 𝑏𝑥1 − (𝑥2 + 𝑎)2 = .....
Kunci Jawab : 80

16

Misalkan 𝑓(𝑛) = 3𝑛𝑥 + 3−𝑛𝑥


Jika 𝑓(2) − 9𝑓(1) + 20 = 0 , maka nilai 𝑓(3) = ....
Kunci Jawab : 198

17
Jika (x , y) merupakan pasangan solusi pada sistem persamaan :

|𝑥 − 𝑦| = 𝑥 + 𝑦 − 2
|𝑥 + 𝑦| = 𝑥 + 2

Banyaknya pasangan (x , y) yang memenihi adalah ...


Kunci Jawab : 1

18
Jika 𝑚 = |𝑥 + 2| + |𝑥 − 1| − |2𝑥 − 4|, maka nilai maksimum m adalah ....
Kunci Jawab : 5

19
Jika a, b dan c bilangan positif dan abc = 1, maka nilai x pada persamaan :
2𝑎𝑥 2𝑏𝑥 2𝑐𝑥
+ + =1
𝑎𝑏 + 𝑎 + 1 𝑏𝑐 + 𝑏 + 1 𝑎𝑐 + 𝑐 + 1
adalah .....
𝟏
Kunci Jawab : 𝟐

57 | A L J A B A R
Olimpiade Matematika SMA 2020
20

Jika 𝑎, 𝑏, 𝑐 adalah solusi dari 𝑥 3 − 𝑥 − 1 = 0 , maka 𝑎5 + 𝑏 5 + 𝑐 5 = .....


Kunci Jawab : 5

21

Banyaknya akar real yang berbeda dari 𝑥7 + 𝑥6 + 𝑥5 + 𝑥4 + 𝑥3 + 𝑥2 + 𝑥 + 1


adalah .....
Kunci Jawab : 1

22
Hasil dari :
5 5 2 5 4 5 8 5 16
( + 1) (( ) + 1) (( ) + 1) (( ) + 1) (( ) + 1) ….
6 6 6 6 6
adalah ....
Kunci Jawab : 6

23

Jika 𝑓(𝑥) = 𝑥 2 − 2𝑥 , 𝑥 > 0 maka nilai x agar 𝑓(1 + 𝑓(𝑥)) = 2020 .2018 − 20192
terpenuhi adalah ...
Kunci Jawab : 2

24
Jika 𝑓(𝑥) = (𝑥 − 1) + (𝑥 − 2)2 + (𝑥 − 3)3 + (𝑥 − 4)4 + (𝑥 − 5)5 maka jumlah dari
semua akar dari 𝑓(𝑥) adalah ...
Kunci Jawab : 24

25

Misalkan 𝑓(𝑥 + 3) + 𝑓(𝑥) = 2𝑥 + 5


Jika 𝑓(2) + 𝑓(8) = 12, maka 2𝑓(5) + 𝑓(8) = .....
Kunci Jawab : 21

58 | A L J A B A R
Olimpiade Matematika SMA 2020

26
𝑥
Misalkan 𝑓(𝑥) = dan 𝑓𝑛 (𝑥) = ⏟
𝑓(𝑓(𝑓(𝑓 … . 𝑓(𝑥) ) … . ))) , dapat diartikan
√1+𝑥 2
𝑛

bahwa untuk 𝑓1 (𝑥) = 𝑓(𝑥) dan 𝑓𝑛+1 (𝑥) = 𝑓(𝑓𝑛 (𝑥)). Nilai (10 . 𝑓99 (1)) = ...

Kunci Jawab : 1

27
Jika diketahui fungsi f derdefinisi untuk :
𝑓(𝑥) + 𝑓(1 − 𝑥) = 2020
𝑓(1 + 𝑥) − 𝑓(𝑥) = 2019
𝑓(2020) + 𝑓(−2020) = ......

Kunci Jawab : 1

28
Diketahui n bilangan real 𝑥1 , 𝑥2 , 𝑥3 , … . . , 𝑥𝑛 dengan |𝑥𝑖 | < 1 , (i = 1, 2, 3, 4, .....n)
|𝑥1 | + |𝑥2 | + |𝑥3 | + ⋯ … . . + |𝑥𝑛 | = 49 + |𝑥1 + 𝑥2 + 𝑥3 + ⋯ . . +𝑥𝑛 |
Nilai minimum n adalah ....
Kunci Jawab : 50

29

Jika 𝑥 (𝑥 + 1)(𝑥 + 3 + (𝑥 + 4) = 180


Maka nilai dari 𝑥 (𝑥 − 1)(𝑥 − 3 + (𝑥 − 4) = ....
Kunci Jawab : 4

30

Jika 𝑓(𝑥) = √12 + 𝑥


Maka nilai dari 𝑓(𝑓(𝑓(…
⏟ … . 𝑓(𝑥) … ))adalah .....
𝑠𝑎𝑚𝑝𝑎𝑖 𝑡𝑎𝑘 ℎ𝑖𝑛𝑔𝑔𝑎
Kunci Jawab : 4

59 | A L J A B A R
Olimpiade Matematika SMA 2020
31

Jika diketahui 𝑎 + 𝑏 + 𝑐 = 6 , 𝑎2 + 𝑏 2 + 𝑐 2 = 26 , 𝑎3 + 𝑏 3 + 𝑐 3 = 90
Maka nilai dari 𝑎4 + 𝑏 4 + 𝑐 4 = .....
Kunci Jawab : 338

32

Jika 𝑎 + 𝑏 + 𝑐 = 1 dimana 𝑎, 𝑏, 𝑐 adalah bilangan real positif, maka nilai


𝑎2 𝑏2 𝑐2
minimum dari + + adalah ....
𝑏 𝑐 𝑎
𝟏
Kunci Jawab :
𝟐𝟕

33
Jika 𝑎, 𝑏, 𝑐, 𝑑 real, nilai minimum dari
1 1 4 16
(𝑎 + 𝑏 + 𝑐 + 𝑑) ( + + + )
𝑎 𝑏 𝑐 𝑑
adalah ....
Kunci Jawab : 64

34
Untuk setiap x positif, berlaku (2𝑥 + 1)(𝑥 + 2) ≥ 𝑥 𝑁
Nilai 𝑁 real terbesar adalah ....
Kunci Jawab : 9

35

Jika 𝑓(𝑥) = (𝑥 + 1)(𝑥 − 3)(𝑥 + 2) , sisa pembagian 𝑓(𝑥) oleh (𝑥 − 1) adalah ...
Kunci Jawab : −12

60 | A L J A B A R
Olimpiade Matematika SMA 2020

Soal-soal Latihan Uraian Singkat

1. Jika a + b + c = 0. Tunjukan bahwa 2 (𝑎4 + 𝑏 4 + 𝑐 4 ) 𝑎4 = (𝑎2 + 𝑏 2 + 𝑐 2 )2 !

2. Buktikan bahwa 𝑎3 + 𝑏 3 + 𝑐 3 − 3𝑎𝑏𝑐 habis dibagi oleh 𝑎 + 𝑏 + 𝑐 !

3. Untuk 𝑛 ∈ Asli dimana 𝑛4 + 2𝑛3 − 14𝑛2 + 2𝑛 − 8 jika dibagi 𝑛2 + 1 bersisa 7,


maka nilai n yang mungkin adalah ....

4. Diberikan fungsi 𝑓(𝑥) = 𝑥 2 + 4. Misalkan x dan y adalah bilangan-bilangan


real positif yang memenuhi :
𝑓(𝑥𝑦) + 𝑓(𝑦 − 𝑥) = 𝑓(𝑦 + 𝑥)
Maka nilai minimum x + y adalah ....

5. Jika 𝑎, 𝑏, 𝑐, 𝑑 adalah bilangan-bilangan bulat positif, 𝑎 + 𝑏 + 𝑐 + 𝑑 = 24, nilai


maksimum 𝑎𝑏 + 𝑏𝑐 + 𝑐𝑑 + 𝑎𝑑 adalah ...

2𝑥 2 𝑐𝑜𝑠2 𝑥 + 8 
6. Misalkan 𝑓(𝑥) = untuk 0 < 𝑥 < 2 , Nilai minimum 𝑓(𝑥) = ....
𝑥 cos 𝑥

2
7. Jumlah semua nilai 𝑥  𝐵 agar log(𝑥 2 − 6𝑥 − 3) juga merupakan bilangan
bulat adalah ...

8. Diketahui barisan bilangan real 𝑎1 , 𝑎2 , 𝑎3 , … . 𝑎𝑛 merupakan barisan geometri.


Jika 𝑎1 + 𝑎4 = 20 , nilai minimum dari jumlah 6 suku pertama deret ini
adalah ...

2021 2020
9. Buktikan 2020 ! ≤ ( )
2

10. Jika 𝑎, 𝑏, 𝑐, 𝑑  𝑅 +, buktikan bahwa :


1 1 1 𝑎 𝑏 𝑐 1 1 1 3 3
(𝑎 + 𝑏 + 𝑐 ) ( + + ) ( + + ) ( + + ) (𝑎 + 𝑏3 + 𝑐 3 ) ≥ 81
𝑎 𝑏 𝑐 𝑏 𝑐 𝑎 3𝑎 3𝑏 3𝑐

61 | A L J A B A R
Olimpiade Matematika SMA 2020

Soal dan Pembahasan

Soal & Pembahasan nomor 1 :


Jika a + b + c = 0. Tunjukan bahwa 2 (𝑎4 + 𝑏 4 + 𝑐 4 ) = (𝑎2 + 𝑏 2 + 𝑐 2 )2 !

Pembahasan :
2 (𝑎4 + 𝑏 4 + 𝑐 4 ) − (𝑎2 + 𝑏 2 + 𝑐 2 )2 = 0
2 (𝑎4 + 𝑏 4 + 𝑐 4 ) − (𝑎4 + 𝑏 4 + 𝑐 4 + 2𝑎2 𝑏 2 + 2𝑎2 𝑐 2 + 2𝑏 2 𝑐 2 ) = 0
𝑎4 + 𝑏 4 + 𝑐 4 − 2𝑎2 𝑏 2 − 2𝑎2 𝑐 2 − 2𝑏 2 𝑐 2 = 0
(𝑎2 − 𝑏 2 − 𝑐 2 )2 − 4𝑏 2 𝑐 2 = 0
(𝑎2 − 𝑏 2 − 𝑐 2 )2 − (2𝑏𝑐)2 = 0

(𝑎2 − (𝑏 − 𝑐)2 )(𝑎2 − (𝑏 + 𝑐)2 ) = 0


(𝑎 − 𝑏 + 𝑐)(𝑎 + 𝑏 − 𝑐)(𝑎 − 𝑏 − 𝑐)(𝑎 + 𝑏 + 𝑐) = 0

Terbukti, 𝑎 + 𝑏 + 𝑐 = 0, pembuat nol.

Soal & Pembahasan nomor 2 :


Buktikan bahwa 𝑎3 + 𝑏 3 + 𝑐 3 − 3𝑎𝑏𝑐 habis dibagi oleh 𝑎 + 𝑏 + 𝑐 !

Pembahasan :
Untuk membuktikannya kita coba hanya menggunakan persamaan umum :

𝑥 3 + 𝑦 3 = (𝑥 + 𝑦)3 − 3𝑥𝑦(𝑥 + 𝑦)

𝑎3 + 𝑏 3 + 𝑐 3 − 3𝑎𝑏𝑐 = (𝑎3 + 𝑏 3 ) + 𝑐 3 − 3𝑎𝑏𝑐


= (𝑎 + 𝑏)3 − 3𝑎𝑏(𝑎 + 𝑏) + 𝑐 3 − 3𝑎𝑏𝑐
= (𝑎 + 𝑏)3 + 𝑐 3 − 3𝑎𝑏(𝑎 + 𝑏) − 3𝑎𝑏𝑐
= (𝑎 + 𝑏 + 𝑐)3 − 3(𝑎 + 𝑏)𝑐 (𝑎 + 𝑏 + 𝑐) − 3𝑎𝑏(𝑎 + 𝑏 + 𝑐)
= (𝑎 + 𝑏 + 𝑐)((𝑎 + 𝑏 + 𝑐)2 − 3(𝑎 + 𝑏)𝑐 − 3𝑎𝑏)
= (𝑎 + 𝑏 + 𝑐)(𝑎2 + 𝑏 2 + 𝑐 2 − 𝑎𝑐 − 𝑏𝑐 − 𝑎𝑐)

Terbukti, karena (𝑎 + 𝑏 + 𝑐) adalah faktor dari 𝑎3 + 𝑏 3 + 𝑐 3 − 3𝑎𝑏𝑐

62 | A L J A B A R
Olimpiade Matematika SMA 2020

Soal & Pembahasan nomor 3 :


Untuk 𝑛 ∈ Asli dimana 𝑛4 + 2𝑛3 − 14𝑛2 + 2𝑛 − 8 jika dibagi 𝑛2 + 1 bersisa 7,
maka nilai n yang mungkin adalah ....

Pembahasan :
𝑛4 + 2𝑛3 − 14𝑛2 + 2𝑛 − 8  7 (mod 𝑛2 + 1)
𝑛4 + 2𝑛3 − 14𝑛2 + 2𝑛 − 15  0 (mod 𝑛2 + 1)
Maka 𝑛2 + 1 adalah faktor dari 𝑛4 + 2𝑛3 − 14𝑛2 + 2𝑛 − 15
𝑛4 + 2𝑛3 − 14𝑛2 + 2𝑛 − 15 = (𝑛2 + 1 ) (𝑛2 + 𝑎𝑛 − 15)
𝑛4 + 2𝑛3 − 14𝑛2 + 2𝑛 − 15 = 𝑛4 + 𝑎𝑛3 − 15𝑛2 + 𝑎𝑛 − 15
Diperoleh 𝑎 = 2, sehingga faktor lainnya 𝑛2 + 2𝑛 − 15

Jika kita memandang sistem persamaan keterbagian berikut :

𝑛4 + 2𝑛3 − 14𝑛2 + 2𝑛 − 8 7
𝑛2 + 2𝑛 − 15 = 2
− 2
𝑛 +1 𝑛 +1
2
Maka nilai 𝑛 jika 𝑛 + 2𝑛 − 15 = 0
𝑛4 + 2𝑛3 − 14𝑛2 + 2𝑛 − 8  7 (mod 𝑛2 + 1), dengan hasil bagi 𝑛2 + 2𝑛 − 15 = 0
Sehingga :
𝑛2 + 2𝑛 − 15 = 0
(𝑛 − 3)(𝑛 + 5) = 0
Sehingga nilai 𝑛 ∈ Asli yang mungkin adalah 3

Soal & Pembahasan nomor 4 :


Diberikan fungsi 𝑓(𝑥) = 𝑥 2 + 4. Misalkan x dan y adalah bilangan-bilangan real
positif yang memenuhi :
𝑓(𝑥𝑦) + 𝑓(𝑦 − 𝑥) = 𝑓(𝑦 + 𝑥)
Maka nilai minimum x + y adalah ....
Pembahasan :
𝑓(𝑥) = 𝑥 2 + 4
𝑓(𝑥𝑦) + 𝑓(𝑦 − 𝑥) = 𝑓(𝑦 + 𝑥)
(𝑥𝑦)2 + 4 + (𝑦 − 𝑥)2 + 4 = (𝑦 + 𝑥)2 + 4
(𝑥𝑦)2 + 4 + 𝑦 2 − 2𝑥𝑦 + 𝑥 2 + 4 = 𝑦 2 + 2𝑥𝑦 + 𝑥 2 + 4
(𝑥𝑦)2 + 4 − 2𝑥𝑦 = 2𝑥𝑦
(𝑥𝑦)2 − 4𝑥𝑦 + 4 = 0 atau dapat ditulis sebagai (𝑥𝑦 − 2)2 = 0
Karena 𝑥 dan 𝑦 bilangan real positif, maka 𝑥𝑦 = 2
Untuk 𝑥 dan 𝑦 real positif berlaku (𝑥 − 𝑦)2 ≥ 0 sehingga 𝑥 + 𝑦 ≥ 2𝑥𝑦
Maka nilai minimum 𝑥 + 𝑦 = 2√2

63 | A L J A B A R
Olimpiade Matematika SMA 2020

Soal & Pembahasan nomor 5 :


Jika 𝑎, 𝑏, 𝑐, 𝑑 adalah bilangan-bilangan bulat positif dimana 𝑎 + 𝑏 + 𝑐 + 𝑑 = 24,
nilai maksimum 𝑎𝑏 + 𝑏𝑐 + 𝑐𝑑 + 𝑎𝑑 adalah ...
Pembahasan :
𝑎𝑏 + 𝑏𝑐 + 𝑐𝑑 + 𝑎𝑑 = (𝑎 + 𝑐)(𝑏 + 𝑑)
Berdasarkan 𝐴𝑀 ≥ 𝐺𝑀, maka 𝑎 + 𝑏 + 𝑐 + 𝑑 ≥ 2√(𝑎 + 𝑐)(𝑏 + 𝑑)

2√(𝑎 + 𝑐)(𝑏 + 𝑑) ≤ 𝑎 + 𝑏 + 𝑐 + 𝑑

2√(𝑎 + 𝑐)(𝑏 + 𝑑) ≤ 24

√(𝑎 + 𝑐)(𝑏 + 𝑑) ≤ 12
(𝑎 + 𝑐)(𝑏 + 𝑑) ≤ 144
𝑎𝑏 + 𝑏𝑐 + 𝑐𝑑 + 𝑎𝑑 ≤ 144
Jadi nilai maksimum 𝑎𝑏 + 𝑏𝑐 + 𝑐𝑑 + 𝑎𝑑 adalah 144

Soal & Pembahasan nomor 6 :


2𝑥 2 𝑐𝑜𝑠2 𝑥 + 8 
Misalkan 𝑓(𝑥) = untuk 0 < 𝑥 < 2 , Nilai minimum 𝑓(𝑥) = ....
𝑥 cos 𝑥
Pembahasan :

2𝑥 2 𝑐𝑜𝑠2 𝑥 + 8
𝑓(𝑥) =
𝑥 cos 𝑥

2𝑥 2 𝑐𝑜𝑠2 𝑥 8
𝑓(𝑥) = + , dimisalkan sebagai 𝑓(𝑥) = 𝐴 + 𝐵
𝑥 cos 𝑥 𝑥 cos 𝑥

Karena 0 < 𝑥 < 2 maka 𝐴 > 0 dan 𝐵 > 0
maka berdasarkan AM ≥ GM , haruslah 𝐴 + 𝐵 ≥ 2√𝐴𝐵

𝑓(𝑥) ≥ 2√𝐴𝐵

2𝑥 2 𝑐𝑜𝑠 2 𝑥 8
𝑓(𝑥) ≥ 2 √( )( )
𝑥 cos 𝑥 𝑥 cos 𝑥
𝑓(𝑥) ≥ 2√16

Jadi nilai minimum 𝑓(𝑥) adalah 8

64 | A L J A B A R
Olimpiade Matematika SMA 2020

Soal & Pembahasan nomor 7 :


2
Jumlah semua nilai 𝑥  𝐵 agar log(𝑥 2 − 6𝑥 − 3) juga merupakan bilangan bulat
adalah ...
Pembahasan :
2
Misalkan log(𝑥2 − 6𝑥 − 3) = 𝑝 dengan p adalah bilangan bulat
2
log ((𝑥 − 3)2 − 12) = 𝑝
(𝑥 − 3)2 − 12 = 2𝑝
(𝑥 − 3)2 = 2𝑝 + 12
Agar (𝑥 − 3)  B , maka 2𝑝 + 12 haruslah bilangan kuadrat sempurna, sehingga nilai 𝑝
yang memenuhi adalah 2

(𝑥 − 3)2 = 22 + 12 = 16
(𝑥 − 3)2 = (4)2 atau (𝑥 − 3)2 = (−4)2
Nilai 𝑥 yang memenuhi adalah 7 dan – 1. Jumlah semua nilai x adalah 6

Soal & Pembahasan nomor 8 :


Diketahui barisan bilangan real 𝑎1 , 𝑎2 , 𝑎3 , … . 𝑎𝑛 merupakan barisan geometri. Jika 𝑎1 +
𝑎4 = 20 , tentukan nilai minimum dari jumlah 6 suku pertama deret ini !
Pembahasan :

Pada deret geometri, 𝑎1 + 𝑎4 = 20 berarti 𝑎1 + 𝑎1 𝑟 3 = 20


𝑎1 + 𝑎2 + 𝑎3 + 𝑎4 + 𝑎5 + 𝑎6 = (𝑎1 + 𝑎4 ) + (𝑎2 + 𝑎5 ) + (𝑎3 + 𝑎6 )
𝑆6 = (𝑎1 + 𝑎1 𝑟 3 ) + 𝑟 (𝑎1 + 𝑎1 𝑟 3 ) + 𝑟 2 (𝑎1 + 𝑎1 𝑟 3 )
𝑆6 = 20 + 𝑟 20 + 𝑟 2 20
(1)2 −4 .1 .1 3
𝑆6 = 20 (𝑟 2 + 𝑟 + 1) , nilai minimum dari 𝑟 2 + 𝑟 + 1 = − =4
4
3
Jadi nilai minimum 𝑎1 + 𝑎2 + 𝑎3 + 𝑎4 + 𝑎5 + 𝑎6 = 4 (20) = 15

Soal & Pembahasan nomor 9 :


2021 2020
Buktikan 2020 ! ≤ ( )
2

Pembahasan :
Oke langsung kita misalkan 2020 dengan 𝑛 saja biar keliatannya enak.
𝑛+1 𝑛
Jadi, kita akan melakukan pembuktian 𝑛 ! ≤ ( )
2

Rata-rata Aritmetik dari jumlah n suku pertama bilangan asli adalah


1 + 2 + 3 + 4 +5 + ⋯+ 𝑛
𝐴𝑀 =
𝑛

65 | A L J A B A R
Olimpiade Matematika SMA 2020

Rata-rata Geometrik dari hasilkali n suku pertama bilangan asli adalah


𝑛
𝐺𝑀 = √1 . 2 . 3 . 4 . 5 . 6 . . . . . . . . . . 𝑛

Maka menurut 𝐴𝑀 ≥ 𝐺𝑀
1+2+3+4+5+⋯+𝑛 𝑛
≥ 𝑛 √1 . 2 . 3 . 4 . 5 … . 𝑛
𝑛
1
Mengingat 1 + 2 + 3 + 4 + 5 + ⋯ + 𝑛 = 2 𝑛 (𝑛 + 1) , maka :

1
2 𝑛 (𝑛 + 1) ≥ 𝑛 𝑛√𝑛 !
𝑛
(𝑛 + 1) 𝑛
≥ 𝑛 √𝑛 !
2
𝑛+1 𝑛
( ) ≥ 𝑛!
2
𝑛+1 𝑛
𝑛! ≤ ( )
2
2021 2020
Jadi jika n dikembalikan sebagai 2020, maka 2020 ! ≤ ( ) terbukti.
2

Soal & Pembahasan nomor 10 :


Jika 𝑎, 𝑏, 𝑐, 𝑑  𝑅 +, buktikan bahwa :

1 1 1 𝑎 𝑏 𝑐 1 1 1 3 3
(𝑎 + 𝑏 + 𝑐 ) ( + + ) ( + + ) ( + + ) (𝑎 + 𝑏3 + 𝑐 3 ) ≥ 81
𝑎 𝑏 𝑐 𝑏 𝑐 𝑎 3𝑎 3𝑏 3𝑐

Pembahasan :

Untuk 𝑎 , 𝑏 dan 𝑐 real positif, berlaku 𝐴𝑀 ≥ 𝐺𝑀 ≥ 𝐻𝑀 telah dibuktikan


sebelumnya :
1 1 1
(𝑎 + 𝑏 + 𝑐) ( + + ) ≥ 9 ................... (1)
𝑎 𝑏 𝑐

𝑎 𝑏 𝑐
+ + ≥ 3 ............................ (2)
𝑏 𝑐 𝑎
1 1 1 3
Mencari hubungan (
3𝑎
+
3𝑏
+
3𝑐
) (𝑎3 + 𝑏 3 + 𝑐 3 ) ≥ 3

66 | A L J A B A R
Olimpiade Matematika SMA 2020

𝐺𝑀 ≥ 𝐻𝑀
3 3
√3𝑎 . 3𝑏 . 3𝑐 ≥
1 1 1
+ +
3𝑎 3𝑏 3𝑐

3 1 1 1
√𝑎𝑏𝑐 ( + + )≥1
3𝑎 3𝑏 3𝑐

1 1 1 3
(𝑎𝑏𝑐) ( + + ) ≥ 1 ............................ (3)
3𝑎 3𝑏 3𝑐

𝐴𝑀 ≥ 𝐺𝑀
3
𝑎 3 + 𝑏 3 + 𝑐 3 ≥ 3 √𝑎 3 . 𝑏 3 . 𝑐 3

𝑎3 + 𝑏3 + 𝑐 3 ≥ 3𝑎𝑏𝑐

𝑎3 + 𝑏3 +𝑐 3
𝑎𝑏𝑐 = ............................ (4)
3

Substitusi (3) dan (4)

1 1 1 3
(3𝑎 + 3𝑏 + 3𝑐) (𝑎3 + 𝑏 3 + 𝑐 3 ) ≥ 3 ............................ (5)

Substitusi (1), (2) dan (5) menghasilkan


(1). (2). (5) ≥ 9.3.3

1 1 1 𝑎 𝑏 𝑐 1 1 1 3 3
(𝑎 + 𝑏 + 𝑐 ) ( + + ) ( + + ) ( + + ) (𝑎 + 𝑏3 + 𝑐 3 ) ≥ 81
𝑎 𝑏 𝑐 𝑏 𝑐 𝑎 3𝑎 3𝑏 3𝑐

67 | A L J A B A R
Olimpiade Matematika SMA 2020

Catatan :
........................................... ....................................... .................................................
........................................... ....................................... .................................................
........................................... ....................................... .................................................
........................................... ....................................... .................................................
........................................... ....................................... .................................................
........................................... ....................................... .................................................
........................................... ....................................... .................................................
........................................... ....................................... .................................................
........................................... ....................................... .................................................
........................................... ....................................... .................................................
........................................... ....................................... .................................................
........................................... ....................................... .................................................
........................................... ....................................... .................................................
........................................... ....................................... .................................................
........................................... ....................................... .................................................
........................................... ....................................... .................................................
........................................... ....................................... .................................................
........................................... ....................................... .................................................
........................................... ....................................... .................................................
........................................... ....................................... .................................................
........................................... ....................................... .................................................
........................................... ....................................... .................................................
........................................... ....................................... .................................................
........................................... ....................................... .................................................
........................................... ....................................... .................................................
........................................... ....................................... .................................................
........................................... ....................................... .................................................
........................................... ....................................... .................................................
........................................... ....................................... .................................................
........................................... ....................................... .................................................
........................................... ....................................... .................................................
........................................... ....................................... .................................................
........................................... ....................................... .................................................
........................................... ....................................... .................................................
........................................... ....................................... .................................................

68 | A L J A B A R
Olimpiade Matematika SMA 2020

TEORI BILANGAN
Bab2
PENGANTAR TEORI

• TEOREMA KETERBAGIAN

Teori bilangan tentunya memiliki ruang kajian yang lebih khusus tentang
bilangan dan sifat-sifat operasi aritmatikanya, walaupun di dalam Aljabar juga
memuat materi sistem bilangan real yang memulai pembahasan jenis, sifat
bilangan dan aksioma seperti yang telah dibahas sebelumnya.
3𝑛+25
Misalnya ada soal, menentukan semua bilangan asli n sehingga juga
2𝑛−5
3𝑛+25
merupakan bilangan asli. Secara aljabar biasa, kita anggap sebuah
2𝑛−5
bilangan pecahan.

3𝑛 + 25 3 . 2 (3𝑛 + 25) 3 . (6𝑛 + 50) 3 . (6𝑛 − 15 + 65) 3 1 65


= = = = + ( )
2𝑛 − 5 2 . 3 (2𝑛 − 5) 2 . (6𝑛 − 15) 2 . (6𝑛 − 15) 2 2 2𝑛 − 5

3 1 65 65
Agar + ( ) bilangan Asli, maka 2𝑛−5 harus bilangan Asli.
2 2 2𝑛−5
Maka 2n – 5 haruslah bernilai 1, 5, 13 atau 65, sehingga nilai n yang memenuhi
adalah 3, 5, 9 atau 35.

Dengan konsep Keterbagian


3𝑛+25
Agar merupakan bilangan Asli haruslah 2𝑛 − 5|3𝑛 + 25
2𝑛−5
2𝑛 − 5|2(3𝑛 + 25) − 3(2𝑛 − 5)
2𝑛 − 5|65 , akibatnya 2n – 5 adalah faktor dari 65, yaitu 2n – 5 = 1, 5, 13 dan 65
Sehingga n = 3, 5, 9 dan 35.

Tampaknya dengan konsep keterbagian, soal tersebut lebih mudah diselesaikan.

69 | T E O R I B I L A N G A N
Olimpiade Matematika SMA 2020

Contoh Soal 1
Jika 𝑎 bilangan ganjil, maka 𝑎2 + (𝑎 + 2)2 + (𝑎 + 3)2 + 8(𝑎 + 8) + 5 jika dibagi 12
akan bersisa .......

Pembahasan :
Rumus umum bilangan ganjil adalah 2𝑛 − 1 dengan 𝑛 bilangan asli.
Misalkan , 𝐴 = 𝑎2 + (𝑎 + 2)2 + (𝑎 + 3)2 + 8(𝑎 + 8) + 5
𝐴 = (2𝑛 − 1)2 + (2𝑛 − 1 + 2)2 + (2𝑛 − 1 + 3)2 + 8(2𝑛 − 1 + 8) + 5
𝐴 = (2𝑛 − 1)2 + (2𝑛 + 1)2 + (2𝑛 + 2)2 + 8(2𝑛 + 7) + 5
𝐴 = 4𝑛2 − 4𝑛 + 1 + 4𝑛2 + 4𝑛 + 1 + 4𝑛2 + 8𝑛 + 4 + 16𝑛 + 56 + 5
𝐴 = 12𝑛2 + 24𝑛 + 67
𝐴 = 12(𝑛2 + 2𝑛 + 5) + 7 , maka pembagian A dengan 12 menghasilkan sisa 7.

Keistimewaan Bilangan Pembagi


Sebelum memasuki pembahasan teori bilangan yang lebih dalam, kita akan
mengenal dulu beberapa bilangan pembagi yang istimewa, seperti :

• Suatu bilangan habis dibagi 2 jika digit bilangan terakhhir habis dibagi 2.
• Suatu bilangan habis dibagi 4 jika 2 digit bilangan terakhir habis dibagi 4.
• Suatu bilangan habis dibagi 8 jika 3 digit bilangan terakhir habis dibagi 8.
• Bilangan a habis dibagi 3 jika jumlah angka-angkanya habis dibagi 3
• Bilangan a habis dibagi 9 jika jumlah angka-angkanya habis dibagi 9
• Bilangan a habis dibagi 11 jika selisih jumlah angka-angka genap dan
ganjilnya 0 atau habis dibagi 11

Contoh Soal 2
Apakah bilangan 7173628 dan 6008316457 habis dibagi 11 ?

Pembahasan :
7173628 terdiri dari 7 digit bilangan.
Jumlah digit bilangan urutan genap = 1 + 3 + 2 = 6
Jumlah digit bilangan urutan ganjil = 7 + 7 + 6 + 8 = 28
Selisih jumlah digit berselang seling = 28 – 6 = 22 (kelipatan 11)
Maka 7173628 habis dibagi 11
• 6008316457 terdiri dari 10 digit bilangan. Kalau kita hitung, selisih jumlah
angka-angka genap dan ganjilnya adalah 0, maka 6008316457 habis dibagi 11

70 | T E O R I B I L A N G A N
Olimpiade Matematika SMA 2020

Contoh Soal 3
Jika 𝑎 dan 𝑏 adalah bilangan asli, sehingga bilangan 1𝑎37𝑏6 habis dibagi 9
tetapi tidak habis dibagi 8, maka banyaknya pasangan (𝑎 , 𝑏) yang memenuhi
adalah ....

Pembahasan :
Bilangan 1𝑎37𝑏6 habis dibagi delapan jika 3 digid terakhirnya juga habis
dibagi 8. Tiga digid terakhir nya adalah 7b6.
Jadi nilai 𝑏 agar 7𝑏6 habis dibagi 8 adalah 3 dan 7.
736  0 mod 8 , artinya 736 jika dibagi 8 bersisa 0
776  0 mod 8 , artinya 776 jika dibagi 8 bersisa 0
Maka nilai 𝑏 yang membuat 1𝑎37𝑏6 tidak habis dibagi 8 adalah selain 3 dan
7, yaitu : 0, 1, 2, 4, 5. 6, 8 dan 9

Sekarang kita bahas bilangan yang habis dibagi 9.


Nilai 𝑎 yang membuat 1𝑎37𝑏6  0 mod 9 dengan 𝑏 = (0, 1, 2, 4, 5. 6, 8, 9) ada
sebanyak bilangan yang ada pada 𝑏.
Maka banyaknya pasangan (𝑎 , 𝑏) yang memenuhi adalah 8.

Contoh Soal 4
Pengembangan dari 915 menghasilkan 205.891.1m2.094.64n. Nilai m + n adalah ...

Pembahasan :
Kita mengetahui bahwa 91 = 9 , 92 = 81, 93 = 729 , 94 = 6561 dan seterusnya.
9 pangkat bilangan ganjil digit terakhirnya selalu 9
9 pangkat bilangan genap digit terakhirnya selalu 1
Maka digid terakhir 915 adalah 9, sehingga n = 9
Syarat bilangan kelipatan 9, memiliki jumlah digid yang selalu 9, maka
205.891.1m2.094.649  2 + 0 + 5 + 8 + 9 + 1 + 1 + m + 2 + 0 + 9 + 4 + 6 + 4 + 9
26 + m + 34  2 + 6 + m + 3 + 6
17 + m  1 + 7 + m  8 + m
8 + m = 9 , m = 1 , Jadi m + n = 1 + 9 = 10

Teorema Keterbagian
Misalkan a dan b adalah bilangan bulat dengan a  0. Bilangan a dikatakan habis
membagi b jika terdapat bilangan bulat k sehingga b = ka. Untuk selanjutnya kita
tulis a|b , contohnya 6|42 , terdapat bilangan bulat k = 7 sehingga 42 = 7 × 6

71 | T E O R I B I L A N G A N
Olimpiade Matematika SMA 2020

Ada beberapa sifat keterbagian :


• Untuk setiap bilangan bulat a yang tidak nol selalu berlaku 𝑎|𝑎 dan 𝑎|0
• Untuk setiap bilangan bulat a selalu berlaku 1|𝑎 jika 𝑎|𝑏 maka |𝑎| ≤ |𝑏|
• 𝑎𝑐|𝑏𝑐 untuk setiap bilangan bulat c yang tidak nol.
• Jika 𝑎|𝑏 dan 𝑎|𝑐 maka 𝑎|(𝑚𝑏 + 𝑛𝑐) untuk setiap bilangan bulat m dan n.

Bilangan Prima
Pada pembahasan keterbagian, kita kenal istilah a membagi b, untuk selanjutnya
pernyataan a membagi b dapat kita katakan a faktor dari b.
• Bilangan bulat positif p dikatakan bilangan prima jika p mempunyai tepat
dua faktor positif yaitu 1 dan p sendiri.
• Bilangan bulat positif n dikatakan bilangan komposit jika n mempunyai
lebih dari 2 faktor positif.
• Setiap bilangan prima p > 3 selalu dapat kita nyatakan sebagai 6k + 1 atau
6k – 1, dengan 𝑘 bilangan Asli
• Teorema Eratosthenes, untuk setiap bilangan komposit n ada bilangan
prima p sehingga 𝑝|𝑛 dan 𝑝 ≤ √𝑛
• Jika p bilangan prima dan 𝑝|𝑛 maka 𝑝2 |𝑛2

Banyak bilangan prima adalah tak hingga.

Contoh Soal 5
Tentukan bilangan prima yang berbentuk 𝑛3 + 1 untuk suatu bilangan asli n.

Pembahasan :
Perhatikan bahwa 𝑛3 + 1 = (𝑛 + 1)(𝑛2 − 𝑛 + 1) dengan demikian salah satu
faktor yaitu 𝑛 + 1 atau 𝑛2 − 𝑛 + 1 harus sama dengan 1.
Jika 𝑛 + 1 = 1 maka n = 0 (tidak memenuhi)
Jika 𝑛2 − 𝑛 + 1 = 1 maka n = 0 atau n = 1. Untuk n = 1 kita peroleh 𝑛3 + 1 = 2
Jadi bilangan prima yang berbentuk 𝑛3 + 1 adalah 2.

72 | T E O R I B I L A N G A N
Olimpiade Matematika SMA 2020

Contoh Soal 6
Jika n bilangan asli, buktikan bahwa 𝑛3 + 5𝑛 habis dibagi 6

Pembahasan :
𝑛3 + 5𝑛 = 𝑛3 − 𝑛 + 6𝑛 = (𝑛 − 1)𝑛 (𝑛 + 1) + 6𝑛
(𝑛 − 1)𝑛 (𝑛 + 1) merupakan 3 bilangan yang berurutan, jadi selalu habis dibagi 6.
Jelas bahwa 6|6𝑛 , maka 6|𝑛3 + 5𝑛

Contoh Soal 7
Jika 3|𝑎 + 4𝑏 , buktikan bahwa 3|10𝑎 + 𝑏

Pembahasan :
3|𝑎 + 4𝑏
3|𝑎 + 𝑏 + 3𝑏 , karena tentulah 3|3𝑏, maka 3|𝑎 + 𝑏
10𝑎 + 𝑏 = 9𝑎 + 𝑎 + 𝑏
tentulah 3|9𝑎 dan sebelumnya 3|𝑎 + 𝑏
maka terbukti 3|10𝑎 + 𝑏

Contoh Soal 8
Jika 7|2𝑥 + 5 apakah 7|6𝑥 2 + 𝑥 − 35 ?

Pembahasan :
6𝑥 2 + 𝑥 − 35 = 3𝑥(2𝑥 + 5) − 14𝑥 − 35
6𝑥 2 + 𝑥 − 35 = 3𝑥(2𝑥 + 5) − 7(2𝑥 + 5)
7|3𝑥(2𝑥 + 5) karena 7|2𝑥 + 5
2𝑥 + 5|6𝑥 2 + 𝑥 − 35 , maka benar bahwa 7|6𝑥 2 + 𝑥 − 35

Contoh Soal 9
Jika p > 3 dengan p bilangan prima, buktikan bahwa 24|𝑝2 − 1

Pembahasan :
Untuk p bilangan prima, maka p – 1 dan p + 1 adalah bilangan genap
Tentunya p + 1 habis dibagi 2 dan p – 1 habis dibagi 4
Maka 𝑝2 − 1 habis dibagi 8, atau dapat ditulis 8|𝑝2 − 1

Untuk p > 3 maka ada banyak p yang tidak habis dibagi 3. Namun salah satu
dari p – 1 dan p + 1 pastilah habis dibagi 3, maka dapat ditulis 3|𝑝2 − 1.
Karena 8|𝑝2 − 1 dan 3|𝑝2 − 1 maka terbukti bahwa 24|𝑝2 − 1

73 | T E O R I B I L A N G A N
Olimpiade Matematika SMA 2020

• FPB dan KPK

Faktor Persekutuan Terbesar (FPB)


Kita telah mengenal Faktor persekutuan terbesar (FPB), yaitu pembagi sekutu
terbesar atau disebut juga greatest common divisor (gcd).

Perhatikan beberapa teorema berikut ini :

• Bilangan asli d disebut pembagi sekutu terbesar dari a dan b atau ditulis
dengan d = gcd (a, b) jika
(i). 𝑑|𝑎 dan 𝑑|𝑏 , menyatakan bahwa d adalah pembagi sekutu dari a dan b
(ii). untuk setiap bilangan asli c dengan 𝑐|𝑎 dan 𝑐|𝑏 haruslah berlaku c ≤ d,
berarti d merupakan pembagi sekutu yang terbesar.

• Bilangan bulat a dan b dikatakan saling prima jika gcd (a, b) = 1.

• Untuk sebarang bilangan bulat a, b dan c didefinisikan


gcd (a, b, c) = gcd (gcd (a, b) , c) = gcd (a, gcd (b, c))

Maka dapat diturunkan beberapa sifat di bawah ini:


1. gcd (a, b) = gcd (b, a) = gcd (|𝑎|, |𝑏|)
2. gcd (a, 1) = 1 untuk setiap bilangan bulat a,
3. gcd (a, 0) = |𝑎| untuk setiap bilangan bulat tak nol a,
4. gcd (ma, mb) = |𝑚| gcd (a, b) untuk setiap bilangan bulat tak nol m,

Menentukan gcd dua bilangan dengan algoritma Euclide


Misalkan a dan b bilangan bulat yang tidak keduanya nol.
gcd (a, b) = gcd (b, a) = gcd (|𝑎|, |𝑏|), untuk a dan b bilangan asli dengan a > b.

Menentukan gcd (540, 138)


gcd (540, 138) = gcd (3 x 138 + 126 , 138)
gcd (540, 138) = gcd (126 , 1 x 126 + 12)
gcd (126, 12) = gcd (10 x 12 + 6 , 12)
gcd (126, 12) = gcd (6 , 2 x 6 + 0)
gcd (126, 12) = gcd (6 , 0)
gcd (540, 138) = 6

74 | T E O R I B I L A N G A N
Olimpiade Matematika SMA 2020

Menentukan gcd (1353 , 1716)


gcd (1353 , 1716) = gcd (1716 , 1353)
gcd (1716 , 1353) = gcd (1 x 1353 + 363 , 1353)
gcd (1716 , 1353) = gcd (363 , 3 x 363 + 264)
gcd (363 , 264) = gcd (1 x 264 + 99, 264)
gcd (363 , 264) = gcd (99, 2 x 99 , 66)
gcd (99 , 66) = gcd (1 x 66 + 33 , 66)
gcd (99 , 66) = gcd (33 , 2 x 33 + 0)
gcd (99 , 66) = gcd (33 , 0)
gcd (1353 , 1716) = 33

Menentukan gcd (16524 , 73756)


gcd (16524 , 73756) = gcd (73756 , 16524)
gcd (73756 , 16524) = gcd (4 x 16524 + 7660 , 16524)
gcd (73756 , 16524) = gcd (7660 , 2 x 7660 + 1204)
gcd (7660 , 1204) = gcd (6 x 1204 + 436 , 1204)
gcd (7660 , 1204) = gcd (436 , 2 x 436 + 332)
gcd (436 , 332) = gcd (1 x 332 + 104 , 332)
gcd (436 , 332) = gcd (104 , 3 x 104 + 20)
gcd (104 , 20) = gcd (5 x 20 + 4 , 20)
gcd (4 , 20) = gcd (4 , 5 x 4 + 0)
gcd (4 , 20) = gcd (4 , 0)
gcd (16524 , 73756) = 4

Dengan menggunakan teorema di atas kita dapat menurunkan beberapa sifat


sebagai berikut :

• Jika d = gcd (a, b) maka untuk sebarang bilangan bulat c dengan 𝑐|𝑎 dan
𝑐|𝑏 haruslah berlaku 𝑐|𝑑.
• Jika a|bc dan gcd (a, b) = 1, maka 𝑎|𝑐.

Perhatikan bahwa 𝑎|𝑏𝑐, artinya terdapat bilangan bulat k sehingga bc = ka. Selain
itu, kita juga punya gcd (a, b) = 1. Kita dapat menemukan bilangan bulat x dan y
dengan sifat 𝑎𝑥 + 𝑏𝑦 = 1.

75 | T E O R I B I L A N G A N
Olimpiade Matematika SMA 2020

Contoh Soal 10
Diberikan gcd (15, 24) = 3
Cari salah satu pasangan bulat (x, y) sehingga 15x + 24y = 3.

Pembahasan :
Perhatikan, menurut Algoritma Euclide :
24 = 15.1 + 9
15 = 9.1 + 6
9 = 6.1 + 3
Dengan demikian
3=9−6
3 = 9 − (15 − 9)
3 = 2.9 − 15
3 = 2 (24 − 15) − 15
3 = −3.15 + 2.24
Persamaan terakhir 15 (−3) + 24 (2) menujukan bahwa x = −3 dan y = 2.
Jadi salah satu pasangan bulat (x, y) sehingga 15x + 24y = 3 adalah (−3 , 2)

Kelipatan persekutuan terkecil (KPK)


Selain pembagi sekutu terbesar, kita juga mengenal kelipatan persekutuan
terkecil (KPK). Dalam pembahasan selanjutnya, untuk sebarang bilangan bulat a
dan b, KPK dari a dan b kita tulis dengan [a, b] .

Bilangan bulat positif m disebut KPK dari bilangan bulat bukan nol a, b dan c
jika :
• 𝑚|𝑎 dan 𝑚|𝑏
• untuk setiap bilangan bulat positif n dengan 𝑛|𝑎 dan 𝑛|𝑏 haruslah m ≤ n.
• Jika a dan b adalah bilangan bulat yang tidak keduanya nol, maka
𝑎𝑏
[𝑎 , 𝑏] =
gcd (𝑎 , 𝑏)
maka :
1. [a, b] = [b, a]
2. [a, 0] = 0 untuk setiap bilangan bulat tak nol a,
3. [a, 1] = |𝑎| untuk setiap bilangan bulat a.

• KPK dari a, b, dan c didefinisikan sebagai [a, b, c] = [[a, b] , c] = [a, [b, c]]

76 | T E O R I B I L A N G A N
Olimpiade Matematika SMA 2020

Contoh Soal 11
Hitung [540 , 138] .
Pembahasan :
Karena gcd (540, 138) = 6 maka :

540 . 138 540 . 138


[540, 138] = = = 12420
gcd(540 , 138) 6

Contoh Soal 12
Tentukan bilangan bulat positif terkecil lebih dari 1 yang bersisa 1 ketika dibagi k
untuk setiap 2 ≤ k ≤ 10.

Pembahasan :
KPK dari 2, 3, 4, 5, 6, 7, 8, 9 dan 10 atau [2, 3,4, 5, 6, 7. 8, 9, 10] = 2520
Dengan demikian 2520 adalah bilangan yang habis dibagi oleh 2, 3, 4, 5, 6, 7, 8, 9
dan 10. Maka jika harus menghasilkan sisa 1 dari pembagian ini, maka bilangan
yang dimaksud adalah 2521.

Faktorisasi Prima
Setiap bilangan asli n > 1 dapat dinyatakan secara tunggal sebagai
𝑎 𝑎 𝑎 𝑎
𝑛 = 𝑝1 1 . 𝑝2 2 . 𝑝3 3 . . . 𝑝𝑘 𝑘
dengan k suatu bilangan asli, 𝑝1 < 𝑝2 < 𝑝3 < ⋯ . < 𝑝𝑘 bilangan-bilangan prima
berbeda , dan 𝑎𝑖 ≥ 1 , untuk setiap 𝑖 = 1, 2, 3, ..., k.
maka kita mengetahui beberapa hal sebagai berikut:
• n mempunyai k faktor prima yaitu 𝑝1 , 𝑝2 , 𝑝3 , … . 𝑝𝑘 .
• Banyak faktor positif dari n adalah (1 + 𝑎1 ) (1 + 𝑎2 ) ... (1 + 𝑎𝑘 )

Contoh Soal 13
Tentukan banyaknya faktor positif masing-masing dari 2016 dan 2020
Pembahasan :
2016 = 25 𝑥 33 𝑥 7
Banyaknya faktor positif dari 2016 adalah = (5 + 1) (3 + 1) (1 + 1) = 36

2020 = 22 𝑥 5 𝑥 101
Banyaknya faktor positif dari 2020 adalah = (2 + 1) (1 + 1) (1 + 1) = 12
Yaitu (1, 2, 4, 5, 10, 20, 101, 202, 404, 505, 1010, 2020)

77 | T E O R I B I L A N G A N
Olimpiade Matematika SMA 2020

• KEKONGRUENAN

Konsep tentang kongruensi ini banyak digunakan dalam menyelesaikan soal-


soal yang berkaitan dengan mencari sisa suatu bilangan jika dibagi dengan
bilangan lain atau mencari angka satuan suatu bilangan.

Misalkan n adalah suatu bilangan bulat positif, a dan b adalah suatu bilangan
bulat, a dikatakan kongruen b modulo n, ditulis a  b (mod n) , jika dan hanya
jika a – b adalah kelipatan n.

Penulisan 3  24 (mod 7) sudah benar karena 3 – 24 = – 21 (kelipatan 7).


Contoh lain, penulisan 13  –1 (mod 7), benar karena 13 + 1 = 14 kelipatan dari 7

3  24 (mod 7) juga bisa dibaca “24 dibagi 7 sisanya 3”. Sebelum melangkah lebih
jauh, perhatikan beberapa contoh Aritmatika Modulo dibawah ini :

Contoh 1 :
• 33  3 (mod 10)
• 17  7 (mod 10)
• 16  –1 (mod 9)  7 (mod 9)
• 33 . 17  3 . 7 (mod 10)  21 (mod 10)  1 (mod 10)
• 32 . 15 + 18 . 42  (4 . 1 + 4 . 0) (mod 7)  4 (mod 7)

Untuk pedoman contoh 2 dibawah ini, perlu diingat tentang eksponen.

(𝑎)𝑏𝑥𝑐 = (𝑎𝑏 )𝑐 dan (𝑎)𝑏+𝑐 = 𝑎𝑏 x 𝑎𝑐

Contoh 2 :
• 23  1 (mod 7)
• 25 = 22 . 23  4 . (1) (mod 7)  4 (mod 7)
• 210 = 21 . (23 )3  2 . (1)3 (mod 7)  2 (mod 7)
• 35 = 243  3 (mod 10), satu digit terakhir 35 adalah 3
• 12345  45 (mod 100), dua digit terakhir 12345 adalah 45
• 52020 = (54 )505  (1)505 (mod 13)  1 (mod 13)

78 | T E O R I B I L A N G A N
Olimpiade Matematika SMA 2020

Misalkan n suatu bilangan bulat positif dan a, b, c, dan d bilangan bulat sebarang
berlaku sifat-sifat berikut :
• a  a (mod n)
• Jika a  b (mod n) maka b  a (mod n)
• Jika a  b (mod n) dan b  c (mod n) maka a  c (mod n)
• Jika a  b (mod n) dan c  d (mod n) maka a + c b + d (mod n)
• Jika a  b (mod n) dan c  d (mod n) maka ac  bd (mod n)
• Jika a  b (mod n) maka a + c  b + c (mod n)
• Jika a  b (mod n) maka ac  bc (mod n)
• Jika a  b (mod n) maka ak  bk(mod n) untuk k bilangan bulat positif
sebarang.

Contoh Soal 14
Jika 𝑎 ≡ 4 𝑚𝑜𝑑 13 dan 𝑏 ≡ 9 𝑚𝑜𝑑 13 buktikan bahwa 13|𝑎 + 𝑏

Pembahasan :
𝑎 ≡ 4 𝑚𝑜𝑑 13 dapat ditulis 13|𝑎 − 4
𝑏 ≡ 9 𝑚𝑜𝑑 13 dapat ditulis 13|𝑏 − 9
Maka :
Jika 13|𝑎 − 4 dan 13|𝑏 − 9, maka 13|𝑎 − 4 + 𝑏 − 9
Atau 13|𝑎 + 𝑏 − 13
Karena −13|13 maka 13|𝑎 + 𝑏

Contoh Soal 15
Jika 𝑝 ≡ 𝑞 𝑚𝑜𝑑 16 dan 𝑝 ≡ 𝑞 𝑚𝑜𝑑 24 maka nilai terkecil 𝑥 agar terdefinisi
pernyataan 𝑝 ≡ 𝑞 𝑚𝑜𝑑 𝑥 benar adalah ....

Pembahasan :
Berdasarkan teorema :

𝑝 ≡ 𝑞 𝑚𝑜𝑑 𝑛 dan 𝑝 ≡ 𝑞 𝑚𝑜𝑑 𝑚 jika dan hanya jika 𝑝 ≡ 𝑞 𝑚𝑜𝑑 (𝑛 , 𝑚)

Dengan (𝑛 , 𝑚) adalah KPK dari n dan m


Jika 𝑝 ≡ 𝑞 𝑚𝑜𝑑 16 dan 𝑝 ≡ 𝑞 𝑚𝑜𝑑 24 jika dan hanya jika 𝑝 ≡ 𝑞 𝑚𝑜𝑑 𝑥 , apabila 𝑥
adalah KPK dari 16 dan 24.
Jadi 𝑥 = 48

79 | T E O R I B I L A N G A N
Olimpiade Matematika SMA 2020

Contoh Soal 16
Jika 𝐴 = 321 + 52020 , maka 𝐴 dibagi 8 akan bersisa .....

Pembahasan :
Perhatikan bahwa 32 jika dibagi 8 akan bersisa 1, ditulis dalam bentuk :
32 ≡ 1 𝑚𝑜𝑑 8
(32 )10 . 3 ≡ 3 𝑚𝑜𝑑 8
(1)10 . 3 ≡ 3 𝑚𝑜𝑑 8
1. 3 ≡ 3 𝑚𝑜𝑑 8
3 ≡ 3 𝑚𝑜𝑑 8
Dengan demikian 321 jika dibagi 8 akan bersisa 3

Perhatikan 52 jika dibagi 8 akan bersisa 1, ditulis dalam bentuk :


52 ≡ 1 𝑚𝑜𝑑 8
(52 )1010 ≡ 1 𝑚𝑜𝑑 8
(1)1010 ≡ 1 𝑚𝑜𝑑 8
1 ≡ 1 𝑚𝑜𝑑 8
Dengan demikian 52020 jika dibagi 8 akan bersisa 1

Jadi 321 + 52020 ≡ 3 𝑚𝑜𝑑 8 + 1 𝑚𝑜𝑑 8

Jadi 321 + 52020 ≡ 4 𝑚𝑜𝑑 8

Sisa 𝐴 = 321 + 52020 dibagi 8 adalah 4

Contoh Soal 17
Tunjukkan bahwa 55552222 + 22225555 habis dibagi 7

Pembahasan :
55552222 + 22225555 = ( 7 × 793 + 4)2222 + ( 7 × 317 + 3 )5555

(( 7 × 793 + 4)𝟐𝟐𝟐𝟐 + ( 7 × 317 + 3 )5555 ) ≡ (4𝟐𝟐𝟐𝟐 + 35555 ) (𝑚𝑜𝑑 7)

(43 𝑥 740+2 + 33 𝑥 1851+2 ) 𝑚𝑜𝑑 7 ≡ ((43 )740 . 42 + (33 )1851 . 32 ) (𝑚𝑜𝑑 7)


≡ ((1)740 . 16 + (−1)1851 . 9) (𝑚𝑜𝑑 7)
≡ (16 − 9) (𝑚𝑜𝑑 7)
≡ 7 (𝑚𝑜𝑑 7)
≡ 0 (𝑚𝑜𝑑 7)

80 | T E O R I B I L A N G A N
Olimpiade Matematika SMA 2020

Contoh Soal 18
Tentukan 2 angka terakhir dari bilangan 31234
Pembahasan :
Dua angka terakhir 31234 = sisa pembagian 31234 oleh 100
31234 ( mod 100 ) ≡ 35 𝑥 206+4 (𝑚𝑜𝑑 100)
≡ (35 )206 . 34 (𝑚𝑜𝑑 100)
≡ (243)206 . 81 (𝑚𝑜𝑑 100)
≡ (43)2 𝑥 103 . 81 (𝑚𝑜𝑑 100)
≡ (432 )103 . 81 (𝑚𝑜𝑑 100)
≡ (1849)103 . 81 (𝑚𝑜𝑑 100)
≡ (49)103 . 81 (𝑚𝑜𝑑 100)
≡ (49)2 𝑥 51+1 . 81 (𝑚𝑜𝑑 100)
≡ (492 )51 . 49. 81 (𝑚𝑜𝑑 100)
≡ (2401)51 . 49. 81 (𝑚𝑜𝑑 100)
≡ (1)51 . 49. 81 (𝑚𝑜𝑑 100)
≡ 3969 (𝑚𝑜𝑑 100) ≡ 69 (𝑚𝑜𝑑 100)
Jadi, dua angka terakhir dari 31234 adalah 69

Contoh Soal 19
Sisa pembagian 11112019 oleh 11111 adalah ....
Pembahasan :
105 = 9 (11111) + 1 → 105  1 (𝑚𝑜𝑑 11111)
11112 = 111 (11111) + 1000 → 11112  1000 (𝑚𝑜𝑑 11111)
11110 = 11111 – 1 → 11110  − 1 (𝑚𝑜𝑑 11111)
(1111)2019  (103 )1009 . 1111 (𝑚𝑜𝑑 11111)
(1111)2019  (103 )5 . 201+4
. 1111 (𝑚𝑜𝑑 11111)
(1111)2019  (1015 ) 201 . 1012 . 1111 (𝑚𝑜𝑑 11111)
(1111)2019  (1015 ) 201 . 1010 102 . 1111 (𝑚𝑜𝑑 11111)
(1111)2019  (1) 201 . (1) 102 . 1111 (𝑚𝑜𝑑 11111)
(1111)2019  102 . 1111 (𝑚𝑜𝑑 11111)
(1111)2019  10 . 11110 (𝑚𝑜𝑑 11111)
(1111)2019  10 . (−1) (𝑚𝑜𝑑 11111)
(1111)2019  − 10 (𝑚𝑜𝑑 11111)
(1111)2019  11101 (𝑚𝑜𝑑 11111)
Jadi, sisa pembagian 11112019 oleh 11111 adalah 11101

81 | T E O R I B I L A N G A N
Olimpiade Matematika SMA 2020

Contoh Soal 20
Berapa banyak bilangan x ≤ 100 bulat positif sedemikian sehingga x  3 (mod 11).
Pembahasan :
x  3 (mod 11) jika dan hanya jika x – 3 = 11k untuk setiap k bilangan cacah.
Jika k = 0, 1, 2, 3, … maka berturut-turut x = 11k + 3
x ≤ 100
x = 3, 14, 25, 36, 47, 58, 69, 80 dan 91 (ada 9 nilai x yang memenuhi)

• PERSAMAAN DIOPHANTINE
Persamaan Diophantine terdiri dari persamaan linier dan non linier. Kita akan
membahas persamaan Diophantine linier terlebih dahulu.
Misalkan a, b, dan c adalah bilangan-bilangan bulat. Persamaan Diophantine
berbentuk 𝑎𝑥 + 𝑏𝑦 = 𝑐 disebut Persamaan Diophantine linear dan setiap
pasangan bilangan bulat (x, y) yang memenuhi 𝑎𝑥 + 𝑏𝑦 = 𝑐 disebut solusi.

Perhatikan teorema-teorema berikut :

• Persamaan Diophantine 𝑎𝑥 + 𝑏𝑦 = 𝑐 mempunyai solusi jika dan hanya jika


gcd (𝑎, 𝑏)|𝑐 .
• Jika Persamaan Diophantine 𝑎𝑥 + 𝑏𝑦 = 𝑐 mempunyai solusi (𝑥𝑜 , 𝑦𝑜 ) maka
persamaan tersebut mempunyai tak hinga banyaknya solusi.
• Dan setiap solusinya berbentuk (𝑥 , 𝑦)
𝑏 𝑎
𝑥 = 𝑥𝑜 + 𝑘 dan 𝑦 = 𝑦𝑜 − 𝑘
gcd(𝑎 ,𝑏) gcd(𝑎 ,𝑏)

atau jika gcd (a , b) = d


𝑏 𝑎
𝑥 = 𝑥𝑜 + 𝑘 dan 𝑦 = 𝑦𝑜 − 𝑘
𝑑 𝑑
untuk sebarang bilangan bulat k.

Untuk menentukan solusi awal (𝑥𝑜 , 𝑦𝑜 ) ini, kita harus mengingat kembali
Algoritma Euclide.
Kita bisa mencari bilangan bulat m dan n sehingga 𝑎𝑚 + 𝑏𝑛 = gcd(𝑎 , 𝑏). Karena
persamaan 𝑎𝑥 + 𝑏𝑦 = 𝑐 memiliki solusi jika dan hanya jika gcd (𝑎, 𝑏)|𝑐 maka
terdapat k sehingga 𝑐 = 𝑘 gcd(𝑎 , 𝑏).
Dengan demikian kita bisa mengambil solusi awal 𝑥𝑜 = km dan 𝑦𝑜 = kn

82 | T E O R I B I L A N G A N
Olimpiade Matematika SMA 2020

Contoh Soal 21
Tentukan semua solusi dari Persamaan Diophantine linear 18x + 12y = 108.

Pembahasan :
Kita punya gcd (18, 12) = 6 dan 6|108, yang berarti persamaan ini punya solusi.
Satu solusinya adalah (6, 0) . Dengan demikian solusi umumnya adalah

𝑏 𝑎
𝑥 = 𝑥𝑜 + 𝑘 dan 𝑦 = 𝑦𝑜 − 𝑘
gcd(𝑎 ,𝑏) gcd(𝑎 ,𝑏)

12 18
𝑥 = 6+𝑘 dan 𝑦 = 0 − 𝑘
6 6

𝑥 = 6 + 2𝑘 dan 𝑦 = −3𝑘
Solusi (x , y) = (6 + 2𝑘 , −3𝑘) untuk sebarang bilangan bulat k.

Contoh Soal 22
Buktikan bahwa ada banyak solusi bulat (𝑥 , 𝑦) persamaan 738 x + 621 y = 45
Pembahasan :
738 = 3 × 621 + 117
621 = 5 × 117 + 36
117 = 3 × 36 + 9
36 = 4 × 9 + 0
Jadi gcd ( 738, 621 ) = 9
9 = 117 – 3 × 36
9 = 117 – 3 × ( 621 – 5 × 117 )
9 = – 3 × 621 + 16 (738 – 621 )
9 = 16 x 738 – 19 x 621
Agar tampak ekuivalen dengan persamaan 738 x + 621 y = 45, maka kedua ruas
dikali dengan 5, sehingga menjadi 45 = 80 x 738 – 95 x 621

Didapat 𝑥𝑜 = 80 dan 𝑦𝑜 = – 95
Dengan demikian solusi umumnya adalah
𝑏 𝑎
𝑥 = 𝑥𝑜 + 𝑘 dan 𝑦 = 𝑦𝑜 − 𝑘
gcd(𝑎 ,𝑏) gcd(𝑎 ,𝑏)

621 738
𝑥 = 80 + 𝑘 dan 𝑦 = −95 − 𝑘
9 9

𝑥 = 80 + 69𝑘 dan 𝑦 = −95 − 82𝑘

Solusi (x , y) = (80 + 69𝑘 , −95 − 82𝑘) untuk sebarang bilangan bulat k.

83 | T E O R I B I L A N G A N
Olimpiade Matematika SMA 2020

Persamaan Diophantine non linier


Untuk persamaan Diophantine non linier pada bagian ini langsung pada contoh
soal dan pembahasan. Beberapa persamaan Diophantine non linier akan lebih
mudah diselesaikan dengan cara mendaftar bilangan atau aljabar biasa.

Contoh Soal 23
1 1 4
Tentukan nilai 𝑛 dan 𝑚 bulat positif sehingga + =
𝑛 𝑚 7

Pembahasan :
Dengan cara mendaftarkan bilangan bulat
Untuk n = 1, tidak memenuhi karena akan menghasilkan m negatif
Untuk n = 2,
1 1 4
+ =
2 𝑚 7

1 4 1 1
= − = , memenuhi untuk m = 14
𝑚 7 2 14

Untuk n > 2 tidak memenuhi karena akan menghasilkan m tidak bulat


Sehingga solusi (n , m) = (2 , 14)

Contoh Soal 24
Jika 𝑎 dan 𝑏 bilangan bulat positif 𝑎 ≠ 𝑏 sehingga 𝑎3 + 𝑏 3 = 𝑎4 , maka nilai
terkecil 𝑎𝑏 = ....

Pembahasan :
Kita akan mengubah bentuk 𝑎3 + 𝑏 3 = 𝑎4 menjadi persamaan pangkat yang
sederajat.
𝑏 3 = 𝑎4 − 𝑎3
𝑏 3 = 𝑎3 (𝑎 − 1) dengan (𝑎 − 1) adalah bilangan kubik
(𝑎 − 1) = 𝑘 3 , 𝑎 = 𝑘 3 + 1 dan 𝑏 3 = 𝑎3 (𝑎 − 1) , perhatikan tabel berikut :

𝑘 𝑎 𝑏 𝑎3 + 𝑏 3 = 𝑎4 Hasil
1 2 2 23 + 23 = 24 Tidak memenuhi karena 𝑎 = 𝑏
2 9 18 93 + 183 = 94 Memenuhi

Untuk k = 2, diperoleh 𝑎 = 9 dan 𝑏 = 18, sehingga 𝑎𝑏 = 9 . 18 = 162


Kita tidak perlu meneruskan untuk k = 3 , 4, 5 .... karena k = 2 sudah memenuhi

84 | T E O R I B I L A N G A N
Olimpiade Matematika SMA 2020

Contoh Soal 25
Tentukan solusi bulat dari persamaan 𝑥𝑦 = 2𝑥 − 𝑦

Pembahasan :
𝑥𝑦 = 2𝑥 − 𝑦
𝑥𝑦 − 2𝑥 + 𝑦 = 0
(𝑥 + 1)(𝑦 − 2) + 2 = 0
(𝑥 + 1)(𝑦 − 2) = −2
Karena 𝑥 dan 𝑦 bulat, maka (𝑥 + 1) dan (𝑦 − 2) juga bulat.
Karena faktor dari – 2 adalah – 2 , – 1 , 1 dan 2, maka ada 4 kemungkinan yang
terjadi, yaitu :
x + 1 = −2 dan y − 2 = 1. Dari sini diperoleh solusi x = −3, y = 3
x + 1 = −1 dan y − 2 = 2. Dari sini diperoleh solusi x = −2, y = 4
x + 1 = 1 dan y − 2 = −2. Dari sini diperoleh solusi x = 0, y = 0
x + 1 = 2 dan y − 2 = −1. Dari sini diperoleh solusi x = 1, y = 1
Jadi semua solusi (𝑥 , 𝑦) = (−3, 3) , (−2, 4) , (0, 0) dan (1, 1)

• FUNGSI TANGGA
Fungsi tangga, atau Fungsi Bilangan Bulat Terbesar, kalau ditulis mirip dengan
harga mutlak. Fungsi tangga itu sebenarnya mengenai pembulatan ke atas atau
ke bawah. Ada 3 definisi penting yang harus kita ketahui :

• Misalkan x adalah sebarang bilangan real. Nilai fungsi floor x dinotasikan


dengan ⌊𝑥⌋ merupakan bilangan bulat terbesar yang kurang dari atau sama
dengan x.

• Untuk sebarang bilangan real x , Nilai 𝑥 − ⌊𝑥⌋, dinotasikan dengan {𝑥} dan
disebut dengan bagian desimal dari x.

• Misalkan x adalah sebarang bilangan real. Nilai fungsi ceiling dinotasikan


dengan ⌈𝑥⌉ merupakan bilangan bulat terkecil yang lebih dari atau sama
dengan x .
Contohnya :
⌊198,4⌋ = 198 , faktor desimalnya {𝑥} = 0,4 dan ⌈198,4⌉ = 199
⌊−12,7⌋ = −13 , faktor desimalnya {𝑥} = 0,3 dan ⌈−12,7⌉ = −12

85 | T E O R I B I L A N G A N
Olimpiade Matematika SMA 2020

Beberapa sifat sebagai berikut:


• Untuk sebarang bilangan real x selalu berlaku 𝑥 − 1 < ⌊𝑥⌋ ≤ 𝑥
• ⌊𝑥⌋ = 𝑥 jika dan hanya jika 𝑥 bilangan asli
• ⌊𝑥 + 𝑘⌋ = ⌊𝑥⌋ + 𝑘 untuk sebarang bilangan bulat k
• ⌊𝑥⌋ + ⌊𝑦⌋ ≤ ⌊𝑥 + 𝑦⌋ untuk setiap (𝑥 , 𝑦) bilangan real
• ⌊𝑥𝑦⌋ ≤ ⌊𝑥⌋ ⌊𝑦⌋ untuk setiap (𝑥 , 𝑦) bilangan real

Fungsi ceiling disebut juga fungsi pembulatan ke atas. Untuk sebarang bilangan
real x, nilai fungsi ceiling dari x kita tulis dengan ⌈𝑥⌉.
Misalkan x adalah sebarang bilangan real. Nilai fungsi bulat x kita tulis dengan
1
[x] merupakan bilangan bulat terdekat dengan x. Jika 𝑥 = 𝑘 + 2 untuk suatu
bilangan bulat k, maka kita definisikan [𝑥] = k + 1.
Beberapa sifat sebagai berikut:
• ⌊𝑥⌋ ≤ [𝑥] ≤ ⌈𝑥⌉ untuk setiap bilangan real x
• ⌈𝑥⌉ = 𝑥 jika dan hanya jika [𝑥] = 𝑥
• ⌈𝑥 + 𝑘⌉ = ⌈𝑥⌉ + 𝑘 dan [𝑥 + 𝑘] = [𝑥] + 𝑘 untuk sebarang bilangan bulat k
• ⌈𝑥⌉ + ⌈𝑦⌉ ≤ ⌈𝑥 + 𝑦⌉ untuk setiap (𝑥 , 𝑦) bilangan real
• ⌈𝑥𝑦⌉ ≤ ⌈𝑥⌉ ⌈𝑦⌉ untuk setiap (𝑥 , 𝑦) bilangan real

Contoh Soal 26
5+6𝑥 15𝑥−7
Tentukan himpunan penyelesaian dari ⌊ ⌋=
8 5

Pembahasan :
5+6𝑥
Misal ⌊ ⌋ = m, maka 15x – 7 = 5m
8
7
Jika m = 0, maka 𝑥 =
15
7 7
5+6(15) 15(15)−7
• ⌊ ⌋=
8 5

39 39
⌊ ⌋ = 0 , memenuhi karena 0 bilangan bulat terbesar yang kurang
40 40

12
Jika m = 1, maka 𝑥 =
15

86 | T E O R I B I L A N G A N
Olimpiade Matematika SMA 2020
12 12
5+6(15) 15(15)−7
• ⌊ ⌋=
8 5

49 49
⌊ ⌋ = 1 , memenuhi karena 1 bilangan bulat terbesar yang kurang
40 40

17
Jika m = 1, maka 𝑥 =
15
17 17
5+6(15) 15(15)−7
• ⌊ ⌋=
8 5

57 57
⌊ ⌋ = 2 , tidak memenuhi, 2 bukan bulat terbesar kurang dari
40 40
7 12
Jadi himpunan penyelesaiannya adalah x = {15 , 15}

Contoh Soal 27
Ada berapa angka Nol dibelakang dan tidak terputus dari 46 ! ?

Pembahasan :
Angka nol merupakan hasil perkalian 2 dan 5 yang saling prima terhadap 10.
Misalkan 𝑛! = 𝑘 . 2𝑎 5𝑝 karena 𝑎 > 𝑝 maka untuk mengetahui banyaknya
angka nol, kita cukup menghitung 𝑝 saja.
𝑛
Setiap angka ke-5, habis dibagi 5, maka 𝑝1 =
5
𝑛
Setiap angka ke-25, habis dibagi 25, maka 𝑝2 =
25
𝑛
Setiap angka ke-125, habis dibagi 125, maka 𝑝3 = dan seterusnya.
125

Sehingga banyaknya angka nol tanpa terputus dibagian belakang pada 𝑛! adalah

𝑝 = ⌊𝑝1 ⌋ + ⌊𝑝2 ⌋ + ⌊𝑝3 ⌋ + ⋯

46 46
𝑝=⌊ ⌋ + ⌊ ⌋ = 9 + 1 = 10
5 25
Dengan bantuan kalkulator digital, terbukti ada 10 angka nol di belakang.
46 ! = 5502622159812088949850305428800254892961651752960000000000

87 | T E O R I B I L A N G A N
Olimpiade Matematika SMA 2020

Contoh Soal 28
Jika ⌊𝑥 2 ⌋ + ⌈𝑥 2 ⌉ = 2020, tentukan himpunan penyelesaian harga x yang
memenuhi.

Pembahasan :

Karena ⌊𝑥 2 ⌋ + ⌈𝑥 2 ⌉ = 2020 dan ⌊𝑥 2 ⌋  ⌈𝑥 2 ⌉ maka kita bisa mengasumsikan bahwa

⌊𝑥 2 ⌋ = 2019 , sehingga 𝑥 2 = 2019

⌈𝑥 2 ⌉ = 2021, sehingga 𝑥 2 = 2021

Maka untuk memenuhi ⌊𝑥 2 ⌋ + ⌈𝑥 2 ⌉ = 2020 , nilai 𝑥 2 = {2019 , 2021}

Olehkarena itu himpunan penyelesaian untuk x adalah


𝑥 = {−√2021 , −√2019 , √2019 , √2021 }

Contoh Soal 29
Untuk setiap bilangan real z, ⌊z⌋ menyatakan bilangan bulat terbesar yang
lebih kecil dari atau sama dengan z. Jika diketahui ⌊x⌋ + ⌊y⌋ + y = 43,8 dan
x + y − ⌊x⌋= 18,4. Nilai 10(x + y) adalah ...

Pembahasan :
Misalkan {x} = x − ⌊𝑥⌋.
Dari persamaan ⌊𝑥⌋+ ⌊𝑦⌋ + y = 43,8 diperoleh ⌊𝑥⌋ + 2 ⌊𝑦⌋+ {y} = 43,8

akibatnya, {y} = 0,8. Jadi,


⌊𝑥⌋ + 2 ⌊𝑦⌋ = 43

Sedangkan persamaan x + y − ⌊𝑥⌋ = 18,4 setara dengan

{x} + ⌊𝑦⌋ + {y} = 18,4 ⇔ {x} + ⌊𝑦⌋ = 18,4 – 0,8 = 17,6

Oleh karena itu, ⌊𝑦⌋ = 17 dan {x} = 0,6.

Jika nilai ⌊𝑦⌋ = 17 disubstitusikan ke persamaan ⌊𝑥⌋ + 2 ⌊𝑦⌋+ {y} = 43,8

⌊𝑥⌋ + 2 ⌊𝑦⌋+ {y} = 43,8 ⇔ ⌊𝑥⌋ + 2 . 17+ 0,8 = 43,8 ⇔ ⌊𝑥⌋= 9

Jadi, x = ⌊𝑥⌋+{x} = 9,6 dan y = ⌊𝑦⌋+{ y} = 17,8.

Oleh karena itu, 10(x + y) = 10 (9,6 + 17,8) = 274.

88 | T E O R I B I L A N G A N
Olimpiade Matematika SMA 2020

Problem 30
Untuk sebarang bilangan real x, simbol ⌊𝑥⌋ menyatakan bilangan bulat terbesar
yang tidak lebih besar daripada x, dan simbol ⌈𝑥⌉ menyatakan bilangan bulat
terkecil yang tidak lebih kecil daripada x, Interval [𝑎 , 𝑏) adalah himpunan semua
bilangan real x yang memenuhi
⌊2𝑥⌋2 = ⌈𝑥⌉ + 7
Nilai 𝑎 . 𝑏 adalah ....

Pembahasan :
Untuk x  Bulat, ⌊2𝑥⌋ = 2𝑥 dan ⌈𝑥⌉ = 𝑥
⌊2𝑥⌋2 = ⌈𝑥⌉ + 7
(2𝑥)2 = 𝑥 + 7
4𝑥 2 − 𝑥 − 7 = 0 , D = (–1)2 – 4 . 4 (–7) = 113, (tidak memenuhi)
1
Untuk x tidak bulat, maka ⌊2𝑥⌋ = 2⌊𝑥⌋ dan ⌈𝑥⌉ = ⌊𝑥⌋ + 1, ambil 0 < {𝑥} < 2

⌊2𝑥⌋2 = ⌈𝑥⌉ + 7
4⌊𝑥⌋2 = (⌊𝑥⌋ + 1) + 7
4⌊𝑥⌋2 = ⌊𝑥⌋ + 8
4⌊𝑥⌋2 − ⌊𝑥⌋ − 8 = 0 , D = (–1)2 – 4 . 4 (–8) = 123, tidak memenuhi)
1
Untuk x tidak bulat, maka ⌊2𝑥⌋ = 2⌊𝑥⌋ + 1 dan ⌈𝑥⌉ = ⌊𝑥⌋ + 1, ambil ≤ {𝑥} < 1
2
⌊2𝑥⌋2 = ⌈𝑥⌉ + 7
(2⌊𝑥⌋ + 1)2 = (⌊𝑥⌋ + 1) + 7
4⌊𝑥⌋2 + 4⌊𝑥⌋ + 1 = ⌊𝑥⌋ + 8
4⌊𝑥⌋2 + 3⌊𝑥⌋ − 7 = 0 , D = (3)2 – 4 . 4 (–7) = 121 (bilangan kuadrat), maka x bulat.

−𝑏 + √𝐷 −3 + √121
⌊𝑥⌋ = = =1
2𝑎 8
1
𝑥 = ⌊𝑥⌋ +
2

himpunan semua bilangan real x yang memenuhi adalah [𝑎 , 𝑏), maka 𝑎 ≤ 𝑥 < 𝑏
1 1
dipenuhi oleh interfal 1 2 ≤ 𝑥 < 2. Sehingga 𝑎𝑏 = 1 2 . 2 = 3

89 | T E O R I B I L A N G A N
Olimpiade Matematika SMA 2020

Problem Solving
TEORI BILANGAN

Problem 01
Jumlah semua bilangan :
9 – 7 + 5 – 8 + 6 – 3 + 2 – 4 + 9 – 7 + 5 – 8 + 6 – 3 + 2 – 4 + 9 – 7 + 5 – 8 + 6 – 3 + ......
sampai dengan bilangan ke-2020 adalah .....

Pembahasan :
Pola bilangan tersebut terdapat perulangan setiap 8 bilangan, yaitu :
9 – 7 + 5 – 8 + 6 – 3 + 2 – 4 , hasil penjumlahan 9 – 7 + 5 – 8 + 6 – 3 + 2 – 4 = 0
2020 jika dibagi 8 akan menghasilkan 252 dan sisa 4, maka bilangan yang jatuh
pada urutan ke 2020 adalah bilangan urutan keempat, yaitu – 8.
Jumlah 2020 bilangan = 252 (9 – 7 + 5 – 8 + 6 – 3 + 2 – 4) + (9 – 7 + 5 – 8)
= 252 ( 0 ) + (– 5)
=–5

Problem 02
2020 bilangan asli pertama secara berurutan ditempatkan dalam tabel berikut :

Misalkan bilangan 2020 berada pada baris b dan kolom k, maka b + k = .....

Pembahasan :
Setiap baris terdapat 9 bilangan, baris ganjil urutan nomor bertambah ke kanan
dan baris genap, urutan nomor bertambah ke kiri.
2020 jika dibagi sembilan akan menghasilkan 224 bersisa 4.
Jadi 2020 akan terlihat di baris ke 225 (ganjil) dan pada kolom ke empat dari kiri,
yaitu kolom 4. Sehingga b = 225 dan k = 4
Nilai b + k = 225 + 4 = 229

90 | T E O R I B I L A N G A N
Olimpiade Matematika SMA 2020

Problem 03
Buktikan bahwa A = 2903𝑛 − 803𝑛 − 464𝑛 + 261𝑛 habis dibagi 1897.

Pembahasan :
1897 = 7 x 271
Jika A habis dibagi 1897, berarti A habis dibagi 7 dan 271

A = 2903𝑛 − 803𝑛 − 464𝑛 + 261𝑛

Pembuktian A habis dibagi 7


A = 2903𝑛 − 803𝑛 − (464𝑛 − 261𝑛 )

2903𝑛 − 803𝑛 = (2903 − 803)(… . #@$%& … )


2903𝑛 − 803𝑛 = 2100 (… . #@$%& … ) , habis dibagi 7
464𝑛 − 261𝑛 = (464 − 261)(… . #@$%& … )
2903𝑛 − 803𝑛 = 203 (… . #@$%& … ) , habis dibagi 7
Maka :
A = 2903𝑛 − 803𝑛 − (464𝑛 − 261𝑛 ) habis dibagi 7

Pembuktian A habis dibagi 271


A = 2903𝑛 − 464𝑛 − (803𝑛 − 261𝑛 )

2903𝑛 − 464𝑛 = (2903 − 464)(… . #@$%& … )


2903𝑛 − 464𝑛 = 2439 (… . #@$%& … ) , habis dibagi 2 71
803𝑛 − 261𝑛 = (803 − 261)(… . #@$%& … )
2903𝑛 − 803𝑛 = 542 (… . #@$%& … ) , habis dibagi 271
Maka :
A = 2903𝑛 − 464𝑛 − (803𝑛 − 261𝑛 ) habis dibagi 271

Jadi A = 2903𝑛 − 803𝑛 − 464𝑛 + 261𝑛 terbukti habis dibagi 7 dan 271

Problem 04
Berapa digit terakhir dari 3 7 ?

Pembahasan :
Untuk menentukan satu digit terakhir, kita gunakan mudulo 10, karena sisa dari
pembagian terhadap 10 pasti digit terakhir suatu bilangan asli
37 = (33 )2 . 3  (7)2 . 3 (mod 10)  9. 3 (mod 10)  27 (mod 10)  7 (mod 10)
Jadi digit terakhir dari 37 adalah 7

91 | T E O R I B I L A N G A N
Olimpiade Matematika SMA 2020

Problem 05
Jika √𝑛 − √𝑚 = 12, tunjukan bahwa nilai maksimum dari 𝑛 − 3𝑚 merupakan
bilangan kubik.

Pembahasan :

√𝑛 − √𝑚 = 12
𝑛 = 𝑚 + 24√𝑚 + 144
𝑛 − 3𝑚 = 𝑚 + 24√𝑚 + 144 − 3𝑚
𝑛 − 3𝑚 = −2𝑚 + 24√𝑚 + 144
2
𝑛 − 3𝑚 = −2(√𝑚 − 6) + 72 + 144
2
𝑛 − 3𝑚 = −2(√𝑚 − 6) + 216
Maka nilai maksimum 𝑛 − 3𝑚 adalah 216 (merupakan bilangan kubik, 63 )

Problem 06
Banyaknya bilangan asli n sehingga
𝑛4 − 5𝑛3 + 5𝑛2 + 4𝑛 + 10 merupakan bilangan prima adalah ...
Pembahasan :
𝑛4 − 5𝑛3 + 5𝑛2 + 4𝑛 + 10 = 𝑛4 + 2𝑛3 + 3𝑛2 + 2𝑛 + 1 − 7𝑛3 + 2𝑛2 + 2𝑛 + 9
= (𝑛2 + 𝑛 + 1)2 − 7𝑛3 + 2𝑛2 + 2𝑛 + 9
= (𝑛2 + 𝑛 + 1)2 − 7𝑛3 − 7𝑛2 − 7𝑛 + 9𝑛2 + 9𝑛 + 9
= (𝑛2 + 𝑛 + 1)2 − 7𝑛(𝑛2 + 𝑛 + 1) + 9(𝑛2 + 𝑛 + 1)
= (𝑛2 + 𝑛 + 1) (𝑛2 + 𝑛 + 1 − 7𝑛 + 9)
= (𝑛2 + 𝑛 + 1) (𝑛2 − 6𝑛 + 10)

Karena n bilangan asli maka 𝑛2 + 𝑛 + 1 ≥ 3

Agar (𝑛2 + 𝑛 + 1) (𝑛2 − 6𝑛 + 10) merupakan bilangan prima maka :


n2 − 6n + 10 = 1
n2 − 6n + 9 = 0
(𝑛 − 3)2 = 0 dan n = 3

Untuk menguji n = 3 ke persamaan 𝑛2 + 𝑛 + 1 dan 𝑛4 − 5𝑛3 + 5𝑛2 + 4𝑛 + 10


𝑛2 + 𝑛 + 1 = (3)2 + 3 + 1 = 13
𝑛4 − 5𝑛3 + 5𝑛2 + 4𝑛 + 10 = (3)4 – 5(3)3 + 5(3)2 + 4(3) + 10 = 13
jadi banyaknya bilangan asli n adalah 1

92 | T E O R I B I L A N G A N
Olimpiade Matematika SMA 2020

Problem 07
Berapa sisanya jika 72020 + 32019 dibagi 8 ?
Pembahasan :
Perhatikan 72 dibagi 8 sisanya 1 dan 34 dibagi 8 sisanya 1.
Perhatikan penggunaan tanda sama dengan (=) dan tanda kongruen (≡) pada
proses solusi di bawah ini :

72020 + 32019 = (72 )1010 + (3)4 .504+ 3 = (72 )1010 + (34 ) 504 . 33

(72 )1010 + (34 ) 504 . 33  ((1)1010 + (1) 504 . 27 ) (mod 8)  (1 + 1. 27 ) (mod 8)


(1 + 1. 27 ) (mod 8)  28 (mod 8)  4 (mod 8)
Jadi 72020 + 32019 dibagi 8 akan bersisa 4

Problem 08
Buktikan bahwa jika 9|𝑎3 + 𝑏 3 + 𝑐 3 maka 3|𝑎𝑏𝑐 untuk bilangan bulat a, b, c !

Pembahasan :
Diketahui 9|𝑎3 + 𝑏 3 + 𝑐 3 berarti 3|𝑎3 + 𝑏 3 + 𝑐 3 .

Selain itu dari aljabar


(𝑎 + 𝑏 + 𝑐)3 = 𝑎3 + 𝑏 3 + 𝑐 3 + 3𝑎𝑏 2 + 3𝑎𝑐 2 + 3𝑏𝑐 2 + 3𝑎2 𝑏 + 3𝑎2 𝑐 + 3𝑏 2 𝑐 + 6𝑎𝑏𝑐
(𝑎 + 𝑏 + 𝑐)3 = 𝑎3 + 𝑏 3 + 𝑐 3 + 3 (𝑎𝑏 2 + 𝑎𝑐 2 + 𝑏𝑐 2 + 𝑎2 𝑏 + 𝑎2 𝑐 + 𝑏 2 𝑐 + 2𝑎𝑏𝑐)

maka dari itu,


3|𝑎3 + 𝑏 3 + 𝑐 3 dapat ditulis menjadi
3|(𝑎 + 𝑏 + 𝑐)3 −3 (𝑎𝑏 2 + 𝑎𝑐 2 + 𝑏𝑐 2 + 𝑎2 𝑏 + 𝑎2 𝑐 + 𝑏 2 𝑐 + 2𝑎𝑏𝑐)
akibatnya :
3 (𝑎𝑏 2 + 𝑎𝑐 2 + 𝑏𝑐 2 + 𝑎2 𝑏 + 𝑎2 𝑐 + 𝑏 2 𝑐 + 2𝑎𝑏𝑐))  0 mod 3.
3|(𝑎 + 𝑏 + 𝑐)3  0 mod 3.
Sehingga dapat ditulis 3|(𝑎 + 𝑏 + 𝑐)3 = 3|(𝑎 + 𝑏 + 𝑐)(𝑎 + 𝑏 + 𝑐) (𝑎 + 𝑏 + 𝑐).
Suatu bilangan habis dibagi n maka ada salah satu faktornya juga habis dibagi n.

Karena semua faktor sama maka dapat ditulis


3|(𝑎 + 𝑏 + 𝑐)3  3|(𝑎 + 𝑏 + 𝑐)
Sesuai dengan sifat bilangan yang habis dibagi 3 adalah bilangan yang jumlah
digit-digitnya habis dibagi 3.
Sehingga 3|(𝑎 + 𝑏 + 𝑐)  3|𝑎𝑏𝑐

93 | T E O R I B I L A N G A N
Olimpiade Matematika SMA 2020

Problem 09
Hanya ada satu tripel (a, b, c) ∈ Bulat positif dengan b  1, yang memenuhi
𝑎𝑏 + 𝑏𝑐 = 51 dan 𝑎𝑐 + 𝑏𝑐 = 19. Nilai dari 𝑎 + 𝑏 2 + 𝑐 2020 adalah ....

Pembahasan :
𝑎𝑏 + 𝑏𝑐 = 51
𝑏 (𝑎 + 𝑐) = 51
51 merupakan 3 dan 17. Tetapi kita akan memakai 51 = 3 . 17, karena b  1
𝑏 (𝑎 + 𝑐) = 3 . 17 , terjadi jika b = 3 dan 𝑎 + 𝑐 = 17 dan tidak terjadi sebaliknya

𝑎𝑐 + 𝑏𝑐 = 19
𝑐(𝑎 + 𝑏) = 19 , karena 19 adalah perkalian 1 dan 19 maka 𝑐(𝑎 + 𝑏) = 19,
terjadi jika c = 1 dan 𝑎 + 𝑏 = 19 dan tidak terjadi sebaliknya

Jika b = 3 dan c = 1, maka nilai a yang memenuhi adalah a = 16.


Jadi (a, b, c) = (16, 3, 1) merupakan satu-satunya tripel (a, b, c) yang memenuhi

Nilai 𝑎 + 𝑏 2 + 𝑐 2020 = 16 + (3)2 + (1)2020 = 26

Problem 10
Untuk setiap bilangan asli 𝑛 didefinisikan (𝑛) sebagai hasil penjumlahan dari
semua digit-digit dari 𝑛. Banyaknya bilangan asli 𝑑 sehingga 𝑑 habis membagi
𝑛 − (𝑛) untuk setiap bilangan asli 𝑛 adalah ….

Pembahasan :
Kita akan selesaikan soal ini dengan beberapa eksperimen sederhana, berikut ini
Untuk sembarang bilangan asli
̅̅̅ = 10a + b , maka (n) = a + b, maka n – (n) = 9a
n = 𝑎𝑏
n = ̅̅̅̅̅
𝑎𝑏𝑐 = 100a +10b + c , (n) = a + b + c, maka n – (n) = 99a + 9b = 9 (11a + b)

Kita temulan bahwa , n – (n) selalu memiliki faktor 9.


Maka bilangan asli d merupakan faktor dari 9, yaitu {1 , 3, 9}
Sehingga terdapat 3 bilangan asli d yang memenuhi

Sebetulnya, secara sederhana kita bisa mengambil contoh bilangan berapa saja
dan bilangan itu dikurangi dengan jumlah digit bilangannya, pasti menghasilkan
bilangan yang habis dibagi 9.

94 | T E O R I B I L A N G A N
Olimpiade Matematika SMA 2020

Problem 11
Buktikan bahwa ⏟ 222 … 222 merupakan bilangan kuadrat sempurna !
111 … 111 − ⏟
2020 𝑘𝑎𝑙𝑖 1010 𝑘𝑎𝑙𝑖
Pembahasan :
⏟ 111 … 111 x 101010 + ⏟
111 … 111 = ⏟ 111 … 111
2020 𝑘𝑎𝑙𝑖 1010 𝑘𝑎𝑙𝑖 1010 𝑘𝑎𝑙𝑖

111 … 111 − ⏟
⏟ 111 … 111 x 101010 + ⏟
222 … 222 = ⏟ 111 … 111 − ⏟
222 … 222
2020 𝑘𝑎𝑙𝑖 1010 𝑘𝑎𝑙𝑖 1010 𝑘𝑎𝑙𝑖 1010 𝑘𝑎𝑙𝑖 1010 𝑘𝑎𝑙𝑖

111 … 111 − ⏟
⏟ 111 … 111 x 101010 − ⏟
222 … 222 = ⏟ 111 … 111
2020 𝑘𝑎𝑙𝑖 1010 𝑘𝑎𝑙𝑖 1010 𝑘𝑎𝑙𝑖 1010 𝑘𝑎𝑙𝑖

111 … 111 − ⏟
⏟ 111 … 111 (101010 − 1)
222 … 222 = ⏟
2020 𝑘𝑎𝑙𝑖 1010 𝑘𝑎𝑙𝑖 1010 𝑘𝑎𝑙𝑖

111 … 111 − ⏟
⏟ 222 … 222 = ⏟
111 … 111 ⏟
999 … 999
2020 𝑘𝑎𝑙𝑖 1010 𝑘𝑎𝑙𝑖 1010 𝑘𝑎𝑙𝑖 1010 𝑘𝑎𝑙𝑖

111 … 111 − ⏟
⏟ 111 … 111 . 9 . ⏟
222 … 222 = ⏟ 111 … 111
2020 𝑘𝑎𝑙𝑖 1010 𝑘𝑎𝑙𝑖 1010 𝑘𝑎𝑙𝑖 1010 𝑘𝑎𝑙𝑖

2
111 … 111 − ⏟
⏟ 222 … 222 = (111
⏟ … 111) (3)2
2020 𝑘𝑎𝑙𝑖 1010 𝑘𝑎𝑙𝑖 1010 𝑘𝑎𝑙𝑖

2
111 … 111 − ⏟
⏟ 222 … 222 = (333
⏟ … 333)
2020 𝑘𝑎𝑙𝑖 1010 𝑘𝑎𝑙𝑖 1010 𝑘𝑎𝑙𝑖
2
⏟ … 333) merupakan bilangan kuadrat sempurna (Terbukti)
(333
1010 𝑘𝑎𝑙𝑖

Problem 12
Tentukan bilangan Asli 𝑥 jika diketahui 10𝑥 ≡ 6 𝑚𝑜𝑑 14 !
Pembahasan :
10𝑥 ≡ 6 𝑚𝑜𝑑 14 dapat ditulis menjadi 10𝑥 = 14𝑘 + 6 atau 5𝑥 = 7𝑘 + 3
5𝑥 = 7𝑘 + 3 berarti 5𝑥 ≡ 3 𝑚𝑜𝑑 7

5𝑥 ≡ 3 𝑚𝑜𝑑 7
5𝑥 ≡ 10 𝑚𝑜𝑑 7
𝑥 ≡ 2 𝑚𝑜𝑑 7 , dapat ditulis menjadi 𝑥 = 7𝑘 + 2 dengan 𝑘 bilangan cacah
Maka 𝑥 = 2 , 9 , 16 , …. Masing-masing untuk 𝑘 = 0 , 1, 2, … ..

95 | T E O R I B I L A N G A N
Olimpiade Matematika SMA 2020

Problem 13
Berapa banyaknya digit pada bilangan 1520 ?

Pembahasan :
Untuk menentukan banyaknya digit bilangan, selain dengan operasi bilangan
basis 10, kita juga bisa menggunakan bantuan logaritma

log 2 = 0,301
log 3 = 0,477
log 5 = 0,699
log 7 = 0,845

Banyaknya digit bilangan 1520 = 20 log 15


log 15 = log 3 + log 5 = 0,477 + 0,699 = 1,176
Banyaknya digit bilangan 1520 adalah 20 x 1,176 = 23,52  24
(Pembulatan selalu dilakukan ke atas)

Dengan menggunakan kalkulator digital, ternyata 1520 memang memiliki 24 digit


bilangan, 1520 = 332.525.673.007.965.087.890.625.

Problem 14
Diberikan dua bilangan asli dua angka yang selisihnya 10. Diketahui bahwa
bilangan yang kecil merupakan kelipatan 3, sedangkan yang lainnya
merupakan kelipatan 7. Diketahui pula bahwa jumlah semua faktor prima
kedua bilangan tersebut adalah 17. Jumlah dua bilangan tersebut adalah ...

Pembahasan :
Misalkan dua bilangan tersebut adalah 𝑎 dan 𝑏, dengan b = 𝑎 + 10.
Jika 𝑎 kelipatan 3 dan b kelipatan 7, maka nilai terkecil 𝑎 adalah 18 dan
b adalah 28.
Selebihnya, hubungan 𝑎 dan b disajikan pada tabel di bawah ini :

𝑎 𝑏 Faktor Prima a dan b Jumlah Faktor Prima


18 28 2,3,7 12
39 49 3 , 7 , 13 23
60 70 2,3,5,7 17
81 91 3 , 7 , 13 23

Jadi yang memenuhi adalah 𝑎 = 60 dan 𝑏 = 70, sehingga 𝑎 + 𝑏 = 130

96 | T E O R I B I L A N G A N
Olimpiade Matematika SMA 2020

Problem 15
Misalkan 𝑎, 𝑏 dan 𝑐 adalah bilangan ganjil berurutan dengan 𝑎 < 𝑏 < 𝑐
sedemikian sehingga 𝑎2 + 𝑏 2 + 𝑐 2 merupakan bilangan dengan 4 digit yang
sama. tentukan tripel (𝑎, 𝑏, 𝑐) yang memenuhi !
Pembahasan :
Jumlah kuadrat tiga bilangan berurutan haruslah bersisa 3 jika dibagi 8
atau 𝑎2 + 𝑏 2 + 𝑐 2 ≡ 3 𝑚𝑜𝑑 8

Jika 𝑎 = 𝑛 − 2 , 𝑏 = 𝑛 dan 𝑐 = 𝑛 + 2 , maka dapat ditunjukan sebagai berikut :


𝑎2 + 𝑏 2 + 𝑐 2 = (𝑛 − 2)2 𝑛 (𝑛 + 2)2
𝑎2 + 𝑏 2 + 𝑐 2 = 3𝑛2 + 8

Jika 𝑎2 + 𝑏 2 + 𝑐 2 merupakan bilangan 4 digit yang semua digitnya sama maka


bilangan itu ada diantara 1111, 2222, 3333, 4444, 5555, 6666, 7777, 8888 dan 9999.
Ternyata hanya 5555 yang dapat bersisa 3 jika dibagi 8, atau 5555 ≡ 3 𝑚𝑜𝑑 8
Dengan demikian :
3𝑛2 + 8 = 5555 = 3 𝑚𝑜𝑑 8
3𝑛2 + 8 = 5555
3𝑛2 = 5547
𝑛2 = (43)2 = (−43)2
Jadi 𝑛 = 43 atau 𝑛 = −43
Maka (𝑎, 𝑏, 𝑐) adalah (41, 43, 45) dan (−45, −43, −41)

Problem 16
1 1 1
Banyaknya pasangan bilangan bulat positif (𝑎 , 𝑏) sehingga + = adalah ....
𝑎 𝑏 6

Pembahasan :
1 1 1
+ =
𝑎 𝑏 6
𝑎+𝑏 1
=
𝑎𝑏 6
𝑎𝑏 − 6𝑎 − 6𝑏 = 0
𝑎𝑏 − 6𝑎 − 6𝑏 + 36 = 36
𝑎(𝑏 − 6) − 6(𝑏 − 6) = 36
(𝑏 − 6)(𝑎 − 6) = 36
(𝑏 − 6) dan (𝑎 − 6) adalah faktor dari 36
36 = 22 𝑥 32
Banyaknya pasangan (𝑎 , 𝑏)= (2 +1) (2 + 1) = 9

97 | T E O R I B I L A N G A N
Olimpiade Matematika SMA 2020

Problem 17
Buktikan bahwa ada banyak solusi bulat (𝑥 , 𝑦) persamaan 754 x + 221 y = 13

Pembahasan :
Soal pembuktian diatas merupakan persamaan Diophantine.
Suatu persamaan berbentuk ax + by = c dengan a, b, c bilangan-bilangan bulat
dan a, b dua-duanya bukan nol disebut persamaan linear Diophantine

Teorena Diophantine :
Jika d = gcd(a, b) dan (𝑥𝑜 , 𝑦𝑜 ) penyelesaian persamaan Diophantine ax + by = c,
maka penyelesaian umum persamaan tersebut adalah :
𝑏 𝑎
𝑥 = 𝑥𝑜 + ( ) 𝑘 dan 𝑦 = 𝑦𝑜 − ( ) 𝑘 dengan k parameter bilangan bulat.
𝑑 𝑑

Tahap Penyelesaian 754 x + 221 y = 13


Kita harus menentukan terlebih dahulu FPB dari 754 dan 221, atau gcd(754 , 221)

754 = 3 × 221 + 91
221 = 2 × 91 + 39
91 = 2 × 39 + 13 = 3 × 13 + 0
Jadi d = gcd( 754, 221 ) = 13

Sehingga persamaan di atas mempunyai penyelesaian bulat.


13 = 91 – 2 × 39
13 = 91 – 2 × ( 221 – 2 × 91 )
13 = – 2 × 221 + 5 × 991
13 = – 2 × 221 +5(754 – 3 × 221 )
13 = 5 × 754 – 17 × 221

Didapat 𝑥𝑜 = 5 dan 𝑦𝑜 = – 17

Penyelesaian umum 754 x + 221 y = 13, dengan d = gcd(a, b) adalah


𝑏 221
𝑥 = 𝑥𝑜 + ( ) 𝑘 = 5 + ( ) 𝑘 = 5 + 17𝑘
𝑑 13
𝑎 754
𝑦 = 𝑦𝑜 − ( ) 𝑘 = −17 − ( ) 𝑘 = −17 − 58𝑘
𝑑 13

Jadi ada banyak solusi bulat (𝑥 , 𝑦) atau (5 + 17𝑘 , −17 − 58𝑘 ) untuk 𝑘 bulat

98 | T E O R I B I L A N G A N
Olimpiade Matematika SMA 2020

Problem 18
Untuk x, y bilangan Bulat positif, tunjukan bahwa pada sistem persamaan :
𝑚𝑥 + 2𝑦 = 10
}
3𝑥 − 2𝑦 = 0
Tunjukan bahwa m adalah bilangan prima !

Pembahasan :
Pada sistem linier berikut :
𝑚𝑥 + 2𝑦 = 10 , karena 2𝑦 = 3𝑥 maka :
𝑚𝑥 + 3𝑥 = 10
(𝑚 + 3)𝑥 = 10
10 3 15
𝑥= dan 𝑦 = 𝑥 =
𝑚+3 2 𝑚+3
Untuk x, y bilangan Bulat positif, 𝑚 + 3|10 dan 𝑚 + 3|15 sehingga 𝑚 + 3|5.
𝑚 + 3|5 maka 𝑚 = 2 (bilangan prima)

Problem 19
Jika n adalah faktror prima terbesar dari 2020, maka banyaknya angka nol di
belakang tanpa putus pada penjabaran 𝑛 ! adalah ....
Pembahasan :
Faktor prima dari 2020 adalah 22 x 5 x 101, yang terbesar adalah 101. Banyaknya
angka nol di belakang tanpa putus dari pengembangan 101 ! adalah 24

101 101
⌊ ⌋ + ⌊ 2 ⌋ = 20 + 4 = 24
5 5

101!=9425947759838359420851623124482936749562312794702543768327889353416
977599316221476503087861591808346911623490003549599583369706302603264000
000000000000000000000

Problem 20
Jika n adalah bilangan asli sehingga 3𝑛 adalah faktor dari 37 ! , maka nilai n
terbesar yang mungkin adalah ...

Pembahasan :
Pangkat tertinggi dari p dalam 𝑚 ! (p ≤ m , p prima) adalah
𝑛 𝑛 𝑛
⌊ ⌋ + ⌊ 2⌋ + ⌊ 3⌋ + ⋯
𝑝 𝑝 𝑝
37 37
Nilai n terbesar sehingga 3𝑛 adalah faktor dari 37 ! = ⌊ 3 ⌋ + ⌊32 ⌋ = 12 + 4 = 16
Jadi 316 adalah faktor dari 37 !

99 | T E O R I B I L A N G A N
Olimpiade Matematika SMA 2020

Problem 21
Diketahui
=
=
diperoleh dengan menulis perkalian dan operasi penjumlahan dalam pola
bergantian antara bilangan bulat berurutan. Tentukan selisih antara bilangan
bulat A dan B.
Pembahasan :

.
maka,

B – A = – 38 + 4 x (1 + 2 + 3 + .... + 19)
B – A = – 38 + 4 x (190)
B – A = 722

Problem 22
Bilangan n dapat ditulis dalam basis 14 sebagai dan dapat ditulis dalam
basis 15 sebagai . Jika dalam basis 6 bilangan n ditulis sebagai
dengan 𝑎 > 0. Tentukan bilangan n tersebut dalam basis 10 !

Pembahasan :
Dalam basis 14, bilangan n adalah 196a + 14b + c
Dalam basis 15, bilangan n adalah 225a + 15c + b
Dalam basis 6, bilangan n adalah 216a + 36c + 6a + c = 222a + 37c
Maka :
196a + 14b + c = 225a + 15c + b = 222a + 37c
Diperoleh 3a + b = 22c dan 29a + 14 = 13b
Jadi a = 4, b = 10 dan c = 1
Bilangan n dalam basis 10 = 222 (4) + 37 (1) = 925

100 | T E O R I B I L A N G A N
Olimpiade Matematika SMA 2020

Problem 23
Ada bilangan prima 𝑝 sedemikian rupa sehingga 16𝑝 + 1 merupakan bilangan
kubik bulat positif. Tentukan nilai 𝑝

Pembahasan :
Misalkan bilangan kubik tersebut 𝑎3 = 16𝑝 + 1. Dalam hal ini 𝑎 bilangan ganjil
karena 16𝑝 + 1 juga ganjil.
Dengan menggunakan (𝑎3 − 𝑏 3 ) = (𝑎 − 𝑏)(𝑎2 + 𝑎𝑏 + 𝑏 2 )

Karena 𝑎 ganjil, maka 𝑎 − 1 pastilah genap dan 𝑎2 + 𝑎 + 1 adalah ganjil.


Jika 𝑎2 + 𝑎 + 1 adalah ganjil maka 𝑎 − 1 harus merupakan kelipatan 16. Sehingga
𝑎 − 1 = 16 jadi 𝑎 = 17
Sehingga faktor lainnya, yaitu 𝑎2 + 𝑎 + 1 adalah bilangan prima
(𝑎 − 1)(𝑎2 + 𝑎 + 1) = 16𝑝
(17 − 1)(172 + 17 + 1) = 16𝑝
16(289 + 17 + 1) = 16𝑝
𝑝 = 289 + 17 + 1 = 307

Problem 24
Tentukan banyaknya bilangan bulat positif lima digit 𝑛, yang memenuhi kondisi
berikut:
(a) bilangan 𝑛 dapat dibagi 5
(b) digit pertama dan terakhir 𝑛 sama, dan
(c) jumlah digit 𝑛 dapat dibagi dengan 5

Pembahasan :
Misalnya bilangan itu 5x, y, z5, yang mana 5|𝑥 + 𝑦 + 𝑧. Misalkan x dan y digit
yang dapat berubah. Untuk setiap pasangan x , y, ada dua nilai yang tepat
memenuhi untuk masing-masing pasangan x , y. Ada tepat dua nilai z yang
sesuai dengan syarat 5|𝑥 + 𝑦 + 𝑧, oleh karena itu banyaknya bilangan yang
dimaksud adalah 10 x 10 x 2 = 200

101 | T E O R I B I L A N G A N
Olimpiade Matematika SMA 2020

Problem 25
Tentukan 3 angka terakhir abc dari bilangan ganjil 579abc agar bilangan tersebut
habis dibagi 5, 7 dan 9 !
Pembahasan :

Bilangan 579abc adalah ganjil dan habis dibagi 5, berarti nilai c = 5

Untuk c = 5
Bilangan 579ab5 habis dibagi 9 maka
5 + 7 +9 + a + b + 5 = 9n dengan n bilangan bulat positif.
26 + a + b = 9n, sehingga a + b kemungkinan bernilai 1, 10 atau 19.
Kita akan mengambil a + b = 10, karena (a , b) ≤ 9.

Bilangan 579ab5 habis dibagi 7 maka


2a + 3b = 7n dengan n bilangan bulat positif.
2a + 3b yang mungkinan adalah 7, 21 atau 28.

Dari analisa sifat pembagi bilangan tersebut, maka


Untuk a + b = 10 dan 2a + 3b = 7 , a dan b tidak memenuhi
Untuk a + b = 10 dan 2a + 3b = 21 diperoleh a = 9 dan b = 1
Untuk a + b = 10 dan 2a + 3b = 28 diperoleh a = 2 dan b = 8

Jadi untuk c = 5, nilai abc adalah 915 dan 285

102 | T E O R I B I L A N G A N
Olimpiade Matematika SMA 2020

Soal-soal Latihan Isian Singkat

1
Jika A, BC, ABC adalah bilangan yang masing-masing satu, dua dan tiga digit
bukan nol, sehingga 2 𝑥 𝐴𝐵𝐶 + 𝐵𝐶 = 876, maka 𝐴 + 𝐵 + 𝐶= ....
Kunci Jawab : 14

Bilangan yang dimulai dari angka 1 disebar dalam kolom di bawah huruf A, B,
C, D, E, F, G dan H seperti tabel berikut :

A B C D E F G H
1 2 3 4 5 6 7 8
9 10 11 12 13 14 15 16
17 18 ....... ....... ....... ...... ....... .......

Bilangan 2020 muncul dibawah huruf ..........

Kunci Jawab : 6

2020 2020 2020


3 + 5 + 72020
Sisa pembagian ( ) dibagi 8 adalah .....
3

Kunci Jawab : 1

4
Digid terakhir pada 777333 adalah ...
Kunci Jawab : 7

Banyaknya digit pada bilangan 620 adalah ....

Kunci Jawab : 16

103 | T E O R I B I L A N G A N
Olimpiade Matematika SMA 2020
6

Jika 5𝑎90𝑏7 habis dibagi 99, maka nilai a adalah ....


Kunci Jawab : 1

7
Jika 10999999999 dibagi oleh 7, maka sisanya adalah .....
Kunci Jawab : 6

8
Banyaknya bilangan bulat n sehingga 𝑛 + 1|𝑛2 + 1 adalah .....
Kunci Jawab : 3

9
Bilangan bulat positif terbesar n sehinga (𝑛 + 1)(𝑛4 + 2𝑛) + 3(𝑛3 + 57) habis
dibagi oleh 𝑛2 + 2 adalah ......
Kunci Jawab : 13

10
Banyaknya bilangan bulat positif n yang memenuhi 𝑛2 − 660 merupakan
bilangan kuadrat sempurna adalah ...
Kunci Jawab : 4

11
Jika a, b dan c bilangan real, nilai minimum 8𝑎2 + 18𝑏 2 + 3𝑐 2 − 24𝑎𝑏 − 12𝑐 + 29
adalah .....
Kunci Jawab : 17

12
Seorang siswa membuat bilangan asli dua digit yang bersisa 3 jika dibagi 7.
Jumlah semua bilangan prima yang dibuat siswa tersebut adalah ....
Kunci Jawab : 125

13
Banyaknya bilangan positif yang merupakan faktor dari 2020 adalah .....
Kunci Jawab : 12

104 | T E O R I B I L A N G A N
Olimpiade Matematika SMA 2020

14
Jika bilangan asli 𝑁 dinyatan sebagai 𝑁 = 2𝑘 4 − 2𝑘 3 + 3𝑘 2 − 𝑘 + 9 , sehingga
𝑁 ≡ 𝑥 𝑚𝑜𝑑 (2𝑘 2 + 1)
𝑁 ≡ 𝑦 𝑚𝑜𝑑 (𝑘 − 1)
Maka 𝑥 + 𝑦 = ......
Kunci Jawab : 19

15
8 9
Persamaan 𝑥 − 𝑚 = 𝑥 + 123 memiliki solusi x  Bulat ketika m  Bulat.
3 4
Nilai minimum m adalah .....
Kunci Jawab : 2

16

Nilai x pada 1099 ≡ x (mod 7) adalah ....


Kunci Jawab : 6

17
Jika m dan n dua bilangan bulat positif berbeda dan memenuhi 𝑛3 = 3𝑚4 − 𝑚3,
maka nilai minimum dari 𝑛2 − 2𝑚 adalah ...
Kunci Jawab : 30

18

Jika 1234 = 2𝑎1 + 2𝑎2 + 2𝑎3 + 2𝑎4 + 2𝑎5 , dimana 𝑎1, 𝑎2, 𝑎3, 𝑎4 𝑑𝑎𝑛 𝑎5 bilangan
cacah dan 𝑎1 < 𝑎2 < 𝑎3 < 𝑎4 < 𝑎5 , nilai 𝑎1 + 𝑎2 + 𝑎3 + 𝑎4 + 𝑎5 adalah .....
Kunci Jawab : 28

19
Bilangan bulat dua digit terbesar yang dapat membagi habis 4444444444 – 88888
adalah ....
Kunci Jawab : 36

20
Agar 𝑛3 − 𝑎𝑛 dengan 𝑎 bukan bilangan prima, merupakan bilangan yang habis
dibagi 6, maka nilai 𝑎 positif terkecil adalah ....
Kunci Jawab : 25

105 | T E O R I B I L A N G A N
Olimpiade Matematika SMA 2020

21
Jika 𝑓(𝑛) ≡ 2𝑛 + 1 𝑚𝑜𝑑 (𝑛2 + 2), maka (𝑓(𝑝))2 + 2 𝑓(𝑝) ≡ 11 𝑚𝑜𝑑 (𝑝2 + 2)
untuk nilai p = ....
Kunci Jawab : 2

22
Jika 𝑁 menyatakan hasilkali 4 𝑥 8 𝑥 12 𝑥 16 𝑥 … … 𝑥 2008 𝑥 2012 𝑥 2016 𝑥 2020
Dan 𝑛 adalah banyaknya angka nol di belakang pada N, maka n adalah ....
Kunci Jawab : 125

23
Gcd (10764 , 2300) = ....
Kunci Jawab : 92

24
Harga n terbesar sehingga 8n membagi 4444 adalah ...
Kunci Jawab : 29

25
Jika 𝑚 dan 𝑛 merupakan bilangan bulat yang tidak habis dibagi 10, sehingga
memenuhi 𝑚𝑛 = 500, maka nilai 𝑚 + 𝑛 = ....
Kunci Jawab : 633

26
Untuk setiap x sebarang bilangan real, ⌊𝑥⌋ merupakan bilangan bulat terbesar
yang kurang dari atau sama dengan x. Persamaan 𝑥 2 = 3⌊𝑥⌋ + 1 memiliki dua
penyelesaian, yaitu 𝑥1 = √7 dan 𝑥2 = √𝑎 dengan 𝑎 bilangan real positif. Nilai 𝑎
adalah .....
Kunci Jawab : 10

27
Banyaknya bilangan asli 𝑛 ∈ {1, 2, 3, 4, … . , 1000} sehingga terdapat bilangan real
𝑥 yang memenuhi 𝑥 2 + ⌊𝑥⌋2 = 𝑛 adalah .....
Kunci Jawab : 516

106 | T E O R I B I L A N G A N
Olimpiade Matematika SMA 2020

28
Untuk bilangan real x, maka ⌊𝑥⌋ didefinisikan sebagai bilangan bulat yang
kurang dari atau sama dengan x. Sebagai contoh ⌊6,9⌋ = 6 dan ⌊5⌋ = 5. Jika x dan
y bilangan real sehingga ⌊𝑥⌋ = 9 dan ⌊𝑦⌋ = 12 maka nilai terkecil yang mungkin
dicapai oleh ⌊𝑦 − 𝑥⌋ adalah ...
Kunci Jawab : 134

29
Untuk sebarang bilangan real 𝑎, maka ⌊𝑎⌋ didefinisikan sebagai bilangan bulat
yang kurang dari atau sama dengan 𝑎. Jika x bilangan real yang memenuhi
⌊𝑥 + √3⌋ = ⌊𝑥⌋ + ⌊√3⌋. Maka 𝑥 − ⌊𝑥⌋ tidak lebih besar dari ....
Kunci Jawab : 0,2

30
Suatu bilangan apabila dibagi 4 dan 5 maka masing-masing bersisa 1 dan 3. Jika
bilangan itu dibagi 20 maka sisanya adalah ....
Kunci Jawab : 13

31
Banyaknya solusi pasangan (x , y) bulat positif yang memenuhi peramaan
Diophantine 3x + 5y = 501 adalah ....
Kunci Jawab : 33

32
Jika a dan b bukan dua bilangan berurutan sehingga 𝑎2 − 𝑏 2 = 1991 maka nilai
𝑎 + 𝑏 yang memenuhi adalah ...
Kunci Jawaban : 181

107 | T E O R I B I L A N G A N
Olimpiade Matematika SMA 2020

33
Misalkan a * b adalah bilangan bulat untuk setiap bilangan bulat a dan b. Untuk
setiap bilangan bulat a dan b, kita memiliki sifat (a + 1) * b – (a – 1) * b = 4a dan
b * a = –(a * b). Jika 1 * 0 = 1, carilah nilai dari 101 * 100.
Kunci Jawab : 201

34
Ada 5 pernyataan. Masing-masing dari lima pernyataan berikut ini benar atau
salah. Berikut beberapa petunjuk :
(1) Pernyataan (3) dan (4) keduanya benar.
(2) Pernyataan (4) dan (5) keduanya tidak salah.
(3) Pernyataan (1) benar.
(4) Pernyataan (3) salah.
(5) Pernyataan (1) dan (3) keduanya salah.
Berapa banyak pernyataan dari (1) sampai (5) yang benar ?
Kunci Jawab : 3

35
Bilangan 4-angka dibentuk dari 1, 5, 6 dan 9 dimana masing-masing angka
digunakan tepat satu kali. Jika semua bilangan 4-angka yang diperoleh dengan
cara ini dijumlahkan, maka jumlah ini mempunyai angka satuan ⋅⋅⋅⋅⋅⋅⋅
Kunci Jawab : 24

108 | T E O R I B I L A N G A N
Olimpiade Matematika SMA 2020

Soal-soal Latihan Uraian Singkat


1. Jika a dan b dipilih dari bilangan 1, 2, 3, 4 ......9, banyaknya pasangan (𝑎 , 𝑏)
yang memenuhi bilangan bulat 30a0b03 agar habis dibagi 13 adalah ......

2. Buktikan bahwa ada tak hingga banyaknya bilangan N, jika N dibagi 3


bersisa 1, jika N dibagi 5 bersisa 2 dan jika N dibagi 7 bersisa 6 !

3. Diketahui S = {10, 11, 12,··· , N }. Suatu unsur di S dikatakan trubus jika


jumlah digit-digitnya merupakan pangkat tiga dari suatu bilangan asli.
Jika S memiliki tepat 12 trubus, maka nilai terbesar N yang mungkin
adalah ...

4. Jika n adalah banyaknya anggota himpunan bilangan 3 digit dengan selisih


digit pertama dan digit ketiga sebesar 2, maka n adalah ....

5. Tentukan bilangan bulat dua digit terbesar yang membagi habis


33333 … . .333 − ⏟
⏟ 666 … … .66
2𝑛 𝑘𝑎𝑙𝑖 𝑛 𝑘𝑎𝑙𝑖

6. N menyatakan 44 bilangan positif pertama yang merupakan bilangan berdigit


banyak sebagai, N = 12345678910111213………. 4041424344.
Berapa sisanya jika N dibagi 5 ?

7. Tentukan sisanya 3636 + 4141 jika dibagi 77 !

8. Berapa banyak solusi bulat 0 < 𝑥 ≤ 100 untuk 𝑥 ≡ 1 𝑚𝑜𝑑 2 dan 𝑥 ≡ 2 𝑚𝑜𝑑 3 !

9. Carilah sisa pembagian 683 + 883 jika dibagi 49 !

10. Sebuah propinsi terdiri dari 9 kota yang diberi nama 1, 2, 3, 4, 5, 6, 7, 8, dan 9.
Dari kota 𝑎 terdapat jalan langsung ke kota 𝑏 jika dan hanya jika ̅̅̅ 𝑎𝑏 dan ̅̅̅
𝑏𝑎
merupakan bilangan dua digit yang habis dibagi 3.
Dua buah kota berbeda 𝑎1 dan 𝑎𝑛 dikatakan terhubung jika terdapat
barisan kota 𝑎1 , 𝑎2 , 𝑎3 .... 𝑎𝑛 sehingga terdapat jalan langsung dari 𝑎𝑖 ke
𝑎𝑖+1 untuk setiap 𝑖 = 1, 2, 3, … . 𝑛 − 1. Banyaknya kota yang terhubung
dengan kota 4 adalah ...

109 | T E O R I B I L A N G A N
Olimpiade Matematika SMA 2020

Soal dan Pembahasan


Soal & Pembahasan nomor 1 :
Jika a dan b dipilih dari bilangan 1, 2, 3, 4 ......9, banyaknya pasangan (𝑎 , 𝑏) yang
memenuhi bilangan bulat 30a0b03 agar habis dibagi 13 adalah ......

Pembahasan :
Perhatikan posisi angka pada penjabaran 30a0b03 sebagai berikut :
30a0b03 = 3 (100)3 + a (100)2 + b (100) + 3

Karena 100  – 4 mod 13 , maka :


30a0b03  (3 (– 4)3 + a (– 4)2 + b (−4) + 3) mod 13
30a0b03  (3 (−64) + a (16) − 4b + 3) mod 13
30a0b03  (3 (−64) + a (16) − 4b + 3) mod 13
30a0b03  (3 (−4(13) − 12) + 16a − 4b + 3) mod 13
30a0b03  (3(−4)(13)(3) + 3(−12) + 16a − 4b + 3) mod 13

Karena (3(−4)(13)(3))  0 mod 13, maka :


30a0b03  (0 + 3(−12) + 16a − 4b + 3) mod 13
30a0b03  (−33 + 16a − 4b ) mod 13
30a0b03  (−2(13) − 7 + 13a + 3a − 4b ) mod 13

Karena (−2(13) + 13a)  0 mod 13, maka :


30a0b03  (−7 + 3a − 4b ) mod 13

Agar 30a0b03 habis dibagi 13, maka 30a0b03  0 mod 13,


sehingga −7 + 3a − 4b = 0

Karena 3a − 4b = 7 persamaan linier, maka ada 2 pasang (𝑎 , 𝑏) yang


memenuhi.

Pasangan (a ,b) yang dimaksud adalah (5 , 2) dan (9 , 5)

110 | T E O R I B I L A N G A N
Olimpiade Matematika SMA 2020

Soal & Pembahasan nomor 2 :


Buktikan bahwa ada tak hingga banyaknya bilangan N jika N dibagi 3 bersisa 1,
jika N dibagi 5 bersisa 2 , dan jika N dibagi dengan 7 bersisa 6 !

Pembahasan :
Karena bilangan yang ditanyakan tak terhingga banyaknya , maka bilangan
yang dicari dibatasi dengan bilangan yang terkecil. Misalkan N adalah bilangan
bulat positif terkecil, dimana N < 105 , 105 = KPK ( 3, 5, 7).
N dibagi 3 bersisa 1
N dibagi 5 bersisa 2
N dibagi 7 bersisa 6
Karena FPB(3, 5 , 7) = 1 , maka kita dapat menentukan bilangan pengali untuk
sisa masing-masing pembagi.

• Bilangan kelipatan persekutuan terkecil dari (5 dan 7) yang bersisa


1 dibagi 3 adalah 70 ,
• Bilangan kelipatan persekutuan terkecil dari (3 dan 7) yang bersisa
1 dibagi 5 adalah 21 ,
• Bilangan kelipatan persekutuan terkecil dari (3 dan 5) yang bersisa
1 dibagi 7 adalah 15

Sehingga , Salah satu bilangan N = 1 x 70 + 2 x 21 + 6 x 15 = 70 + 42 + 90 = 202 ,


Tetapi bilangan N yang diminta adalah bilangan bulat positif terkecil
Jadi, N = 202 – 105 = 97 .

Jika soal tersebut tanpa pembatasan, maka jawaban soal tersebut haruslah.
bersifat umum, yaitu N = 97 + k . 105 , dengan k adalah bilangan cacah .

N = 97 + k . 105
Untuk k = 0, 1, 2, 3, .... maka nilai N tak hingga banyaknya

Soal & Pembahasan nomor 3 :


Diketahui S = {10, 11, 12,··· , N }. Suatu unsur di S dikatakan trubus jika
jumlah digit-digitnya merupakan pangkat tiga dari suatu bilangan asli. Jika S
memiliki tepat 12 trubus, maka nilai terbesar N yang mungkin adalah ...

111 | T E O R I B I L A N G A N
Olimpiade Matematika SMA 2020

Pembahasan :
Bilangan trubus adalah bilangan yang digid-digidnya merupakan pangkat tiga
(bilangan kubik) dari suatu bilangan asli, contohnya 17 dan 116 karena masing-
masing jumlah digidnya adalah 8 (bilangan kubik)

Jika S = {10, 11, 12,··· , N } memiliki 12 bilangan trubus, maka dapat kita
telusuri bilangan-bilangan trubus yang terdiri dari :
Dua digid sebanyak 9 bilangan, yaitu : 10, 17, 26, 35, 44, 53, 62, 71, dan 80
Diperlukan 3 bilangan Tiga Digid lagi, yaitu 100, 107, 116 dan berikutnya 125
Maka nilai N terbesar tidak boleh melampaui 125. Maka N terbesar adalah 124

Soal & Pembahasan nomor 4 :


Jika n adalah banyaknya anggota himpunan bilangan 3 digit dengan selisih digit
pertama dan digit ketiga sebesar 2, maka n adalah ....

Pembahasan :
Bilangan tiga digid atau ratusan adalah bilangan yang memiliki 3 angka dengan
angka pertama tidak boleh 0. Selisih angka pertama dan angka ketiga adalah 2.
Lihat tabel :

Angka Angka Angka Hasil


pertama kedua ketiga
1 0 sd 9 3 20 cara
2 0 sd 9 4 20 cara
3 0 sd 9 5 20 cara
4 0 sd 9 6 20 cara
5 0 sd 9 7 20 cara
6 0 sd 9 8 20 cara
7 0 sd 9 9 20 cara
2 0 sd 9 0 10 cara

Jadi banyaknya bilangan ratusan yang dimaksud adalah 150


Jadi nilai 𝑎 = 9 dan 𝑏 = 18 pada nilai k terkecil menyebabkan nilai 𝑎𝑏 terkecil.
𝑎𝑏 = 9 . 18 = 162

112 | T E O R I B I L A N G A N
Olimpiade Matematika SMA 2020

Soal & Pembahasan nomor 5 :


Tentukan bilangan bulat dua digit terbesar yang membagi habis
33333 … . .333 − ⏟
⏟ 666 … … .66
2𝑛 𝑘𝑎𝑙𝑖 𝑛 𝑘𝑎𝑙𝑖

Pembahasan :

⏟ 666 … … .66 = ⏟
33333 … . .333 − ⏟ 333 … . .33 𝑥 10𝑛 + ⏟
333 … . .33 − ⏟
666 … … .66
2𝑛 𝑘𝑎𝑙𝑖 𝑛 𝑘𝑎𝑙𝑖 𝑛 𝑘𝑎𝑙𝑖 𝑛 𝑘𝑎𝑙𝑖 𝑛 𝑘𝑎𝑙𝑖
33333 … . .333 − ⏟
⏟ 333 … . .33 𝑥 10𝑛 − ⏟
666 … … .66 = ⏟ 333 … . .33
2𝑛 𝑘𝑎𝑙𝑖 𝑛 𝑘𝑎𝑙𝑖 𝑛 𝑘𝑎𝑙𝑖 𝑛 𝑘𝑎𝑙𝑖
33333 … . .333 − ⏟
⏟ 333 … . .33 (10𝑛 − 1)
666 … … .66 = ⏟
2𝑛 𝑘𝑎𝑙𝑖 𝑛 𝑘𝑎𝑙𝑖 𝑛 𝑘𝑎𝑙𝑖
⏟ 666 … … .66 = ⏟
33333 … . .333 − ⏟ 333 … . .33 (999
⏟ … . .99)
2𝑛 𝑘𝑎𝑙𝑖 𝑛 𝑘𝑎𝑙𝑖 𝑛 𝑘𝑎𝑙𝑖 𝑛 𝑘𝑎𝑙𝑖
⏟ 666 … … .66 = 27 ⏟
33333 … . .333 − ⏟ 111 … . .11 (111
⏟ … . .11)
2𝑛 𝑘𝑎𝑙𝑖 𝑛 𝑘𝑎𝑙𝑖 𝑛 𝑘𝑎𝑙𝑖 𝑛 𝑘𝑎𝑙𝑖
2
⏟ 666 … … .66 = 27 (111
33333 … . .333 − ⏟ ⏟ … . .11)
2𝑛 𝑘𝑎𝑙𝑖 𝑛 𝑘𝑎𝑙𝑖 𝑛 𝑘𝑎𝑙𝑖

Ternyata 27 adalah faktor bulat dua digit terbesar dari


33333 … . .333 − ⏟
⏟ 666 … … .66
2𝑛 𝑘𝑎𝑙𝑖 𝑛 𝑘𝑎𝑙𝑖

Soal & Pembahasan nomor 6 :


N menyatakan 44 bilangan positif pertama yang merupakan bilangan berdigit
banyak sebagai, N = 12345678910111213………. 4041424344
Berapa sisanya jika N dibagi 45 ?

Pembahasan :
N = 12345678910111213………. 4041424344.
Mengingat, 45 = 9 x 5
Jika N dibagi oleh 9 maka 1 + 2 + 3 +…+ 41 + 42 + 43 + 44 = 990
Jumlah digit 990 adalah 9 + 9 + 0 = 18 dan 1 + 8 = 9
Ternyata N habis dibagi 9.
Jika N dibagi 5 maka sisanya 4, berdasarkan digit terakhir N itu sendiri.
Dengan demikian, N jika dibagi 45 maka sisanya 9 karena satu-satunya bilangan
dari 0 sampai 44 yang habis dibagi 9 dan bersisa 4 jika dibagi 5 adalah 9.

113 | T E O R I B I L A N G A N
Olimpiade Matematika SMA 2020

Soal & Pembahasan nomor 7 :


Tentukan sisanya 3636 + 4141 jika dibagi 77 !

Pembahasan :
36  3 mod 11
365  35 mod 11  1 mod 11
Jadi 365  1 mod 77

41  –36 mod 77
4141 ≡ (−36)41 𝑚𝑜𝑑 77

3636 + 4141 ≡ (3636 + (−36)41 )𝑚𝑜𝑑 77

3636 + 4141 ≡ 3636 (1 − 365 ) 𝑚𝑜𝑑 77

3636 + 4141 ≡ 3636 (1 − 1) 𝑚𝑜𝑑 77

3636 + 4141 ≡ 3636 . (0) 𝑚𝑜𝑑 77

3636 + 4141 ≡ 0 𝑚𝑜𝑑 77


Dengan demikian sisa 3636 + 4141 jika dibagi oleh 77 adalah 0.

Soal & Pembahasan nomor 8 :


Berapa banyak solusi bulat 0 < 𝑥 ≤ 100 untuk 𝑥 ≡ 1 𝑚𝑜𝑑 2 dan 𝑥 ≡ 2 𝑚𝑜𝑑 3 !

Pembahasan :
𝑥 ≡ 1 𝑚𝑜𝑑 2 dapat ditulis dalam bentuk x = 1 + 2y
1 + 2𝑦 ≡ 2 𝑚𝑜𝑑 3 sehingga menjadi 2𝑦 ≡ 1 𝑚𝑜𝑑 3

Karena gcd (3 , 2) = 1
maka akan ada bilangan bulat m dan n sehingga 2m + 3n = 1, yaitu m = − 1 dan
n = 1, sehingga 𝑦 ≡ 2 𝑚𝑜𝑑 3 atau y = 2 + 3k dengan k bilangan bulat.

Dengan cara substitusi :


x = 1 + 2y dan y = 2 + 3k, maka :
x = 1 + 2 (2 + 3k)
x = 5 + 6k
jadi 𝑥 ≡ 5 𝑚𝑜𝑑 6
𝑥 = {11, 17, 23, … … . . 95} ada 15 bilangan yang merupakan solusi.

114 | T E O R I B I L A N G A N
Olimpiade Matematika SMA 2020

Soal & Pembahasan nomor 9 :


Carilah sisa pembagian 683 + 883 jika dibagi 49 !

Pembahasan :
683 + 883 = (7 – 1)83 + (7 + 1)83
683 + 883 = (783 – 𝐶831 7
82
+ 𝐶83 81
2 7 . . . . – 1 ) + (7
83
+ 𝐶831 7
82
+
𝐶83 81
2 7 ....+1)
683 + 883 = 2(783 + 𝐶83 2 7
81
+ ⋯ … + 𝐶83 3 83
80 7 + 𝐶82 7 )
683 + 883 = 2(783 + 𝐶83 2 7
81
+ ⋯ … + 𝐶83 3 83
80 7 ) + 2. 𝐶82 7
683 + 883 = 2. 72 (781 + 𝐶83 2 7
79
+ ⋯ … + 𝐶83 83
80 7) + 2. 𝐶82 7

Sisa pembagian 683 + 883 jika dibagi 49 adalah 14 . 𝐶82


83

83
83 !
14 . 𝐶82 = 14 = 14 . 83 = 1162
1 ! . 82 !
1162 masih dapat dibagi 49 hasilnya adalah 23 dan sisa 35.
Jadi Sisa pembagian 683 + 883 jika dibagi 49 adalah 35.

Soal & Pembahasan nomor 10 :


Sebuah propinsi terdiri dari 9 kota yang diberi nama 1, 2, 3, 4, 5, 6, 7, 8, dan 9.
Dari kota 𝑎 terdapat jalan langsung ke kota 𝑏jika dan hanya jika ̅̅̅ 𝑎𝑏 dan ̅̅̅
𝑏𝑎
merupakan bilangan dua digit yang habis dibagi 3. Dua buah kota berbeda 𝑎1
dan 𝑎𝑛 dikatakan terhubung jika terdapat barisan kota 𝑎1 , 𝑎2 , 𝑎3 .... 𝑎𝑛
sehingga terdapat jalan langsung dari 𝑎𝑖 ke 𝑎𝑖+1 untuk setiap 𝑖 = 1, 2, 3, … . 𝑛 − 1.
Banyaknya kota yang terhubung dengan kota 4 adalah ...

Pembahasan :
Kota 4 memiliki karakter 4 ≡ 1 mod 3, artinya 4 jika dibagi 3 akan bersisa 1. Maka
kota 4 akan terhubung dengan kota lain yang apabila dibagi 3 bersisa 2, seperti
Kota 2, kota 5 dan kota 8.

Kota 2, 5 dan 8 adalah kota yang memiliki karakter 2 mod 3.


Maka kota-kota ini akan terhubung secara otomatis dengan kota 4 itu sendiri,
kota 1 dan kota 7, karena kota 1, 4, 7 memiliki karakter 1 mod 3.

Kota 3, 6, 9 adalah kota akan bersisa 0 apabila dibagi 3, sehingga kota 3, 6 dan 9
tidak terhubung dengan kota 4.

Jadi kota 4 terhubung dengan 5 kota, yaitu kota 1, 2, 5, 7 dan 8.

115 | T E O R I B I L A N G A N
Olimpiade Matematika SMA 2020

CATATAN :
.................................................................................................... ...........................................
.................................................................................................... ...........................................
.................................................................................................... ...........................................
.................................................................................................... ...........................................
.................................................................................................... ...........................................
.................................................................................................... ...........................................
.................................................................................................... ...........................................
.................................................................................................... ...........................................
.................................................................................................... ...........................................
.................................................................................................... ...........................................
.................................................................................................... ...........................................
.................................................................................................... ...........................................
.................................................................................................... ...........................................
.................................................................................................... ...........................................
.................................................................................................... ...........................................
.................................................................................................... ...........................................
.................................................................................................... ...........................................
.................................................................................................... ...........................................
.................................................................................................... ...........................................
.................................................................................................... ...........................................
.................................................................................................... ...........................................
.................................................................................................... ...........................................
.................................................................................................... ...........................................
.................................................................................................... ...........................................
.................................................................................................... ...........................................
.................................................................................................... ...........................................
.................................................................................................... ...........................................
.................................................................................................... ...........................................
.................................................................................................... ...........................................
.................................................................................................... ...........................................
.................................................................................................... ...........................................
.................................................................................................... ...........................................
.................................................................................................... ...........................................
.................................................................................................... ...........................................
.................................................................................................... ...........................................
.................................................................................................... ...........................................

116 | T E O R I B I L A N G A N
Olimpiade Matematika SMA 2020

KOMBINATORIKA
Bab 3
PENGANTAR TEORI
• ATURAN PENEMPATAN
(FILLING SLOTS)

Aturan Penjumlahan
Aturan penjumlahan pada prinsipnya berdasarkan himpunan saling lepas atau
saling asing. Jika k1, k2, k3 ... kn adalah kejadian-kejadian saling lepas atau asing,
maka banyaknya cara memilih kejadian adalah k1 + k2 + k3 + ...+ kn

Contoh Soal 1
Untuk melakukan perjalanan dari Jakarta ke Surabaya disediakan jalur darat dan
udara. Jalur darat dengan bus terdapat 5 route sedangkan jalur udara terdapat 3
route. Berapa cara seseorang melakukan perjalanan dari Jakarta ke Surabaya ?

Pembahasan :
Dua macam jalur tersebut saling asing sehingga seseorang yang ingin melakukan
perjalanan dari Jakarta ke Surabaya memiliki 3 + 5 pilihan atau 8 pilihan

Aturan Perkalian
Misalkan ada n tempat tersedia dengan k1 adalah banyaknya cara mengisi
tempat pertama, k2 adalah banyaknya cara mengisi tempat kedua, dan
seterusnya hingga kn adalah banyaknya cara mengisi tempat ke-n. Maka
banyaknya cara mengisi tempat adalah

k1 x k2 x k3 x k4 x ........... x kn

Contoh Soal 2 :
Jika 4 orang akan duduk pada 7 kursi yang tersedia, berapa banyaknya cara
mereka duduk ?

Pembahasan :
Karena ada 4 orang maka jika kita persilahkan orang pertama pasti ada 7 pilihan.
Orang kedua 6 pilihan, orang ketiga 5 pilihan, setelah itu sisa 4 kursi yang dipilih
oleh orang keempat.
Jadi cara mereka memilih tempat duduk adalah 7 x 6 x 5 x 4 = 840 cara

117 | K O M B I N A T O R I K A
Olimpiade Matematika SMA 2020

Contoh Soal 3 :
Misalkan ada rak buku terdiri dari tiga kotak penyimpanan. Setiap kotak berisi
buku-buku bidang studi yang sama. Jika ke dalam rak itu akan disimpan 4 buku
matematika, 3 buku fisika dan 2 buku kimia. Berapa cara penyusunan yang
berbeda buku-buku itu di dalam rak ?

Pembahasan :
Susunan rak (M , F , K) , (M , K , F) , (F , M , K) , (F , K , M), (K , M , F),
(K , F , M), ada 6 cara urutan kotak buku dalam rak. Atau dengan menggunakan
aturan Perkalian, cara menyusun urutan kotak dalam rak ada 3 x 2 x 1 = 6 cara
Penyusunan 4 buku matematika dalam kotaknya = 4 x 3 x 2 x 1 = 24 Cara
Penyusunan 3 buku fisika dalam kotaknya = 3 x 2x 1 = 6 Cara
Penyusunan 2 buku kimia dalam kotaknya = 2 x 1 = 2 Cara
Jadi penyusunan buku-buku pada ketiga kotak dalam rak = 24 x 6 x 2 = 288 cara
Jadi, banyaknya penyusunan buku dalam rak itu adalah 6 x 288 cara = 1728 cara

• PERMUTASI DAN KOMBINASI

Permasalahan permutasi adalah menentukan banyaknya penyusunan yang


berbeda dalam pengaturan obyek-obyek. Permutasi merupakan bentuk khusus
dari penggunaan aturan perkalian. Agar memudahkan dalam penulisan notasi,
kita gunakan Faktorial.

n! = n (n – 1) (n – 2) (n – 3) (n – 4) ....... 3 . 2 . 1
0! = 1
4! = 4 x 3 x 2 x 1 = 24
5! = 5 . 4! = 5 . 24 = 120
6! = 6 . 5! = 6 . 120 = 720

Permutasi r dari n elemen, 𝑃𝑟𝑛 atau P(n, r), adalah banyaknya kemungkinan
urutan r buah elemen yang dipilih dari n buah elemen, dengan r ≤ n, yang dalam
pemilihan ini urutan diperhatikan. Dalam permutasi susunan ab berbeda dengan
susunan ba.
𝑛!
𝑃𝑟𝑛 = 𝑃(𝑛 , 𝑟) =
(𝑛 − 𝑟)!

118 | K O M B I N A T O R I K A
Olimpiade Matematika SMA 2020

Contoh Soal 4 :
Ada berapa banyak bilangan terdiri dari 3 angka berbeda yang disusun dari
angka-angka 1, 2, 3, 4 dan 5 ?

Pembahasan :
Permutasi 3 dari 5 angka dinyatakan dengan

5! 5!
𝑃35 = 𝑃(5 , 3) = = = 60
(5 − 3)! 2!

Jika menggunakan aturan pengisian tempat maka bilangan tersebut masuk


dalam kategori tidak berulang. Karena 123 dianggap berbeda dengan 231 maka
berarti urutan diperhatikan.

Contoh Soal 5
Perhatikan gambar. Jika seseorang akan berjalan dari titik A ke titik B. Ada
berapa banyak cara jalan terpendek yang dapat dipilihnya ?

B
P P P P P P P
P P P P P P P
P P P P P P P
P P P P P P P
A

Pembahasan :
Jalan terpendek yang bisa dipilih orang tersebut adalah jika ia memilih jalan ke
kanan atau ke atas tanpa berjalan ke kiri atau ke bawah. Misalkan jika berjalan ke
kanan diberi tanda 1 dan jika ke atas diberi tanda 2. Jadi jika 12111211212 maka
orang tersebut berjalan ke kanan lalu ke atas lalu ke kanan tiga kali lalu ke atas
lalu ke kanan dua kali lalu ke atas lalu ke kanan lalu ke atas.

Maka persoalan di atas adalah mencari banyaknya susunan 12111211212 yang


merupakan permutasi berulang 11 angka dengan terdapat 7 angka 1 yang sama
dan 4 angka 2 yang sama.

Banyaknya susunan adalah


11! 11 𝑥 10 𝑥 9 𝑥 8
𝑃= = = 330
7! 4! 4𝑥3𝑥2𝑥1

119 | K O M B I N A T O R I K A
Olimpiade Matematika SMA 2020

Contoh Soal 6
Himpunan S merupakan himpunan bilangan-bilangan 7 digit sehingga
masing-masing angka 1, 2, 3, 4, 5, 6, atau 7 tepat muncul satu kali. Bilangan-
bilangan di S diurutkan mulai dari yang paling kecil sampai yang paling
besar. Bilangan yang berada pada urutan ke-2020 adalah ...

Pembahasan :
Bilangan berawalan 1 dan 2 totalnya ada 1440 cara. Enam kotak berikutnya
dapat diisi dengan bilangan sisanya sebanyak 6 bilangan secara permutasi.

Bilangan berawal 1 1 6 ! cara = 720 cara

Bilangan berawal 2 2 6 ! cara = 720 cara

Berawalan 31 3 1 5 ! cara = 120 cara

Berawalan 32 3 2 5 ! cara = 120 cara

Berawalan 34 3 4 5 ! cara = 120 cara

Berawalan 35 3 5 5 ! cara = 120 cara

Berawalan 361 3 6 1 4 ! cara = 24 cara

Berawalan 362 3 6 2 4 ! cara = 24 cara

Berawalan 364 3 6 4 4 ! cara = 24 cara

Berawalan 365 3 6 5 4 ! cara = 24 cara

Banyaknya cara penyusunan bilangan anggota S yang telah disebutkan di atas


adalah 2016.
Maka mulai urutan ke 2017 sampai ke 2020 adalah
Mulai urutan ke 2017 = 3671245 , 3671425 , 3671452 , 3671524
Bilangan yang berada pada urutan ke-2020 adalah 3671524.

120 | K O M B I N A T O R I K A
Olimpiade Matematika SMA 2020

Contoh Soal 7
Banyaknya bilangan delapan digit yang setiap digitnya 1 atau 2 tetapi tidak
memuat tiga digit 1 berurutan adalah ....

Pembahasan :
Formasi dan cara penyusunan bilangan 8 digid dengan digit-digit 1 dan 2 :

• Tidak mengandung angka 1, seperti 22222222, ada 1 cara


8!
• Salah satu digitnya bernilai 1, seperti 22222221, ada = 8 cara
7!
8!
• Dua digit bernilai 1, seperti 22222211, ada = 28 cara
6! 2!

• Tiga digit bernilai 1, seperti 22222111.


Tetapi :
Apabila nilai 1 terletak pada posisi berurutan, 22222111, maka ada 3 ! = 6 cara
8!
Apabila tidak ada nilai 1 berurutan, maka ada −6 = 56 – 6 = 50 cara
5! 3!

8!
• Empat digit bernilai 1, seperti 22221111, ada = 70 cara
4! 4!
Tetapi :
Ada nilai 1 empat kali berurutan, seperti 22221111 , ada 5 cara
Ada nilai 1 tiga kali berurutan dan satu nilai 1 terpisah, 21222111, ada 20 cara
Maka jika ada empat angka 1 sesuai syarat = 70 – 5 – 20 = 45 cara

8!
• Lima digit bernilai 1 , seperti 22211111, ada = 56 cara
3! 5!
Tetapi :
Ada lima digit 1 berurutan, seperti 11111222, ada 4 cara
Ada empat digit 1 berurutan dan satu digit 1 terpisah, 11112212 ada 12 cara
Lima digit 1 terpisah menjadi 11 dan 111, 11221112, ada 12 cara
Lima digit 1 terpisah 1, 1 dan 111 , seperti 122121112, ada 12 cara
Maka jika ada lima angka 1 sesuai syarat = 56 – 4 – 12 – 12 – 12 = 16 cara

• Ada enam digit 1 tersusun seperti 11211211, ada 1 cara

Jadi banyaknya cara = 1 + 8 + 28 + 50 + 45 + 16 + 1 = 149 cara

121 | K O M B I N A T O R I K A
Olimpiade Matematika SMA 2020

Persoalan kombinasi 𝐶𝑟𝑛 atau 𝐶 (𝑛 , 𝑟) adalah menghitung banyaknya himpunan


bagian dengan r elemen yang dapat dibentuk dari himpunan dengan n elemen.
Dengan demikian beberapa himpunan dengan elemen-elemen sama (meskipun
urutan berbeda) merupakan himpunan yang sama, sehingga dihitung sekali.
Banyaknya kombinasi r unsur yang diambil dari n unsur yang tersedia dengan
r ≤ n dirumuskan dengan :
𝑛!
𝐶𝑟𝑛 = 𝐶(𝑛 , 𝑟) =
(𝑛 − 𝑟)! 𝑟!
Dalam hal ini, 𝑃𝑟𝑛 ≥ 𝐶𝑟𝑛
Kata kunci yang membedakan antara kombinasi dan permutasi adalah
memperhatikan atau tidak memperhatikan urutan.

Contoh Soal 8
Ada berapa banyak cara memilih 3 orang siswa dari 8 orang siswa yang akan
ditunjuk sebagai Pengurus Kelas ?

Pembahasan :
Dalam memilih A, B dan C sebagai pengurus kelas urutan tidak diperhatikan.
Maka banyaknya cara ada
8! 8!
𝐶38 = 𝐶 (8 , 3) = = = 56
(8 − 3)! 6! 5 ! 3 !

Banyaknya pemilihan tak berurutan sebanyak r dari n yang memperbolehkan


pengulangan adalah Crn+r−1 = 𝐶 (𝑛 + 𝑟 − 1 , 𝑟)

Contoh Soal 9
Dengan angka-angka 1, 2, 3, 4, ....... 9 akan dibuat bilangan-bilangan enam digit
dengan pola abcdef. Jika 𝑎 < 𝑏 ≤ 𝑐 < 𝑑 ≤ 𝑒 < 𝑓 , maka banyaknya bilangan
tersebut adalah ...

Pembahasan :
Cara penyusunan angka 1, 2, 3, 4 ..... 9 dengan pola abcdef adalah sebagai berikut

Syarat 𝑎 < 𝑏 < 𝑐 < 𝑑 < 𝑒 < 𝑓 sebanyak 𝐶69 = 84 cara


Syarat 𝑎 < 𝑏 = 𝑐 < 𝑑 < 𝑒 < 𝑓 , ada sebanyak 𝐶59 = 126 cara
dan 𝑎 < 𝑏 < 𝑐 < 𝑑 = 𝑒 < 𝑓 , ada sebanyak 𝐶59 = 126 cara
Syarat 𝑎 < 𝑏 = 𝑐 < 𝑑 = 𝑒 < 𝑓 sebanyak 𝐶49 = 126 cara
Maka banyaknya cara penyusunan adalah 84 + 126 + 126 + 126 = 462 cara

122 | K O M B I N A T O R I K A
Olimpiade Matematika SMA 2020

Contoh Soal 10
Berapa banyak cara memilih secara acak 2 bola merah, 3 bola putih dan 1 bola
kuning dari dalam kotak yang berisi 5 bola merah, 4 bola putih dan 3 bola
kuning ?

Pembahasan :
Banyaknya cara memilih 2 bola merah dari 5 bola merah adalah
5! 6!
𝐶25 = = = 10
(5 − 2)! 2! 3 ! 2 !
Banyaknya cara memilih 3 bola putih dari 4 bola putih adalah
4! 4!
𝐶34 = = =4
(4 − 3)! 3 ! 1 ! 3 !
Banyaknya cara memilih 1 bola kuning dari 3 bola kuning adalah
3! 6!
𝐶13 = = =3
(3 − 1)! 1! 2 ! 1 !
Maka banyaknya cara memilih 2 bola merah, 3 bola putih dan 1 bola kuning
adalah 10 x 4 x 3 = 120 cara.

Contoh Soal 11
Tujuh buah bendera dengan motif berbeda akan dipasang pada 4 tiang bendera.
Pada masing-masing tiang dapat dipasang nol, satu atau lebih dari 1 bendera.
Banyaknya cara pemasangan bendera tersebut adalah ....

Pembahasan :
Cara memilih 7 bendera dengan motif berbeda 7 !
4!
Cara pemasangan pada tiang (T1, T2, T3, T4) = (0 , 0 , 0 , 7) = = 4 cara
3!
4!
Cara pemasangan pada tiang (T1, T2, T3, T4) = (0 , 0 , 1 , 6) = = 12 cara
2!
4!
Cara pemasangan pada tiang (T1, T2, T3, T4) = (0 , 0 , 2 , 5) = = 12 cara
2!
4!
Cara pemasangan pada tiang (T1, T2, T3, T4) = (0 , 0 , 3 , 4) = = 12 cara
2!
4!
Cara pemasangan pada tiang (T1, T2, T3, T4) = (0 , 1 , 1, 5) = = 12 cara
2!

123 | K O M B I N A T O R I K A
Olimpiade Matematika SMA 2020

Cara pemasangan pada tiang (T1, T2, T3, T4) = (0 , 1 , 2, 4) = 4! = 24 cara

4!
Cara pemasangan pada tiang (T1, T2, T3, T4) = (0 , 1 , 3, 3) = = 12 cara
2!
4!
Cara pemasangan pada tiang (T1, T2, T3, T4) = (0 , 2 , 2, 3) = = 12 cara
2!
4!
Cara pemasangan pada tiang (T1, T2, T3, T4) = (1 , 1 , 1, 4) = = 4 cara
3!
4!
Cara pemasangan pada tiang (T1, T2, T3, T4) = (1 , 1 , 2, 3) = = 12 cara
2!
4!
Cara pemasangan pada tiang (T1, T2, T3, T4) = (1 , 2 , 2, 2) = = 4 cara
3!

Jumlah cara penyusunan = 3 (4) + 7 (12) + 24 = 120 cara


Namun perlu diingat seperti penjelasan di awal bahwa, misal pada bendera
pemasangan (0 , 0 , 0 , 7) , 7 disitu adalah motif yang berbeda dan berlaku pada
cara-cara penyusunan yang lain, sehingga cara memilih motif pada pemasangan
bendera = 7! Cara = 5040 cara
Jadi, banyaknya cara pemasangan bendera adalah 120 . 5040 = 604.800 cara

Untuk nomor 24 jika menggunakan rumus kombinasi, harus diperhatikan bahwa


keempat tiang boleh kosong tanpa bendera, berarti ada nilai 0 disitu. Oleh karena
itu rumus kombinasi yang digunakan untuk bilangan cacah, bukan bilangan asli.

Cara II (dengan rumus kombinasi)


T1, T2, T3, T4 ada 4 tiang bendera, 𝑘 = 4
Benderanya ada 7 terlepas dari motif sama atau berbeda, 𝑛 = 7

𝑛+𝑘−1
Rumus yang dipakai 𝐶𝑘−1 7+4−1
= 𝐶4−1 = 𝐶310

10!
𝐶310 = = 120
7! 3!
Nah karena 7 bendera tadi berbeda motif maka cara menempatkan bendera
tersebut = 7!
banyaknya cara pemasangan bendera 7! 𝐶310 = (5040), (120) = 604.800 cara

124 | K O M B I N A T O R I K A
Olimpiade Matematika SMA 2020

• PRINSIP SARANG MERPATI


(PIGEONHOLE PRINCIPLE)

Prinsip ini hanya digunakan untuk menunjukkan adanya item (obyek) dengan
sifat tertentu, bukan untuk menemukan obyeknya atau banyaknya obyek dengan
sifat yang telah ditentukan.

“sebanyak r merpati masuk ke dalam n sangkar, dengan r > n, maka terdapat


paling sedikit satu sangkar yang terisi dua ekor atau lebih merpati”

Dengan kata lain, Jika ada n sarang burung merpati dan ada n + 1 burung
merpati maka dapat dipastikan bahwa ada 1 sarang yang ditempati lebih dari 1
burung merpati. Dengan asumsi jumlah Burung lebih banyak dari jumlah
sarangnya

Lebih luas lagi prinsip ini dikembangkan menjadi :


Jika n merpati menempati m sarang, maka salah satu sarang akan terisi paling
𝑛−1
sedikit ( + 1) merpati.
𝑚

Sebagai ilustrasi, jika dalam keranjang ada 5 pasang kaos kaki yang berbeda
warna, maka jika diambil 6 kaos kaki, setidaknya ada sepasang kaos kaki yang
sama warnanya.

Contoh Soal 12
Jika ada 15 orang, banyaknya orang yang lahir setidak-tidaknya pada hari yang
sama adalah ....

Pembahasan :
Dalam hal ini n = 15 dan m = 7, maka

𝑛−1 15 − 1
( + 1) = ( + 1) = 3
𝑚 7

Jadi, dari 16 orang setidak-tidaknya 3 orang lahir pada hari yang sama.

125 | K O M B I N A T O R I K A
Olimpiade Matematika SMA 2020

Contoh Soal 13
Dari angka 1 sampai 10 dipilih 3 bilangan. Berapa minimal banyaknya kombinasi
ketiga bilangan yang jumlahnya sama ?

Pembahasan :
Banyaknya cara memilih 3 bilangan dari 10 bilangan adalah 𝐶(10 , 3)= 120.
Jumlah paling kecil adalah 1 + 2 + 3 = 6
Jumlah paling besar adalah 8 + 9 + 10 = 27
Sehingga himpunan jumlah ketiga bilangan yang dipilih adalah {6, 7, 8, … . . 27}
semuanya berjumlah 22 bilangan.

Maka n = 120 , m = 22
𝑛−1 120 − 1
( + 1) = ( + 1)  6
𝑚 22

Ada paling sedikit 6 kombinasi yang berbeda dimana 3 bilangan akan memiliki
jumlah yang sama.

• PRINSIP INKLUSI DAN EKSKLUSI

Pada dasarnya prinsip Inklusi dan eksklusi menggunakan rumusan yang kita
kenal dalam teori Himpunan. Jika diketahui dua himpunan A dan B, maka
banyaknya anggota dari 𝐴 ∪ 𝑩 dinyatakan dengan |𝑨 ∪ 𝑩|, memenuhi persamaan
di bawah ini :

|𝑨 ∪ 𝑩| = |𝑨| + |𝑩| − |𝑨 ∩ 𝑩|

|𝐴| dan |𝐵| = banyaknya anggota himpunan A dan B


|𝐴 ∩ 𝐵| = banyaknya anggota irisan himpunan A dan b

Untuk tiga himpunan A, B dan C maka :

|𝐴 ∪ 𝐵 ∪ 𝐶| = |𝐴| + |𝐵| + |𝐶| − |𝐴 ∩ 𝐵| − |𝐴 ∩ 𝐶| − |𝐵 ∩ 𝐶| + |𝐴 ∩ 𝐵 ∩ 𝐶|

Untuk perhitungan kita mungkin memerlukan menentukan jumlah anggota


komplemen suatu himpunan. Jika menyakan jumlah anggota himpunan A,
maka |𝐴𝑐 | adalah jumlah anggota himpunan komplemen A.
Dengan demikian |𝑆| = |𝐴| + |𝐴𝑐 |

126 | K O M B I N A T O R I K A
Olimpiade Matematika SMA 2020

Contoh Soal 14
Berapa banyaknya bilangan bulat positif yang kurang atau sama dengan 100
yang habis dibagi 3 atau 7 ?

Pembahasan :
Misalkan A adalah himpunan bilangan bulat positif yang kurang atau sama
dengan 100 yang habis dibagi 3 dan B adalah himpunan bilangan bulat positif
yang kurang atau sama dengan 100 yang habis dibagi 7, maka :
A = {3, 6, 9, 12, 15, 18, ..., 99}
B = { 7, 14, 21, 28, ..., 98}
A ∩ B = {21, 42, 63, 84}. Berarti, |𝐴| = 33, |𝐵|=14, dan |𝐴 ∩ 𝐵| = 4, sehingga
|𝐴 ∪ 𝐵| = |𝐴| + |𝐵| − |𝐴 ∩ 𝐵|
|𝐴 ∪ 𝐵|= 33 + 14 – 3 = 44.
Jadi banyaknya bilangan bulat positif yang kurang atau sama dengan 100 yang
habis dibagi 3 atau 7 adalah 44 bilangan.

Contoh Soal 15
Tentukan banyaknya bilangan bulat positif yang kurang atau sama dengan 100
yang habis dibagi 3 atau 4 atau 7 !

Pembahasan :
Misalkan A adalah himpunan bilangan bulat positif yang kurang atau sama
dengan 100 yang habis dibagi 3, maka A = {3, 6, 9, 12, 15, 18, ..., 99}
B adalah himpunan bilangan bulat positif yang kurang atau sama dengan 100
yang habis dibagi 4, maka B = { 4, 8, 12, 16, ..., 100}
C adalah himpunan bilangan bulat positif yang kurang atau sama dengan 100
yang habis dibagi 7, maka C = { 7, 14, 21, 28, ..., 98}
Dapat kita simpulkan bahwa, |𝐴| = 33 , |𝐵| = 25 , |𝐶| = 14
𝐴 ∩ 𝐵 = {12, 24, 36, 48, 60, 72, 84, 96} , |𝐴 ∩ 𝐵| = 8
𝐴 ∩ 𝐶 = {21, 42, 63, 84} , |𝐴 ∩ 𝐶| = 4
𝐵 ∩ 𝐶 = {28, 56, 84} , |𝐵 ∩ 𝐶 | = 3
𝐴 ∩ 𝐵 ∩ 𝐶 = {84} , |𝐴 ∩ 𝐵 ∩ 𝐶| = 1
|𝐴 ∪ 𝐵 ∪ 𝐶| = |𝐴| + |𝐵| + |𝐶| − |𝐴 ∩ 𝐵| − |𝐴 ∩ 𝐶| − |𝐵 ∩ 𝐶| + |𝐴 ∩ 𝐵 ∩ 𝐶|
|𝐴 ∪ 𝐵 ∪ 𝐶| = 33 + 25 + 14 − 8 − 4 − 3 + 1
|𝐴 ∪ 𝐵 ∪ 𝐶| = 58
Jadi banyaknya bilangan bulat positif yang kurang atau sama dengan 100 yang
habis dibagi 3 atau 4 atau 7 adalah 58 bilangan.

127 | K O M B I N A T O R I K A
Olimpiade Matematika SMA 2020

• SEGITIGA PASCAL DAN BINOMIUM NEWTON


Untuk menjabarkan bentuk aljabar dua suku bisa menggunakan segitiga Pascal
sebagai berikut. Beberapa contoh bentuk ekspansi (𝑎 + 𝑏)𝑛 dengan
𝑛 = 0, 1, 2, 3, 4, 5 … ..

(𝑎 + 𝑏)0 = 1
(𝑎 + 𝑏)1 = 𝑎 + 𝑏
(𝑎 + 𝑏)2 = 𝑎2 + 2𝑎𝑏 + 𝑏 2
(𝑎 + 𝑏)3 = 𝑎3 + 3𝑎2 𝑏 + 3𝑎𝑏 2 + 𝑏 3
(𝑎 + 𝑏)4 = 𝑎4 + 4𝑎3 𝑏 + 6𝑎2 𝑏 2 + 4𝑎𝑏 3 + 𝑏 4

Tetapi ada metoda lain dalam mencari koefisien binomial, yaitu dengan
menggunakan konsep kombinasi yang dinamakan Binomial Newton (Ekspansi
Binomial), sehingga hubungannya dengan Segitiga Pascal dapat dinyatakan
sebagai berikut :

(𝑎 + 𝑏)0 = 𝐶00

(𝑎 + 𝑏)1 = 𝐶01 𝑎 + 𝐶11 𝑏

(𝑎 + 𝑏)2 = 𝐶02 𝑎2 + 𝐶12 𝑎𝑏 + 𝐶22 𝑏 2

(𝑎 + 𝑏)3 = 𝐶03 𝑎3 + 𝐶13 𝑎2 𝑏 + 𝐶23 𝑎𝑏 2 + 𝐶33 𝑏 3

(𝑎 + 𝑏)4 = 𝐶04 𝑎4 + 𝐶14 𝑎3 𝑏 + 𝐶24 𝑎2 𝑏 2 + 𝐶34 𝑎𝑏 3 + 𝐶44 𝑏 4


dan seterusnya ….

Secara umum rumusan ekspansi Binomial Newton adalah

(𝒂 + 𝒃)𝒏 = ∑ 𝑪𝒏𝒓 𝒂𝒏−𝒓 𝒃𝒓


𝒓=𝟎

Atau

(𝑎 + 𝑏)𝑛 = 𝐶0𝑛 𝑎𝑛 𝑏0 + 𝐶1𝑛 𝑎𝑛−1 𝑏1 + ⋯ + 𝐶𝑛𝑛 𝑎0 𝑏 𝑛

dengan 𝑛 , 𝑟 bilangan-bilangan Asli

128 | K O M B I N A T O R I K A
Olimpiade Matematika SMA 2020

Contoh Soal 16
Jabarkan bentuk binomial dari (3𝑥 − 2𝑦)3
Pembahasan :
3

(3𝑥 − 2𝑦) = ∑ 𝐶𝑟3 (3𝑥)3−𝑟 (−2𝑦)𝑟


3

𝑟=0

Penjabaran (3𝑥 − 2𝑦)3 adalah

𝐶03 (3𝑥)3 + 𝐶13 (3𝑥)2 (−2𝑦) + 𝐶23 (3𝑥)(−2𝑦)2 + 𝐶33 (−2𝑦)3


(3𝑥 − 2𝑦)3 = 27𝑥 3 − 54𝑥 2 𝑦 + 36𝑥𝑦 2 − 8𝑦 3

Untuk menentukan suku ke 𝑘 pada penjabaran (𝑎 + 𝑏)𝑛 dengan rumus

𝑪𝒏𝒌−𝟏 𝒂𝒏−(𝒌−𝟏) 𝒃𝒌−𝟏

Contoh Soal 17
Tentukan koefisien dan suku ke-3 dari penjabaran (2𝑥 − 5𝑦)10
Pembahasan :
𝑛
𝐶𝑘−1 𝑎𝑛−(𝑘−1) 𝑏 𝑘−1 , 𝑎 = 2𝑥, 𝑏 = −5𝑦, 𝑛 = 10 , 𝑘 = 3
Suku ke-3 dari (2𝑥 − 5𝑦)10 adalah 𝐶210 (2𝑥)8 (−5𝑦)2
10 !
28 𝑥 8 (−5)2 𝑦 2 = 288.000 𝑥 8 𝑦 2
(10−2)! 2 !
Jadi koefisien suku ke-3 adalah 288.000 dan suku ke-3 itu adalah 288.000 𝑥 8 𝑦 2

Contoh Soal 18
Tentukan jumlah semua koefisien dari penjabaran (2𝑥 + 3𝑦 − 4𝑧)4 (𝑥 + 2𝑦 − 𝑧)2

Pembahasan :
Untuk memudahkan menentukan jumlah semua koefisien pada penjabaran
suku-suku pada soal diatas, maka akan kita gunakan pangkat dari koefisien
sebagai berikut :
Jumlah kefisien 2𝑥 + 3𝑦 − 4𝑧 = 2 + 3 – 4 = 2
Jumlah koefisien 𝑥 + 2𝑦 − 𝑧 = 1 + 2 – 1 = 2
Maka jumlah semua koefisien dari penjabaran
(2𝑥 + 3𝑦 − 4𝑧)4 (𝑥 + 2𝑦 − 𝑧)2 = (2)4 (2)2 = 64

129 | K O M B I N A T O R I K A
Olimpiade Matematika SMA 2020

• PELUANG KEJADIAN

Ruang Sampel adalah himpunan dari semua hasil percobaan yang mungkin.
Ruang Sampel biasanya dilambangkan dengan S yang dalam teori himpunan
disebut dengan himpunan Semesta.
Kejadian adalah himpunan bagian dari Ruang Sample yang dapat berupa kejadian
sederhana maupun kejadian majemuk. Kejadian sederhana adalah suatu kejadian
yang hanya mempunyai sebuah titik kejadian. Jika suatu kejadian memiliki lebih
dari satu titik kejadian disebut dengan kejadian majemuk.

Jika kejadian A dapat terjadi dalam k cara atau n(A) dari seluruh n cara n(S) yang
mungkin, dimana semua kejadian ini mempunyai kemungkinan yang sama,
maka peluang terjadinya kejadian A, ditulis P(A), adalah

𝒏(𝑨) 𝒌
𝑷(𝑨) = =
𝒏(𝑺) 𝒏

Jika peluang suatu kejadian bernilai 0 maka artinya kejadian tersebut tidak
mungkin terjadi sedangkan jika peluang suatu kejadian bernilai 1 artinya kejadian
tersebut pasti terjadi. Peluang suatu kejadian akan berkisar 0 ≤ p(A) ≤ 1. jika
peluang terjadinya kejadian A adalah P(A), maka peluang tidak terjadinya
kejadian A adalah 1 – P(A).

Jenis kejadian dalam teori peluang :

Jika A dan B dua kejadian yang tidak saling lepas maka berlaku

P(A  B) = P(A ) + P(B) − P(A  B)

Kejadian saling lepas, jika A  B =  maka P(A  B) = 0, sehingga

P(A  B) = P(A ) + P(B)

Dua kejadian A dan B dikatakan saling bebas jika dan hanya jika
P(A  B) = P(A ) P(B)

130 | K O M B I N A T O R I K A
Olimpiade Matematika SMA 2020

Contoh Soal 19
Sebuah kotak berisi 3 kelereng merah, 4 kelereng hijau dan 5 kelereng biru. Jika
dari dalam kotak diambil 3 buah kelereng, maka berapa peluang :
a. semua kelereng yang terambil berwarna merah
b. 2 buah berwarna merah dan 1 berwarna hijau
c. 1 berwarna merah, 1 hijau dan 1 biru

Pembahasan :
Jumlah seluruh kelereng ada 12.
Banyaknya cara memilih 3 dari 12 kelereng adalah 𝐶312 = 220 , maka n(S) = 220

a. Banyaknya cara memilih 3 kelereng merah dari 3 kelereng merah adalah 𝐶33 =
1 , maka n(3m) = 1
Peluang 3 kelereng yang terambil semuanya merah adalah ...
𝑛(3𝑚) 1
𝑃= =
𝑛(𝑆) 220

b. 𝑛(2𝑚, 1ℎ) = 𝐶23 𝐶14 = 3 . 4 = 12


Peluang terambil 2 buah berwarna merah dan 1 berwarna hijau adalah ...
𝑛(2𝑚 , 1ℎ) 12
𝑃= =
𝑛(𝑆) 220

c. 𝑛(1𝑚, 1ℎ, 1𝑏) = 𝐶13 𝐶14 𝐶15 = 3 . 4 . 5 = 60


Peluang terambil nya 1 berwarna merah, 1 hijau dan 1 biru adalah ...

𝑛(1𝑚 , 1ℎ , 1𝑏) 60
𝑃= =
𝑛(𝑆) 220

Contoh Soal 20
Masing-masing satu huruf diambil dari kata “SEHAT” dan “BUGAR”.
Berapakah peluang bahwa kedua huruf tersebut terdiri dari satu vokal dan satu
konsonan ?

Pembahasan :
Kemungkinannya adalah satu vokal dari kata “SEHAT” dan satu konsonan dari
kata “BUGAR” atau satu konsonan dari kata “SEHAT” dan satu vokal dari kata
“BUGAR”.

131 | K O M B I N A T O R I K A
Olimpiade Matematika SMA 2020
2 3 6
Peluang kasus pertama = x =
5 5 25

3 2 6
Peluang kasus kedua = x =
5 5 25

6 6 12
Maka peluang kejadian tersebut adalah + =
25 25 25

Contoh Soal 21
Sejumlah n siswa duduk mengelilingi suatu meja bundar. Diketahui siswa laki-
laki sama banyak dengan siswa perempuan. Jika banyaknya pasangan 2 orang
yang duduk bersebelahan dihitung, ternyata perbandingan antara pasangan
bersebelahan yang berjenis kelamin sama dan pasangan bersebelahan yang
berjenis kelamin berbeda dalah 3 : 2. Nilai n terkecil yang mungkin adalah ...

Pembahasan :
Misalkan x adalah banyaknya pasangan
bersebelahan yang berjenis kelamin sama dan y
adalah banyaknya pasangan bersebelahan yang
berjenis kelamin berbeda, dimana x : y = 3 : 2.
Karena diketahui pula bahwa siswa laki-laki sama
banyak dengan siswa perempuan, maka n adalah
bilangan genap.
Nilai n terkecil dapat diasumsikan sebagai pengali
jumlah minimal siswa yang ada di meja bundar.
Seningga x : y = 3 : 2 dapat diasumsikan
sebagai x : y = 6 : 4 ,
sehingga nilai n terkecil adalah 6 + 4 = 10

Contoh Soal 22
Dalam sebuah kotak terdapat 4 bola merah dan 5 bola biru. Berapakah peluang
jika diambil dua bola satu persatu dengan pengembalian dengan bola pertama
berwarna merah dan bola kedua berwarna biru, jika :
a. Dengan pengembalian
b. Tanpa pengembalian

132 | K O M B I N A T O R I K A
Olimpiade Matematika SMA 2020

Pembahasan :
a. Dengan pengembalian
4
Bola pertama yang terambil adalah bola merah, peluangnya adalah 𝑃(𝑚) = 9
Setelah dikembalikan, jumlah bola tetap 9 dengan bola biru tetap 4 buah.
5
Maka peluang pengambilan bola kedua adalah bola biru adalah 𝑃(𝑏) = 9
Maka peluang pengambilan kedua bola dengan cara ini adalah

4 5 20
𝑃 = 𝑃(𝑚)𝑥 𝑃(𝑏) = 𝑥 =
9 9 81

b. Tanpa pengembalian
Bola pertama yang terambil adalah bola merah maka peluangnya adalah
4
𝑃(𝑚) = 9
Bola merah dianggap tinggal tiga dan bola biru tetap 5 sehinggga bola di
keranjang ada 8 buah
5
Maka peluang pengambilan bola kedua adalah bola biru adalah 𝑃(𝑏) = 8
Maka peluang pengambilan kedua bola dengan cara ini adalah

4 5 5
𝑃 = 𝑃(𝑚)𝑥 𝑃(𝑏) = 𝑥 =
9 8 18

Contoh Soal 23
Ani dan Banu bermain dadu 6 sisi. Jika dadu yang keluar bernilai genap, maka
Ani mendapatkan skor 1, namun jika dadu yang keluar ganjil, Banu
mendapatkan skor 1. Pemenang dari permainan ini adalah orang yang pertama
mendapat skor 5. Setelah dilakukan pelemparan dadu sebanyak 5 kali, Ani
mendapatkan skor 4 dan Banu mendapatkan skor 1. Peluang Ani memenangkan
permainan ini adalah ....

Pembahasan :
Setelah 5 kali pelemparan, skor Ani 4 dan skor Banu 1. Artinya untuk
mendapatkan kemenagan yang adil bagi keduanya, paling tidak ada 4 lagi
permainan, sehingga jumlah permainan minimal 9 kali.

1 1
Peluang Ani dan Banu menang adalah sama, yaitu 𝑃(𝑎) = 2 dan 𝑃(𝑏) = 2.

133 | K O M B I N A T O R I K A
Olimpiade Matematika SMA 2020

Ani akan menang jika setidaknya ada 1 kali muncul dadu genap pada
pelemparan 4 kali berikutnya, atau dengan kata lain tidak muncul dadu ganjil
selama 4 pelemparan tersebut.

Jika banyaknya pelemparan 4, jumlah keluar angka ganjil = 0, maka peluang


tidak ada dadu ganjil yang keluar dalam 4 kali pelemparan adalah

1 0 1 4 1
𝑃 = 𝐶04 ( ) ( ) =
2 2 16

1 15
Maka peluang Ani menang = 1 − =
16 16

Contoh Soal 24
Dari 4 pria dan 3 wanita, mereka duduk berjajar. Tentukan peluang cara duduk
mereka jika semua wanita duduk berdampingan !

Pembahasan :

Dari 4 pria dan 3 wanita, mereka duduk berjajar. Agar semua wanita duduk
berdampingan, maka cara duduk wanita ada 3! cara, atau 6 cara

Kelompok wanita tersebut dengan 4 pria berjajar seolah menjadi 5 obyek, maka
banyaknya 5 obyek berjajar adalah 5! Cara, atau 120 cara.

Jadi banyaknya cara mereka duduk dengan para wanita berdampingan adalah 6 x
120 cara atau 720 cara.

Untuk menentukan banyaknya ruang sampel berarti ke tujuh orang itu duduk
berjajar dengan 7 ! cara atau 7 x 6! , sehingga Peluang mereka duduk sesuai
persyaratan tadi adalah :
𝑛(𝐴) 720 1
𝑃= = =
𝑛(𝑆) 7 . 6! 7

134 | K O M B I N A T O R I K A
Olimpiade Matematika SMA 2020

Contoh Soal 25
Nomor kendaraan terdiri dari 4 buah angka yang berbeda. Berapa peluang
sebuah kendaraan mempunyai nomor tidak terdiri dari angka-angka yang
terurut.

Pembahasan :
Ruang sampel pada adalah himpunan dari semua permutasi 4 dari 10, jadi
10 ! 10 !
𝑛(𝑆) = = = 5040
(10 − 4)! 6!
Misalkan A adalah kejadian dari kendaraan mempunyai nomor dari angka-
angka yang terurut, jadi
A =  0123, 1234, 2345, 3456, 4567, 5678, 6789 

n(A) 7 1
Jadi P(A) = = =
n(S) 5040 720

Akibatnya peluang dari sebuah kendaraan mempunyai nomor tidak terdiri dari
angka-angka yang terurut adalah

𝑃(𝐴𝑐 ) = 1 − 𝑃(𝐴)
1
𝑃(𝐴𝑐 ) = 1 −
720
719
𝑃(𝐴𝑐 ) =
720

135 | K O M B I N A T O R I K A
Olimpiade Matematika SMA 2020

Problem Solving
KOMBINATORIKA

Problem 01
Bilangan bulat positif 2013 dan 3210 memiliki tiga sifat sebagai berikut :
(i) Keduanya terletak diantara 1000 dan 10.000
(ii) Keempat digitnya adalah bilangan bulat berurutan,
(iii) Kedua bilangan habis dibagi 3
Berapa banyak bilangan bulat positif yang memiliki ketiga sifat tersebut ?

Pembahasan :
Bilangan bulat positif dapat dibagi 3 jika jumlah digitnya habis dibagi 3. Karena
jumlah digit tidak bergantung urutan, maka penataan ulang digit bilangan bulat
positif yang dapat dibagi 3 menghasilkan bilangan bulat positif lain yang dapat
dibagi oleh 3 juga.

Perhatikan bahwa 10.000 tidak dapat dibagi dengan 3. Setiap bilangan bulat
positif lainnya antara 1000 dan 10.000 adalah bilangan bulat empat digit.

Dengan cara mendaftar, kita dapat menuliskan bilangan bulat ini dalam urutan
menurun, seperti : 3210, 4321, 5432, 6543, 7654, 8765, dan 9876.
Jumlah digit bilangan bulat ini masing-masing adalah 6, 10, 14, 18, 22, 26, dan 30.
Dari jumlah tersebut, 6, 18 dan 30 adalah satu-satunya jumlah yang dapat dibagi
oleh 3, jadi hanya 3210, 6543 dan 9876 yang dapat dibagi 3.

Karena mengatur ulang digit tidak berpengaruh, maka :


Ada 24 bilangan yang didapat dari penataan ulang 6543. (4 ! = 24 cara) dan
Ada 24 bilangan yang didapat dari penataan ulang 9876. (4 ! = 24 cara)

Sedangkan untuk bilangan 3210 terdapat angka 0 dan angka 0 ini tentu tidak
boleh ditempatkan sebagai bilangan ribuan. Jadi ada 18 bilangan yang didapat
dari penataan ulang 3210. Karena jika 0 bilangan ribuan maka ada 6 cara
meletakan bilangan 3, 2 dan 1.
Secara total, ada 24 + 24 + 18 = 66 bilangan bulat positif dengan tiga sifat ini.

136 | K O M B I N A T O R I K A
Olimpiade Matematika SMA 2020

Problem 02
Ada berapa banyak bilangan 3 digit berbeda masing-masing dalam urutan naik
dan dalam urutan turun ?.

Pembahasan :
Misalnya posisi 3 digit bilangan tersebut dengan kotak penempatan di bawah ini
ratusan, puluhan dan satuan berikut ini :

Dengan urutan Naik :


Kotak pertama boleh diisi dengan 1, 2, 3, 4, 5, 6 dan 7 (ada 7 cara)
Setelah kotak pertama ditetapkan maka yang berikutnya dapat ditempatkan
bilangan kedua dan ketiga secara didaftar seperti berikut :
Contoh :
1 2 3
2 3 4
3 4 5
4 5 6
5 6 7
6 7 8
7 8 9

Jadi ada (7)(7)(7) cara = 343 cara

Dengan urutan Turun :


Kotak pertama boleh diisi dengan 9, 8, 7, 6, 5, 4, 3 dan 2 (ada 8 cara)
Setelah kotak pertama ditetapkan maka yang berikutnya dapat ditempatkan
bilangan kedua dan ketiga secara didaftar seperti berikut :

9 8 7
8 7 6
7 6 5
6 5 4
5 4 3
4 3 2
3 2 1
2 1 0

Jadi (8)(8)(8) cara = 512 cara

137 | K O M B I N A T O R I K A
Olimpiade Matematika SMA 2020

Problem 03
Sebuah keluarga besar beranggotakan 14 orang anak yang terdiri dari dua
kelahiran kembar tiga identik, tiga kelahiran dua identik, dan dua anak yang
lain. Bila kembar identik tak dapat dibedakan, maka banyak pose foto berdiri
dalam satu baris pada 14 anak tersebut adalah …

Pembahasan :
Kasus ini ekuivalen dengan kasus penyusunan kata yang mengandung sejumlah
huruf yang sama. Gunakan permutasi berulang untuk kasus ini, yaitu
14!
(3!)2 . (2!)3

Problem 04
Diberikan persamaan 𝑥1 + 𝑥2 + 𝑥3 + 𝑥4 = 12, dengan 𝑥𝑖 adalah bilangan cacah.
Banyaknya solusi x yang mungkin adalah .....
Pembahasan :
Ini merupakan kasus kombinasi dengan pengulangan, di mana n = 4 dan r = 12.
𝑥1 + 𝑥2 + 𝑥3 + 𝑥4 = 12
Seluruh kemungkinan yang ada adalah
4+12−1 15
15!
𝐶𝑟𝑛+𝑟−1 = 𝐶12 = 𝐶12 = = 455
15! . 3!

Problem 05
Ada berapa cara membentuk tim terdiri dari 2 laki-laki dan 2 wanita yang
diambil dari 8 orang laki-laki dan 6 orang wanita ?

Pembahasan :
Banyaknya cara memilih 2 laki-laki dari 8 laki-laki adalah
8! 8!
𝐶28 = = = 28
(8 − 2)! 2! 6! 2!
Banyaknya cara memilih 2 wanita dari 6 wanita adalah
6! 6!
𝐶26 = = = 15
(6 − 2)! 2! 4 ! 2 !
Dengan kaidah perkalian, maka banyaknya cara membentuk tim terdiri dari 2
laki-laki dan 2 wanita yang diambil dari 8 orang laki-laki dan 6 orang wanita
adalah 28 x 15 = 420 cara.

138 | K O M B I N A T O R I K A
Olimpiade Matematika SMA 2020

Problem 06
Banyak cara menugaskan 5 pekerjaan berbeda kepada 4 orang pegawai berbeda
sedemikian sehingga setiap pegawai ditugaskan ke paling sedikit satu pekerjaan
adalah …

Pembahasan :
Berdasarkan syarat yang diberikan, akan ada 1 pegawai yang mendapat 2
pekerjaan, sedangkan 3 pegawai lainnya mendapatkan 1 pekerjaan. Kondisinya
diberikan oleh tabel di bawah di mana angka-angkanya mewakili banyak
pekerjaan yang diambil.
Pegawai 1 Pegawai 2 Pegawai 3 Pegawai 4
2 1 1 1
1 2 1 1
1 1 2 1
1 1 1 2
Untuk kondisi pertama, pegawai 1 mendapatkan 2 pekerjaan, sedangkan 3
pegawai lainnya mendapatkan 1 pekerjaan saja. Untuk pegawai 1, banyak cara ia
memilih 2 dari 5 pekerjaan yang tersedia adalah C(5, 2) = 10 cara.
Tersisa 3 pekerjaan untuk 3 orang pegawai lainnya. Banyak cara pemberian
pekerjaan ini dapat dihitung dengan aturan permutasi, yaitu P(3, 3) = 3! = 6 cara.
Banyak cara untuk kondisi pertama adalah 10 x 6 = 60 cara.
Karena ada 4 kondisi berbeda, banyak cara seluruhnya ada 4 x 60 = 240 cara.

Problem 07
Jika terdapat 8 siswa dan 7 siswi maka ada berapa cara membentuk panitia
beranggotakan 7 orang dengan syarat sedikitnya 5 siswi harus masuk dalam
kepanitiaan tersebut ?

Pembahasan :
Karena sedikitnya 7 orang panitia tersebut terdiri dari 5 siswi, maka akan ada
tiga kasus dalam persoalan ini yaitu panitia terdiri dari 5 siswi dan 2 siswa atau 6
siswi dan 1 siswa atau semuanya siswi.
Banyaknya susunan kasus pertama 𝐶57 𝑥 𝐶28 = 588

Banyaknya susunan kasus kedua adalah 𝐶67 𝑥 𝐶18 = 56

Banyaknya susunan kasus ketiga adalah 𝐶77 𝑥 𝐶08 = 1


Maka banyaknya cara membentuk panitia dengan susunan seperti itu adalah
588 + 56 + 1 = 645.

139 | K O M B I N A T O R I K A
Olimpiade Matematika SMA 2020

Problem 08
Banyak cara mengisi persegi panjang berukuran 2 x 16 dengan persegi panjang
berukuran 2 x 2, 2 x 3 dan 2 x 4 adalah ....

Pembahasan :
Karena setiap persegi panjang yang diberikan memiliki ukuran panjang yang
sama, yaitu 2, maka kita hanya perlu meninjau ukuran lebarnya. Jika a,b dan c
masing-masing ukuran lebar persegipanjang yang mungkin, maka untuk mengisi
persegi panjang berukuran 2 x 16 tersebut, kita perlu menentukan nilai a, b, c
sedemikian sehingga persamaan berikut berlaku
2a + 3b + 4c = 16
Dengan cara mendaftar, maka kemungkinan nilai a, b dan c adalah

a b c N
0 0 4 1
2 0 3 5!
= 10
2! .3!
1 2 2 5!
= 30
2! .2!
4 0 2 6!
= 15
4! .2!
0 4 1 5!
=5
4!
3 2 1 6!
= 60
3! .2!
6 0 1 7!
=7
6!
2 4 0 5!
= 15
2! .3!
5 2 0 7!
= 21
5! .2!
8 0 0 1

Jadi, ada 10 cara mengisi persegi panjang tersebut. Tetapi, perlu diperhatikan
bahwa penempatan urutan nilai a,b,c (mewakili persegi panjang dengan ukuran
yang disebutkan pada soal) juga mengakibatkan perbedaan cara pengisiannya.
Banyaknya cara 1 + 10 + 30 + 15 + 5 + 60 + 7 + 15 + 21 + 1 = 165

140 | K O M B I N A T O R I K A
Olimpiade Matematika SMA 2020

Problem 09
Dalam berapa banyak cara kita dapat menulis angka 12 sebagai jumlah dari tiga
bilangan bulat positif ? anggap, 1 + 1 + 10 berbeda dari 1 + 10 + 1.

Pembahasan :
Menurut teorema De Moivre, Misalkan n bilangan bulat positif. Jumlah solusi
𝑛−1
bilangan bulat positif untuk x1 + x2 + · · · + xr = n adalah 𝐶𝑟−1
Pada persoalan ini tiga bilangan misalnya a > 0 , b > 0 dan c > 0 sehingga jumlah
ketiga bilangan a + b + c = 12
𝑛−1 12−1
Maka banyaknya cara atau kombinasi a, b, c adalah 𝐶𝑟−1 = 𝐶3−1 = 𝐶211 = 55

Problem 10
Penyusunan nomer telepon dengan 7 digit menggunakan 0, 1, 2, 3, 4, 5, 6 dan 7
dengan tidak ada nomer telepon yang diijinkan menggunakan awal 0, 1 atau 5.
Tentukan banyaknya nomer telepon yang mungkin dengan digit-digit tidak
boleh diulang dan nomer telepon harus ganjil.

Pembahasan :
Hal ini dilakukan dengan memisahkan didepan genap dan didepan ganjil.
Jika didepan genap maka diperoleh

3 6 5 4 3 2 4

bilangan-bilangan pada kotak adalah banyaknya digit yang mungkin dapat


mengisi. Jika didepan genap maka diperoleh = 3 x 6 x 5 x 4 x 3 x 2 x 4 = 8640

Jika didepan ganjil, dari 4 macam bilangan ganjil 1 dan 5 tidak boleh, maka
didepan hanya ada 2 macam digit, yaitu 3 atau 7.
Sedangkan jika pada digit terakhir karena harus ganjil dan tidak boleh diulang.
Jadi jika sudah digunakan didepan tidak bisa digunakan dibelakang. Maka
kemungkinan digit terakhir hanya ada 3 macam, yaitu (1, 5, 7) atau (1, 3, 5).

Diperoleh :

2 6 5 4 3 2 3

Jika didepan ganjil maka diperoleh = 2 x 6 x 5 x 4 x 3 x 2 x 3 = 4320


banyaknya nomer telepon yang mungkin dengan digit-digit tidak boleh diulang
dan nomer telepon harus ganjil adalah 12960

141 | K O M B I N A T O R I K A
Olimpiade Matematika SMA 2020

Problem 11
Terdapat banyak kartu bergambar dengan perincian kartu-kartu itu sebagai
berikut :
1 kartu bergambar harimau
2 kartu bergambar gajah
3 kartu bergambar ular
4 kartu bergambar kerbau
Dan seterusnya samppai 50 kartu dengan gambar hewan yang berbeda-beda.
Semua kartu ber jumlah 1 + 2 + 3 + 4 + ...... + 49 + 50 dimasukan dalam kotak
besar. Berapa buah kartu minimal harus diambil agar dapat dipastikan terdapat
sekurang-kurangnya 10 kartu dengan gambar hewan yang sama ?

Pembahasan :
Soal ini tentang prinsip sarang merpati. Jika ada n sarang burung merpati dan
ada n + 1 burung merpati maka dapat dipastikan bahwa ada 1 sarang yang
ditempati lebih dari 1 burung merpati. Dengan asumsi jumlah Burung lebih
banyak dari jumlah sarangnya

Dalam hal ini, Sarang ke-1 berisi 1 burung, sarang ke-2 berisi 2 burung, sampai
dengan sarang ke-50 berisi 50 burung.

mula-mula kita ambil kartu bergambar urutan 1 sampai 9 (anggap saja sarang
yang pertama sampai sarang urutan gambar ke Sembilan),
sehingga totalnya ada 1 + 2+ 3 + 4 + ...... + 8 + 9 = 45 kartu
Kita belum mendapatkan kartu yang bertanda gambar hewan ke-10 atau lebih.

Kartu bergambar (dalam kotak/Sarang) yang masih tersisa adalah gambar


hewan ke-10 sampai gambar hewan ke-50, sehingga masih ada sekitar 41 jenis
kartu bergambar yang berbeda.

Sesuai dengan pertanyaan maka kartu yang dibutuhkan sehingga yang terambil
sekurang-kurangnya 10 kartu dengan gambar hewan yang sama adalah

(10 − 1) 𝑥 41 + 1 = 370

Sehingga total kartu bergambar yang perlu diambil 45 +370 = 415 kartu

142 | K O M B I N A T O R I K A
Olimpiade Matematika SMA 2020

Problem 12
Jika (a , b) bulat dipilih dari himpunan {5, 6, 7, 8, … … . . ,49}, maka peluang
1 1 1
mendapatkankan pasangan (a , b) yang merupakan solusi dari + = adalah
𝑎 𝑏 6

Pembahasan :
1 1 1
+ =
𝑎 𝑏 6
𝑎+𝑏 1
=
𝑎𝑏 6
𝑎𝑏 − 6𝑎 − 6𝑏 = 0
𝑎𝑏 − 6𝑎 − 6𝑏 + 36 = 36
𝑎(𝑏 − 6) − 6(𝑏 − 6) = 36
(𝑏 − 6)(𝑎 − 6) = 36
(𝑏 − 6) dan (𝑎 − 6) adalah faktor dari 36
36 = 22 𝑥 32

Banyaknya pasangan (𝑎 , 𝑏)= (2 +1) (2 + 1) = 9, yaitu : (7 , 42) ; (8 , 24) ; (9 , 18) ;


(10 , 15) ; (12 , 12) ; (15 , 10) ; (18 , 9) ; (24 , 8) dan (42 , 7).
Akan tetapi (a , b) yang diperbolehkan adalah {5, 6, 7, 8, … … . . ,49} maka
banyaknya pasangan (a , b) yang menjadi ruang sampel atau himpunan semesta
adalah 𝑛(𝑆) = 45 x 44 , ada pemakaian (a , b) = (12 , 12) dua kali.

Jika A adalah banyaknya kejadian munculnya pasangan (a , b) yang merupakan


1 1 1 𝑛(𝐴)
solusi bulat dari + = , maka peluang munculnya A adalah 𝑃 =
𝑎 𝑏 6 𝑛(𝑆)
9 1
𝑃= =
45 𝑥 44 220

Problem 13
Sebuah dadu bersisi 6 tak beraturan, setiap sisinya diberi nomor 1, 2, 3, 4, 5 dan 6.
Jika dadu tersebut dilempar maka dadu akan jatuh pada salah satu sisinya. Jika
𝑃(𝑛) adalah nilai peluang benda tersebut jatuh pada sisi bernomor n dan
𝑥
𝑃(𝑛) = maka 𝑥 adalah ...
2𝑛−1

Pembahasan :
𝑃(𝑛) adalah nilai peluang benda tersebut jatuh pada sisi bernomor n
𝑥
dan 𝑃(𝑛) =
2𝑛−1

𝑃(1) + 𝑃(2) + 𝑃(3) + 𝑃(4) + 𝑃(5) + 𝑃(6) = 1

143 | K O M B I N A T O R I K A
Olimpiade Matematika SMA 2020
𝑥 𝑥 𝑥 𝑥 𝑥 𝑥
+ + + + + =1
21−1 22−1 23−1 24−1 25−1 26−1
𝑥 𝑥 𝑥 𝑥 𝑥 𝑥
0
+ 1+ 2+ 3+ 4+ 5=1
2 2 2 2 2 2

𝑥 𝑥 𝑥 𝑥 𝑥 32𝑥 + 16𝑥 + 8𝑥 + 4𝑥 + 2𝑥 + 𝑥
𝑥+ + + + + = 1 𝑎𝑡𝑎𝑢 =1
2 4 8 16 32 32
63𝑥 32
= 1 Jadi nilai 𝑥 =
32 63

Problem 14
Dari 7 orang yang terdiri dari 4 pria dan 3 wanita, akan dipilih 3 orang untuk
menjadi ketua, sekretaris dan bendahara. Peluang terpilihnya ketua pria atau
sekretaris wanita adalah ..

Pembahasan :
Dari 7 orang yang terdiri dari 4 pria dan 3 wanita,
Banyaknya susunan dimana pria sebagai ketua, n(A) = 4 . 6 . 5 = 120 cara
Perhatikan disini, cara susunan pengurus dipilih berdasarkan satu pengurus saja,
yaitu ketua (sekretaris dan bendahara bisa pria atau wanita)
Banyaknya susunan, wanita sebagai sekretaris adalah n(B) = 5 . 6 . 3 = 90 cara
Perhatikan disini, cara susunan pengurus dipilih berdasarkan satu pengurus saja,
yaitu Sekretaris (ketua dan bendahara bisa pria atau wanita)
Banyaknya susunan dimana ketua pria dan sekretaris wanita adalah
n(A  B) = 4 . 5 . 3 = 60 cara

Maka Banyaknya susunan pengurus dimana ketua pria atau sekretaris wanita
adalah
n(A  B) = n (A) + n (B) – n(A  B)
n(A  B) = 120 + 90 – 60 = 150

Ruang sampel atau banyaknya cara yang mungkin menyusun 3 dari 7 orang
untuk ditempatkan sebagai pengurus adalah n (S) = 7 . 6 . 5 = 210
Maka peluang terpilihnya ketua pria atau sekretaris wanita adalah P(A  B)

𝑛(𝐴𝐵) 150 5
𝑃(𝐴𝐵) = = =
𝑛(𝑆) 210 7

144 | K O M B I N A T O R I K A
Olimpiade Matematika SMA 2020

Problem 15
Diberikan satu koin yang tidak seimbang. Bila koin tersebut ditos satu kali,
1
peluang muncul angka adalah . Jika ditos n kali, peluang muncul tepat dua
4
angka sama dengan peluang muncul tepat tiga angka. Nilai n adalah ...

Pembahasan :
1
Diketahui peluang muncul angka adalah dan peluang muncul gambar
4
3
adalah . Jika dadu ditos n kali, maka
4
1 2 3 𝑛−2
• peluang muncul tepat dua angka yaitu 𝐶2𝑛 ( ) ( )
4 4

1 3 3 𝑛−3
• peluang muncul tepat tiga angka yaitu 𝐶3𝑛 ( ) ( )
4 4

1 2 3 𝑛−2 1 3 3 𝑛−3
sedemikian sehingga 𝐶2𝑛 (4) (4) = 𝐶3𝑛 (4) (4)

𝑛! 1 2 3 𝑛 4 2 𝑛! 1 3 3 𝑛 4 3
( ) ( ) ( ) = ( ) ( ) ( )
(𝑛 − 2) ! 2 ! 4 4 3 (𝑛 − 3) ! 3 ! 4 4 3

𝑛 (𝑛 − 1)(𝑛 − 2)! 𝑛(𝑛 − 1)(𝑛 − 2) (𝑛 − 3)! 1 4


= ( ) ( )
(𝑛 − 2) ! 2 (𝑛 − 3) ! 6 4 3

1 (𝑛 − 2) 1
= ( ) 𝑑𝑎𝑛 𝑑𝑖𝑝𝑒𝑟𝑜𝑙𝑒ℎ, 𝑛 = 11
2 6 3

𝑛 + 𝑚 = 11 dan misalkan 𝑛 = 5 dan 𝑚 = 6 , maka nilai terkecil dari :

𝑛2 + 𝑚2 = (𝑛 + 𝑚)2 − 2𝑛𝑚
𝑛2 + 𝑚2 = 112 − 2. 5. 6 = 121 − 60 = 61

145 | K O M B I N A T O R I K A
Olimpiade Matematika SMA 2020

Problem 16
Sebuah dadu dilempar 2 (dua) kali. Misalnya 𝑎 dan 𝑏 berturut turut adalah
angka yang muncul pada pelemparan pertama dan kedua. Besarnya peluang
terdapat bilangan real x, y dan z yang memenuhi persamaan :
𝑥 + 𝑦 + 𝑧 = 𝑎 dan 𝑥 2 + 𝑦 2 + 𝑧 2 = 𝑏 sebesar ....

Pembahasan :
𝑥+𝑦+𝑧 =𝑎
(𝑥 + 𝑦 + 𝑧)2 = 𝑎2
𝑥 2 + 𝑦 2 + 𝑧 2 + 2(𝑥𝑦 + 𝑦𝑧 + 𝑥𝑧) = 𝑎2
𝑏 + 2(𝑥𝑦 + 𝑦𝑧 + 𝑥𝑧) = 𝑎2
Karena 𝑥 2 + 𝑦 2 + 𝑧 2 ≥ 𝑥𝑦 + 𝑦𝑧 + 𝑥𝑧 maka
𝑏 + 2𝑏 ≥ 𝑎2
𝑎2 ≤ 3𝑏
Maka dengan cara mendaftar anggota (𝑎, 𝑏) = {1, 2, 3, 4, 5, 6} diperoleh
Jika 𝑎 = 1 maka b = 1, 2, 3, 4, 5, 6 ada 6 pasang (𝑎, 𝑏)
Jika 𝑎 = 2 maka b = 2, 3, 4, 5, 6 ada 5 pasang (𝑎, 𝑏)
Jika 𝑎 = 3 maka b = 3, 4, 5, 6 ada 4 pasang (𝑎, 𝑏)
Jika 𝑎 = 4 maka b = 6 ada 1 pasang (𝑎, 𝑏)
Jadi seluruhnya ada 16 kemungkinan pasangangan (𝑎, 𝑏)
Besarnya peluang terdapat bilangan real x, y dan z yang memenuhi adalah

𝑛(𝑎, 𝑏) 16 4
𝑃= = =
𝑛(𝑆) 36 9

Problem 17
Berapakah koefisien 𝑥 3 pada ekspansi (1 + 𝑥 + 𝑥 2 + 𝑥 3 )12 ?
Pembahasan :
(1 + 𝑥 + 𝑥 2 + 𝑥 3 )12 = 𝐶𝑚
12 (1)12−𝑚 (𝑥
+ 𝑥 2 + 𝑥 3 )𝑚 , 0 ≤ 𝑚 ≤ 12

(1 + 𝑥 + 𝑥 2 + 𝑥 3 )12 = 𝐶𝑚
12 𝑚 (𝑥)𝑚−𝑠 (𝑥 2
𝐶𝑠 + 𝑥 3 )𝑠 , 0 ≤ 𝑠 ≤ 𝑚 ≤ 12
12 𝑚 𝑠 (𝑥)𝑚−𝑠 (𝑥 2 ) 𝑠−𝑡 (𝑥 3 )𝑡
(1 + 𝑥 + 𝑥 2 + 𝑥 3 )12 = 𝐶𝑚 𝐶𝑠 𝐶𝑡 , 0 ≤ 𝑡 ≤ 𝑠 ≤ 𝑚 ≤ 12

Maka 𝑚 − 𝑠 + 2(𝑠 − 𝑡) + 3𝑡 = 3 atau 𝑚 + 𝑠 + 𝑡 = 3 maka kemungkinan triple


(𝑚, 𝑠, 𝑡) = (1, 1, 1), (2, 1, 0), (3, 0 ,0) sehingga koefisien 𝑥 3 adalah

𝐶112 𝐶11 𝐶11 + 𝐶212 𝐶12 𝐶01 + 𝐶312 𝐶03 𝐶00 = 12 + 132 + 220 = 364

146 | K O M B I N A T O R I K A
Olimpiade Matematika SMA 2020

Problem 18
1 12
Nilai konstanta pada ekspansi (𝑥 + 𝑥 3 ) adalah ....

Pembahasan :
12
1 12
(𝑥 + 3 ) = ∑ 𝐶𝑘12 𝑥 𝑘 (𝑥 −3 )12−𝑘
𝑥
𝑘=0

12
1 12
(𝑥 + 3 ) = ∑ 𝐶𝑘12 𝑥 −36+4𝑘
𝑥
𝑘=0

Untuk mendapatkan koefien suatu konstanta maka −36 + 4𝑘 = 0 sehingga 𝑘 = 9


maka :
12!
𝐶𝑘12 = 𝐶912 = = 220
9! . 3!

Problem 19
Dengan teorema bimial Newton, berapakah dua digit pertama dari 1111 ?
Pembahasan :
1111 = (1 + 10)11
1111 = 1 + 𝐶111 10 + 𝐶211 102 + 𝐶311 103 + ⋯ . . + 𝐶10
11 11
1010 + 𝐶11 1011
1111 = 1 + 10 + 55 102 + 165 103 + ⋯ . . + 55 1010 + 11 1010 + 1011
1111 = 1011 + 11 1010 + 55 1010 + ⋯ … + 165 103 + 55 102 + 10 + 1
1111 = 1011 (1 + 1,1 + 0,55 + 0,165 + ⋯ … )
1111 = 1011 (2,86 … . . )
Maka dua digit pertama dari 1111 adalah 28

Problem 20
Diketahui himpunan A = {3, 11, 19, 27,. . . , 147, 155}
Berapa maksimal banyaknya pasangan dua anggota himpunan A yang
berjumlah 158 ?

Pembahasan :
Jumlah anggota himpunan A adalah 20 dicari dengan rumus suku ke-n barisan
aritmatika. Maka dengan teori sarang merpati, akan ada 10 pasangan maksimum
dua anggota himpunan A yang berjumlah 158.

147 | K O M B I N A T O R I K A
Olimpiade Matematika SMA 2020

Problem 21
Sebuah guci berisi 4 bola hijau dan 6 bola biru. Guci kedua berisi 16 bola hijau
dan N bola biru. Sebuah bola diambil secara acak dari masing-masing guci.
Kemungkinan kedua bola memiliki warna yang sama adalah 0,58. Tentukan N !

Pembahasan :
Pertama, kita temukan probabilitas keduanya hijau, lalu probabilitas keduanya
biru, dan tambahkan dua probabilitas yang sama yaitu 0,58. Probabilitas
4 16
keduanya berwarna hijau adalah 𝑥 dan probabilitas keduanya
10 16+𝑁
6 𝑁
berwarna biru adalah 𝑥 , jadi :
10 16+𝑁

4 16 6 𝑁
𝑥 + 𝑥 = 0,58
10 16 + 𝑁 10 16 + 𝑁

6𝑁 + 64
= 5,8
16 + 𝑁
6𝑁 + 64 = 5,8𝑁 + 92,8
0,2𝑁 = 28,8
𝑁 = 144

Problem 22
Jenn secara random memilih nomor J yaitu 1, 2, 3, 4, ......., 19, 20 dan Bela secara
random juga memilih nomor B yaitu 1, 2, 3, 4, ......., 19, 20 berbeda dari J.
Nilai B – J sekurang-kurangnya 2 dengan probabilitas yang dapat diekspresikan
𝑚
dalam bentuk dimana 𝑚 dan 𝑛 bilangan bulat positif yang relatif prima. Maka
𝑛
𝑚 + 𝑛 adalah ...

Pembahasan :
Masalah ini pada dasarnya menanyakan berapa banyak cara yang ada untuk
memilih 2 elemen berbeda dari 20 elemen yang diatur sehingga tidak ada 2
elemen yang berdekatan. Menggunakan 𝐶𝑘𝑛−𝑘+1 = 𝐶220−2+1 = 𝐶219 = 171
Dengan ruang sampel 19 x 20 = 380, peluang yang diinginkan adalah
171 9
𝑃= =
380 20
𝑚 9
Dengan demikian = , sehingga 𝑚 + 𝑛 = 29
𝑛 20

148 | K O M B I N A T O R I K A
Olimpiade Matematika SMA 2020

Problem 23
Melinda memiliki tiga kotak kosong dan 12 buku teks, tiga di antaranya adalah
buku teks matematika. Satu kotak akan menampung tiga buku teksnya, satu
akan menampung empat buku teksnya, dan satu akan menampung lima buku
teksnya. Jika Melinda mengemas buku pelajarannya ke dalam kotak-kotak ini
dalam urutan acak, probabilitas bahwa ketiga buku teks matematika tertampung
𝑚
di kotak yang sama dapat ditulis sebagai , di mana m dan n relatif bilangan
𝑛
bulat positif. Tentukan nilai m + n

Pembahasan :

Anggap buku-buku itu matematika atau bukan matematika di mana buku-buku


dalam setiap kategori tidak dapat dibedakan satu sama lain. Lalu, ada 𝐶312 total
cara yang bisa dibedakan untuk mengemas buku. Sekarang, untuk menentukan
propabilitas yang diinginkan, kita harus menemukan jumlah cara kondisi bahwa
semua buku matematika di kotak yang sama dapat dipenuhi. Pada setiap kotak :

Buku matematika yang ditempatkan di dalam kotak terkecil, 𝐶33 cara = 1 cara

Buku matematika yang ditempatkan di dalam kotak sedang, 𝐶34 cara = 4 cara

Buku matematika yang ditempatkan di dalam kotak terbesar, 𝐶35 cara = 10 cara

Jadi total cara menyimpat buku = 1 + 4 + 10 = 15 cara

Sedang ruang sampel = 𝐶312 = 220 cara

𝑚 15 3
Maka = = , sehingga m + n = 3 + 44 = 47
𝑛 220 44

Problem 24
Koin yang muncul kepala dengan probabilitas p > 0 dan ekor dengan
probabilitas 1 − 𝑝 > 0, setiap flip diputar delapan kali. Misalkan probabilitas tiga
1
kepala dan lima ekor sama dengan dari probabilitas lima kepala dan tiga ekor.
25
𝑚
Jika 𝑝 = , di mana m dan n adalah bilangan bulat positif yang relatif prima.
𝑛
Temukan m + n

149 | K O M B I N A T O R I K A
Olimpiade Matematika SMA 2020

Pembahasan :

Probabilitas tiga kepala dan lima ekor adalah 𝐶38 𝑝3 (1 − 𝑝)5


Probabilitas lima kepala dan tiga ekor adalah 𝐶38 𝑝5 (1 − 𝑝)3
1
Maka 𝐶38 𝑝3 (1 − 𝑝)5 = 25 𝐶38 𝑝5 (1 − 𝑝)3

25 (1 − 𝑝)2 = 𝑝2
5(1 − 𝑝) = 𝑝
6𝑝 = 5
5
𝑝=
6
𝑚 5
Sehingga = , jadi m + n = 5 + 6 = 11
𝑛 6

Problem 25
Sepuluh peti identik masing-masing ber dimensi 3 x 4 x 6 m3. Peti pertama
ditempatkan rata di lantai. Masing-masing dari sembilan peti yang tersisa
ditempatkan, pada gilirannya, datar di atas peti sebelumnya, dan orientasi setiap
𝑚
peti dipilih secara acak. Misalkan adalah probabilitas bahwa tumpukan peti
𝑛
persis tingginya 41 m, di mana m dan n adalah bilangan bulat positif yang relatif
prima. Tentukan nilai m !
Pembahasan :

Hanya ketinggian yang penting, dan setiap peti memiliki tinggi 3, 4, atau 6 m
dengan probabilitas yang sama.
3𝑎 + 4𝑏 + 6𝑐 = 41
𝑎 + 𝑏 + 𝑐 = 10
Hasil eliminasinya 𝑏 + 3𝑐 = 11 kita dapatkan (𝑎 , 𝑏) = (2 , 3), (5 , 2), (8 , 1), (11 , 0)
(11 , 0) tidak mungkin memenuhi, sehingga hanya bisa tiga penyelesaian
(𝑎, 𝑏, 𝑐) = (5, 2, 3), (3, 5, 2), (1, 8, 1)
10 !
Untuk (5, 2, 3) dan (3, 5, 2) berlaku 2 = 10 . 9 . 8 . 7 cara
5! 2! 3!
Untuk (1, 8, 1) berlaku 2 𝐶210 = 90
Dan ada 310 cara untuk menumpuk peti dengan ketinggian berapapun

𝑚 10 .9 .8 .7+90 190
Jadi, = = , dengan demikian m = 190
𝑛 310 37

150 | K O M B I N A T O R I K A
Olimpiade Matematika SMA 2020

Soal-soal Latihan Isian Singkat

1
Jika 𝑎, 𝑏, 𝑐 dan 𝑑 bilangan bulat tak negatif, banyaknya kombinasi sehingga
memenuhi 𝑎 + 𝑏 + 𝑐 + 𝑑 = 6 adalah .....
Kunci Jawab : 84

2
Dari 100 orang, banyaknya orang yang lahir pada bulan yang sama, paling
sedikit ada ........ orang
Kunci Jawab : 12

3
Pada pola bilangan
1
2 3
4 5 6
7 8 9 10
11 12 13 14 15
.........
Bilangan terakhir pada baris ke 20 adalah .....
Kunci Jawab : 210

4
Pada sebuah lemari terdapat 25 helai baju yang terdiri atas 4 ukuran, yaitu 5 helai
baju berukuran S, 4 helai baju berukuran M, 9 helai baju berukuran L, dan 7 helai
baju berukuran XL. Tentukan jumlah baju paling sedikit yang dapat diambil agar
selalu diperoleh 7 helai baju berukuran sama.
Kunci jawab : 22

151 | K O M B I N A T O R I K A
Olimpiade Matematika SMA 2020

5
Seorang siswa menyusun bilangan-bilangan ganjil secara berkelompok seperti di
bawah ini :
{ 1 } , {3 , 5} , {7, 9, 11} , {13, 15, 17, 19} , {21, 23, 25, 27, 29} , .......
Bilangan ke 20 pada kelompok ke 20 adalah ....
Kunci Jawab : 419

6
Seekor semut bergerak dari A menuju B pada lantai berpetak seperti gambar di
bawah ini :
B

A
Banyaknya cara semut itu berjalan dari A menuju B adalah ...
Kunci Jawab : 35

7
𝑎 𝑏
𝑎 𝑏 𝑎 𝑏 𝑐
Diketahui matriks-matriks 𝐴 = ( ) , 𝐵=( 𝐶 = ( 𝑐 𝑑)
),
𝑐 𝑑 𝑑 𝑒 𝑓
𝑒 𝑓
Banyaknya cara perkalian dua matriks yang mungkin dari ketiga matriks diatas
adalah .....
Kunci Jawab : 3

8
Delapan bola diberi nomor 1 sampai 8 ada dalam keranjang. Banyaknya bola
yang sekaligus diambil agar didapat setidaknya ada sepasang bola yang
nomornya berjumlah 9 adalah ....
Kunci Jawab : 5

152 | K O M B I N A T O R I K A
Olimpiade Matematika SMA 2020

9
Dari bilangan 0, 1, 2, 3, 4, 5, 6, 7, 8 dan 9 akan dibuat bilangan-bilangan terdiri
dari 3 angka. Banyaknya bilangan yang tidak habis dibagi 4 adalah
Kunci Jawab : 775

10
Jika 10 ditulis sebagai jumlah dari 3 bilangan positif, maka banyaknya cara
menuliskannya adalah ....
Kunci Jawab : 36

11
Lima orang berada di dalam lift yang memiliki 8 lantai. Banyaknya cara ke lima
orang itu memilih keluar dari lift adalah ....
Kunci Jawab : 792

12
Persamaan kuadrat 𝑎𝑥 2 + 𝑏𝑥 + 1 = 0 yang memiliki akar-akar real dengan
𝑎 dan 𝑏 konstanta bulat. Jika 𝑎 dan b dipilih dari 0 ≤ (𝑎 , 𝑏) ≤ 4 , banyaknya
persamaan kuadrat yang mungkin adalah ....
Kunci Jawab : 7

13
Nilai dari :
1 2 3 4 5 6 7 8 10
+ + + + + + + + adalah .....
2! 3! 4! 5! 6! 7! 8! 9! 10!
Kunci Jawab : 1

14
Banyaknya penyelesaian bulat tak negatif dari yang memenuhi x + y + z = 20
dengan syarat x > 0 adalah ....
Kunci Jawab : 210

153 | K O M B I N A T O R I K A
Olimpiade Matematika SMA 2020

15
Angka terakhir bila P = 1! + 2! + 3! + . . . + 2020 ! adalah. …
Kunci jawab : 3

16
Diketahui 𝑎 + (𝑎 + 1) + (𝑎 + 2) + (𝑎 + 3) + ⋯ … … … + 50 = 1139, maka nilai 𝑎 positif
adalah ....
Kunci Jawab : 17

17
Peserta Indonesia yang berangkat ke ajang lomba Cerdas Tangkas terdiri dari 5
orang. Untuk dapat berangkat ke lomba tersebut, setiap peserta harus lolos
seleksi. Ada 7 orang pria dan 5 orang wanita yang berhasil lolos seleksi untuk
menjadi peserta lomba. Jika dipersyaratkan bahwa paling sedikit satu orang
peserta adalah wanita, berapa banyaknya cara memilih peserta lomba?
Kunci jawab : 771

18
99 ! 1
Tentukan nilai 𝑛 pada =
101 ! − 99 ! 𝑛
Kunci jawab : 10099

19

Bilangan tiga digit yang merupakan faktorial dari digit-digitnya adalah …


Kunci jawab : 145

20

Suatu lomba maraton diikuti oleh empat kelompok: Melati, Mawar, Dahlia, dan
Anggrek. Setiap kelompok mengirimkan lima pelari. Pelari yang masuk finish
ke-1, 2, 3, 4, 5, 6 memperoleh nilai berturut-turut 7, 5, 4, 3, 2, 1. Nilai setiap
kelompok adalah jumlah nilai kelima pelarinya. Kelompok dengan nilai terbesar
adalah juara lomba. Di akhir lomba ternyata kelompok Dahlia menjadi juara dan
tidak ada dua pelari yang masuk finish bersamaan. Berapa banyak kemungkinan
nilai kelompok pemenang ?
Kunci jawab : 15

154 | K O M B I N A T O R I K A
Olimpiade Matematika SMA 2020

21

Koefisien 𝑎𝑏 2 𝑐 3 pada penjabaran (3𝑎 + 𝑏 − 2𝑐)6 adalah ....


Kunci jawab : −1440

22

Jumlah semua Koefisien pada penjabaran (2𝑎 + 3𝑏 − 5𝑐 + 𝑑 + 2𝑒 − 𝑓)3 adalah


Kunci jawab : 8

23

Diketahui himpunan 𝐴 = {1, 2, 3, 4, … . . 4038}. Jika subhimpunan dari A yang


terdiri dari 𝑘 elemen selalu memuat 2 buah bilangan yang saling prima, maka
nilai 𝑘 yang memenuhi adalah…
Kunci jawab : 2020

24
Perhatikan papan catur dengan aturan tambahan yang menyatakan: "Suatu bidak
hanya boleh bergerak ke kanan dan ke atas saja". Jika bidak tersebut ditempatkan
di diagonal kiri bawah, berapa banyaknya cara yang dapat dilakukan untuk
membawa bidak ke diagonal atas kanan?
Kunci Jawab : 252

25
Sebuah kotak berisi 4 bola merah, 4 bola hijau, dan 4 bola biru. Pada setiap bola
tertulis salah satu bilangan bulat antara 1 sampai 4. Tidak ada dua buah bola
yang memiliki warna dan angka yang sama. Berapa banyak minimal bola yang
harus diambil dari kotak agar pasti terdapat dua buah bola yang memiliki warna
yang sama dan hasil penjumlahan angka-angka pada kedua bola tersebut 5 ?
Kunci jawab : 7

26
Sepuluh loker berurutan masing-masing diberi nomor 1 sampai 10. Setiap loker
harus dicat biru, merah atau hijau. Dua loker bernomor m dan n dicat dengan
warna berbeda, sehingga 𝑚 − 𝑛 selalu ganjil. Jika tidak ada keharusan ketiga
warna itu dipakai, dalam berapa banyak cara koleksi loker dapat dicat ?
Kunci jawab : 186

155 | K O M B I N A T O R I K A
Olimpiade Matematika SMA 2020

27
Pada babak final sebuah turnamen, tim pemenang adalah tim yang pertama
sekali memenangkan 2 pertandingan secara berurutan atau tim yang pertama
sekali memenangkan 4 pertandingan. Banyak cara turnamen dapat terjadi
adalah ....
Kunci jawab : 12

28
Seorang pembuat plat nomor kendaraan ingin membuat plat nomor yang terdiri
dari 4 angka dengan angka pertama bukan nol. Plat nomor itu memuat tiga
atau empat suku barisan aritmetika dengan beda atau selisih barisan tersebut
merupakan bilangan bulat positif.
Berapa banyaknya PNKB istimewa dimaksud ?
Kunci jawab : 331

29
Dari 100.000 buah bilangan bulat positif pertama, berapa banyak bilangan yang
mengandung tepat 1 buah angka 3, 1 buah angka 4, dan 1 buah angka 5?
Kunci jawab : 2940

30
Dari sejumlah besar koin 25-an, 50-an, 100-an, dan 500-an, berapa banyak cara
lima koin dapat diambil?
Kunci jawab : 56

31
Di perpustakaan sekolah terdapat 3 jenis buku berbeda: buku Matematika, buku
Biologi, dan Fisika. Perpustakaan menyediakan sedikitnya 10 buah buku untuk
masing-masing jenisnya. Berapa banyak cara memilih 10 buah buku?
Kunci jawab : 66

32
Banyaknya penyelesaian bulat tak negatif dari adalah 𝑥1 + 𝑥2 + 𝑥3 + 𝑥4 = 25
adalah ....
Kunci jawab : 3276

156 | K O M B I N A T O R I K A
Olimpiade Matematika SMA 2020

33
Ada berapa banyak bilangan 3-digit yang habis dibagi dengan 13
Kunci jawab : 69

34
Sebuah toko roti memproduksi 8 jenis donat. Donat dikemas dalam kotak berisi
12 buah donat. Banyaknya cara untuk mengisi sebuah kotak sehingga terdapat
sedikitnya satu buah donat untuk setiap jenis adalah…
Kunci jawab : 330

35
Jika 𝑎 dan 𝑏 dipilih secara acak dari {2, 3, 5 ,6 , 8} maka banyaknya pasangan (𝑎 , 𝑏)
𝑎
sedemikian sehingga merupakan bilangan bulat adalah
𝑏
Kunci jawab : 8

157 | K O M B I N A T O R I K A
Olimpiade Matematika SMA 2020

Soal-soal Latihan Uraian Singkat

1. Huruf-huruf pada kata IDEPALIMO apabila ditata ulang berdasarkan urutan


huruf, salah satunya akan menjadi kata OLIMPIADE, pada urutan ke ....

2. Dalam satu kelas terdapat 20 orang siswa yang harus mengikuti tes. Setiap
siswa tidak harus hanya mengikuti 1 macam tes saja, namun 4 orang siswa
harus mengikuti 5 macam tes yang tersedia. Berapa banyaknya cara untuk
melakukan hal ini ?

3. Dalam sebuah permainan jika seseorang berada pada rintangan ke-n, orang
tersebut harus melemparkan dadu (seimbang) sebanyak n kali. Jika jumlah
mata dadu dari pelemparan ini lebih besar dari 2n, maka orang tersebut
berhasil melewati rintangan. Tentukan peluang bahwa seseorang berhasil
melewati tiga rintangan pertama.

4. Pada suatu kotak ada sekumpulan bola berwarna merah dan hitam yang
secara keseluruhannya kurang dari 1000 bola. Misalkan diambil dua bola.
Peluang terambilnya dua bola merah adalah p dan peluang terambilnya dua
23
bola hitam adalah q dengan 𝑝 − 𝑞 = . selisih terbesar yang mungkin dari
37
banyaknya bola merah dan bola hitam adalah ....

5. Diberikan persamaan 𝑥1 + 𝑥2 + 𝑥3 = 10, dengan 𝑥𝑖 adalah bilangan cacah. Jika


0 ≤ 𝑥1 ≤ 2 , 𝑥1 > 1 dan 𝑥3 ≥ 0 , maka banyaknya solusi x yang mungkin
adalah .....

6. Jika 𝑈𝑛 = 𝐶𝑜𝑛 + 𝐶1𝑛−1 + 𝐶2𝑛−2 + 𝐶3𝑛−3 + + ⋯ , 𝑛 ≥ 1


Maka 𝑈2020 = .....

7. Di dalam suatu kotak terdapat n kelereng merah dan m kelereng biru.


Diambil 5 kelereng sekaligus. Jika peluang terambilnya 3 kelereng merah
25
dan 2 kelereng biru adalah maka nilai terkecil dari 𝑛2 + 𝑚2 yang
77
mungkin adalah ...

8. Untuk −3 ≤ 𝑎 ≤ 3 dan −3 ≤ 𝑏 ≤ 3 , maka peluang agar 𝑎 dan 𝑏 memenuhi


𝑎2 + 𝑏 2 ≤ 4 adalah ...

9. Berapa peluang menemukan paling sedikit ada 2 orang dari 3 orang, yang
lahir pada bulan yang sama ?

158 | K O M B I N A T O R I K A
Olimpiade Matematika SMA 2020

10. Misalkan S menyatakan himpunan semua faktor positif dari 20202 . Sebuah
bilangan diambil secara acak dari S. Peluang bilangan yang terambil itu
merupakan kelipatan 2020 adalah ....

Soal dan Pembahasan


Soal & Pembahasan nomor 1 :
Huruf-huruf pada kata IDEPALIMO apabila ditata ulang berdasarkan urutan
huruf, salah satunya akan menjadi kata OLIMPIADE, pada urutan ke ....
Pembahasan :

9!
Huruf-huruf pada kata IDEPALIMO jika ditata ulang seluruhnya terdapat .
2!
Jika diurutkan berdasarkan abjad adalah ADEIILMPO untuk menentukan urutan
kata OLIMPIADE kita gunakan cara permutasi.
8!
Awalan A , D, E, L, M dan P terdapat 6 x kata
2!
Awalan I terdapat 8! Kata
Awalan OA, OD, OI dan OL terdapat 4 x 7! Kata
Awalan OLA, OLD, OLE dan OLI terdapat 4 x 6!
Awalan OLIA, OLID, OLIE dan OLIM terdapat 4 x 5!
Awalan OLIMA, OLIMD, OLIME, OLIMI dan OLIMP ada 5 x 4!
Awalan OLIMPA, OLIMPD, OLIMPE, OLIMPI ada 4 x 3!
Awalan OLIMPIADE ada 1

Maka kata OLIMPIADE apa pada urutan ke


8!
6x + 8! + 4 x 7! + 4 x 6! + 4 x 5! + 5 x 4! + 4 x 3! + 1
2!

159 | K O M B I N A T O R I K A
Olimpiade Matematika SMA 2020

Soal & Pembahasan nomor 2 :


Dalam satu kelas terdapat 20 orang siswa yang harus mengikuti tes. Setiap siswa
tidak harus hanya mengikuti 1 macam tes saja, namun 4 orang siswa harus
mengikuti 5 macam tes yang tersedia. Berapa banyaknya cara untuk melakukan
hal ini ?

Pembahasan :
Banyaknya cara melakukan mengambil 4 orang dari 20 siswa yang harus 5
20 ! 20 .19 .18 .17
macam test adalah 𝐶420 = = = 4845.
16 ! . 4 ! 24
Sedangkan 16 siswa yang lain dapat melakukan 1 test, 2 test , 3 test, atau 4 test
atau 5 test, ditulis sebagai 𝐶116 + 𝐶216 + 𝐶316 + 𝐶416 = 16 + 120 + 560 + 1820 = 2516
Oleh karena itu banyaknya cara yang diinginkan : 4845 x 2516 = 12.114.540

Soal & Pembahasan nomor 3 :


Dalam sebuah permainan jika seseorang berada pada rintangan ke-n, orang
tersebut harus melemparkan dadu (seimbang) sebanyak n kali. Jika jumlah mata
dadu dari pelemparan ini lebih besar dari 2n, maka orang tersebut berhasil
melewati rintangan. Tentukan peluang bahwa seseorang berhasil melewati
tiga rintangan pertama.

Pembahasan:
aturan yang sudah ditentukan. Peserta yang berhasil melewati rintang apabila
jumlah mata dadu dari n pelemparan > 2n

Rintangan pertama
Pelemparan pertama agar berhasil melewati rintangan maka jumlah mata dadu
yang muncul harus lebih dari 21, sehingga didapat
n(S) = 61 = 6
n(A) = 6 – 2 = 4
𝑛(𝐴) 4 2
P(A) = = =
𝑛(𝑆) 6 3

Rintangan kedua
Pelemparan kedua agar berhasil melewati rintangan maka jumlah mata dadu
yang muncul harus lebih dari 22 = 4, sehingga didapat n(S) = 62 = 36

160 | K O M B I N A T O R I K A
Olimpiade Matematika SMA 2020

Kemudian dicari jumlah mata dadu ≤ 4, atau 𝑥1 + 𝑥2 = 4, 𝑥1 + 𝑥2 = 3, dan 𝑥1 +


𝑥2 = 2, dimana nilai 𝑥1 dan 𝑥2 merupakan bilangan asli
untuk persamaan dari 𝑥1 + 𝑥2 = 4, bisa menggunakan aturan kombinasi 𝐶13 = 3
untuk persamaan dari 𝑥1 + 𝑥2 = 3, bisa menggunakan aturan kombinasi 𝐶12 = 2
untuk persamaan dari 𝑥1 + 𝑥2 = 2, bisa menggunakan aturan kombinasi 𝐶11 = 1

Dengan demikian,
n(A) = 62 – (3 + 2 + 1)
n(A) = 30
𝑛(𝐴) 30 5
P(A) = = =
𝑛(𝑆) 36 6

Rintangan ketiga
Pelemparan kedua agar berhasil melewati rintangan maka jumlah mata dadu
yang muncul harus lebih dari 23 = 8, sehingga didapat n(S) = 63 = 216
Kemudian dicari jumlah mata dadu ≤ 8, atau 𝑥1 + 𝑥2 + 𝑥3 = 8, 𝑥1 + 𝑥2 + 𝑥3 = 7,
𝑥1 + 𝑥2 + 𝑥3 = 6, 𝑥1 + 𝑥2 + 𝑥3 = 5, 𝑥1 + 𝑥2 + 𝑥3 = 4, dan 𝑥1 + 𝑥2 + 𝑥3 = 3 dengan
𝑥1 , 𝑥2 dan 𝑥3 bilangan asli

untuk 𝑥1 + 𝑥2 + 𝑥3 = 8, bisa menggunakan aturan kombinasi 𝐶27 = 21


untuk 𝑥1 + 𝑥2 + 𝑥3 = 7, bisa menggunakan aturan kombinasi 𝐶26 = 15
untuk 𝑥1 + 𝑥2 + 𝑥3 = 6, bisa menggunakan aturan kombinasi 𝐶25 = 10
untuk 𝑥1 + 𝑥2 + 𝑥3 = 5, bisa menggunakan aturan kombinasi 𝐶24 = 6
untuk 𝑥1 + 𝑥2 + 𝑥3 = 4, bisa menggunakan aturan kombinasi 𝐶23 = 3
untuk 𝑥1 + 𝑥2 + 𝑥3 = 3, bisa menggunakan aturan kombinasi 𝐶22 = 1

Dengan cara yang sama, maka didapat


n(A) = 63 – (21 + 15 + 10 + 6 + 3 + 1)
n(A) = 216 – 56
n(A) = 160
𝑛(𝐴) 160 20
P(A) = = =
𝑛(𝑆) 216 27

2 5 20 100
Dengan demikian, peluang seluruhnya adalah 𝑥 𝑥 =
3 6 27 243

161 | K O M B I N A T O R I K A
Olimpiade Matematika SMA 2020

Soal & Pembahasan nomor 4 :


Pada suatu kotak ada sekumpulan bola berwarna merah dan hitam yang secara
keseluruhannya kurang dari 1000 bola. Misalkan diambil dua bola. Peluang
terambilnya dua bola merah adalah p dan peluang terambilnya dua bola hitam
23
adalah q dengan 𝑝 − 𝑞 = . selisih terbesar yang mungkin dari banyaknya
37
bola merah dan bola hitam adalah ....

Pembahasan :
Misalnya x dan y masing-masing adalah banyaknya bola merah dan hitam.
𝑦
𝐶2𝑥 𝐶2
𝑝= 𝑥+𝑦 𝑑𝑎𝑛 𝑞 = 𝑥+𝑦
𝐶2 𝐶2
23
𝑝−𝑞 =
37
𝑦
𝐶2𝑥 𝐶2 23
𝑥+𝑦 − 𝑥+𝑦 =
𝐶2 𝐶2 37

𝑥! 𝑦!
(𝑥 − 2) ! . 2! (𝑦 − 2) ! . 2! 23
− =
(𝑥 + 𝑦)! (𝑥 + 𝑦)! 37
(𝑥 + 𝑦 − 2) ! . 2! (𝑥 + 𝑦 − 2) ! . 2!
𝑥 (𝑥 − 1) 𝑦 (𝑦 − 1) 23
− =
(𝑥 + 𝑦)(𝑥 + 𝑦 − 1) (𝑥 + 𝑦)(𝑥 + 𝑦 − 1) 37

(𝑥 − 𝑦) (𝑥 + 𝑦 − 1) 23
=
(𝑥 + 𝑦)(𝑥 + 𝑦 − 1) 37

(𝑥 − 𝑦) 23
=
(𝑥 + 𝑦) 37

23
𝑑𝑒𝑛𝑔𝑎𝑛 𝑥 + 𝑦 < 1000, maka bentuk pecahan diperbesar dengan penyebut
37
23 621
mendekati 1000, seperti = , jadi :
37 999

(𝑥 − 𝑦) 621
=
(𝑥 + 𝑦) 999

selisih terbesar yang mungkin dari banyaknya bola merah dan hitam adalah 621

162 | K O M B I N A T O R I K A
Olimpiade Matematika SMA 2020

Soal & Pembahasan nomor 5 :


Diberikan persamaan 𝑥1 + 𝑥2 + 𝑥3 = 10, dengan 𝑥𝑖 adalah bilangan cacah. Jika 0 ≤
𝑥1 ≤ 2 , 𝑥1 > 1 dan 𝑥3 ≥ 0 , maka banyaknya solusi x yang mungkin adalah ..

Pembahasan :
Analogikan dengan membagi 10 buah bola yang identik ke dalam 3 buah kotak,
sebut saja kotak 𝑥1 , 𝑥2 𝑑𝑎𝑛 𝑥3 . Jadi, 𝑥1 ada kemungkinan berisi 0 (tak berisi), 1,
atau 2.

Untuk masing-masing nilai 𝑥1 , kita perinci perhitungannya sebagai berikut.


(i) Kasus 𝑥1 = 0, berarti 𝑥2 + 𝑥3 = 10.
Isikan 2 bola ke dalam kotak 𝑥2 (karena syaratnya 𝑥2 > 1). Bagikan 8 bola
sisanya ke kotak 𝑥2 dan 𝑥3 , semuanya ada 𝐶82+8−1 = 𝐶89 = 9 cara.

(ii) Kasus 𝑥1 = 1, berarti 𝑥2 + 𝑥3 = 9. Isikan 2 bola ke dalam kotak 𝑥2 (karena


syaratnya 𝑥2 > 1). Bagikan 7 bola sisanya ke kotak 𝑥2 dan 𝑥3 , semuanya
ada 𝐶72+7−1 = 𝐶78 = 8 cara.

(iii) Kasus 𝑥1 = 2, berarti 𝑥2 + 𝑥3 = 8. Isikan 2 bola ke dalam kotak 𝑥2 (karena


syaratnya 𝑥2 > 1). Bagikan 6 bola sisanya ke kotak 𝑥2 dan 𝑥3 , semuanya
ada 𝐶62+6−1 = 𝐶67 = 7 cara.

Jumlah solusi seluruhnya adalah 9 cara + 8 cara + 7 cara = 24 cara

Soal & Pembahasan nomor 6 :


Jika 𝑈𝑛 = 𝐶𝑜𝑛 + 𝐶1𝑛−1 + 𝐶2𝑛−2 + 𝐶3𝑛−3 + + ⋯ , 𝑛 ≥ 1
Maka 𝑈2020 = .....

Pembahasan :
𝑈1 = 𝐶01 + 𝐶10 = 1 + 0 = 1
𝑈2 = 𝐶02 + 𝐶11 + 𝐶20 = = 1 + 1+ 0 = 2
𝑈3 = 𝐶03 + 𝐶12 + 𝐶21 + 𝐶30 = = 1 + 2 + 0 + 0 = 3
𝑈4 = 𝐶04 + 𝐶13 + 𝐶22 + 𝐶31 + 𝐶40 = = 1 + 3 + 1 + 0 + 0 = 5
𝑈3 = 8 dan seterusnya

163 | K O M B I N A T O R I K A
Olimpiade Matematika SMA 2020

Pola barisan yang dihasilkan ternyata


𝑈3 = 𝑈2 + 𝑈1
𝑈4 = 𝑈3 + 𝑈2
𝑈5 = 𝑈4 + 𝑈3
Terbentuk barisan Fibonaci
𝑛

𝐹𝑛+1 = ∑ 𝐶𝑘𝑛−𝑘+1
𝑘=0
Suku ke 2020 dapat diturunkan jika 𝑛 = 1
2019

𝐹𝑛+1 = ∑ 𝐶𝑘2020−𝑘
𝑘=0

Soal & Pembahasan nomor 7 :


Di dalam suatu kotak terdapat n kelereng merah dan m kelereng biru. Diambil 5
kelereng sekaligus. Jika peluang terambilnya 3 kelereng merah dan 2 kelereng
25
biru adalah maka nilai terkecil dari 𝑛2 + 𝑚2 yang mungkin adalah ...
77

Pembahasan :
Pengambilan 5 sekaligus kelereng merah dan biru, (3 , 2) dari (𝑛 , 𝑚)
𝑛! 𝑚!
𝐶3𝑛 . 𝐶2𝑚 25 . 25
(𝑛 − 3)! . 3! (𝑚 − 2)! . 2!
𝑛+𝑚 = 𝑎𝑡𝑎𝑢
(𝑛 + 𝑚)!
=
𝐶5 77 77
(𝑛 + 𝑚 − 5)! . 5!
𝑛 (𝑛 − 1)(𝑛 − 2) 𝑚 (𝑚 − 1) 5! 25
. =
(𝑛 + 𝑚)(𝑛 + 𝑚 − 1)(𝑛 + 𝑚 − 2)(𝑛 + 𝑚 − 3)(𝑛 + 𝑚 − 4) 3! . 2! 77
𝑛 (𝑛 − 1)(𝑛 − 2) 𝑚 (𝑚 − 1) 5
=
(𝑛 + 𝑚)(𝑛 + 𝑚 − 1)(𝑛 + 𝑚 − 2)(𝑛 + 𝑚 − 3)(𝑛 + 𝑚 − 4) 154
𝑛 (𝑛 − 1)(𝑛 − 2) 𝑚 (𝑚 − 1) 5
=
(𝑛 + 𝑚)(𝑛 + 𝑚 − 1)(𝑛 + 𝑚 − 2)(𝑛 + 𝑚 − 3)(𝑛 + 𝑚 − 4) 11 . 14

Karena yang ditanyakan terkecil dari 𝑛2 + 𝑚2 maka nilai ini dapat terjadi jika 𝑛 +
𝑚 memiliki nilai terkecil. Pada persamaan pecahan ruas kiri 11 haruslah dapat
membagi (𝑛 + 𝑚)(𝑛 + 𝑚 − 1)(𝑛 + 𝑚 − 2)(𝑛 + 𝑚 − 3)(𝑛 + 𝑚 − 4) sehingga
nilai terkecil 𝑛 + 𝑚 adalah 11.

164 | K O M B I N A T O R I K A
Olimpiade Matematika SMA 2020

Jika 𝑛 + 𝑚 = 11, maka nilai 𝑛 dan 𝑚 masing-masing dapat didaftar dan


diperiksa untuk menentukan nilai 𝑛 dan 𝑚 yang memenuhi. Untuk
memudahkan memprediksi nilai 𝑛 dan 𝑚 , maka bentuk terakhir persamaan ini
sangat membantu.
𝑛 𝑚 ((𝑛 − 1) (𝑚 − 1))(𝑛 − 2) 5
=
11 (11 − 1)(11 − 2)(11 − 3)(11 − 4) 11 . 14
𝑛 𝑚 (𝑛 𝑚 − (𝑛 + 𝑚) + 1)(𝑛 − 2) 5
=
11 (10)(9)(8)(7) 11 . 14
𝑛 𝑚 (𝑛 𝑚 − 10)(𝑛 − 2) 5
=
11 . 14 (10)(9)(4) 11 . 14
𝑛 𝑚 (𝑛 𝑚 − 10) (𝑛 − 2)
=1
30 20 3
Pada persamaan terakhir, setelah diperiksa, ternyata pasangan (𝑛 , 𝑚) yang
memenuhi adalah (5 , 6) atau (6 , 5).

Soal & Pembahasan nomor 8 :


Untuk −3 ≤ 𝑎 ≤ 3 dan −3 ≤ 𝑏 ≤ 3 , maka peluang agar 𝑎 dan 𝑏 memenuhi 𝑎2 +
𝑏 2 ≤ 4 adalah ...
Pembahasan :
𝑎2 + 𝑏 2 = 4 adalah persamaan lingkaran dengan pusat (0 , 0) terhadap sumbu 𝑎
dan 𝑏 dari jari-jari lingkaran 2.

-3 3

-3

Peluang agar 𝑎 dan 𝑏 yang memenuhi agar 𝑎2 + 𝑏 2 ≤ 4 merupakan


perbandingan luas daerah yang diarsir terhadap Luas persegi ABCD yang
membatasi nilai-nilai 𝑎 dan 𝑏.
22 22
𝑙𝑢𝑎𝑠 𝑙𝑖𝑛𝑔𝑘𝑎𝑟𝑎𝑛 (2)2 4 22
𝑃= = 7 = 7 =
𝐿𝑢𝑎𝑠 𝑝𝑒𝑟𝑠𝑒𝑔𝑖 𝐴𝐵𝐶𝐷 6 .6 36 63

165 | K O M B I N A T O R I K A
Olimpiade Matematika SMA 2020

Soal & Pembahasan nomor 9 :


Berapa peluang menemukan paling sedikit ada 2 orang dari 3 orang, yang lahir
pada bulan yang sama ?

Pembahasan :
Ruang sampel (S) adalah banyaknya cara 3 orang lahir pada tahun yang sama
berdasarkan bulan (1 tahun = 12 bulan), yaitu 12 x 12 x 12 cara
Jika A = himpunan 3 orang yang lahir tidak pada bulan yang sama, maka
banyaknya kejadian A adalah 12 x 11 x 10 cara
Maka Ac = himpunan sedikitnya 2 orang yang lahir pada bulan yang tidak sama,
maka banyaknya kejadian Ac adalah (12 x 12 x 12 – 12 x 11 x 10) cara
maka dapat disimpulkan, peluang menemukan paling sedikit ada 2 orang dari 3
orang, yang lahir pada bulan yang sama adalah ...

𝑛(𝐴𝑐 ) 12 𝑥 12 𝑥 12 − 12 𝑥 11 𝑥 10 17
𝑃(𝐴𝑐 ) = = =
𝑛(𝑆) 12 𝑥 12 𝑥 12 72

Soal & Pembahasan nomor 10 :


Misalkan S menyatakan himpunan semua faktor positif dari 20202 . Sebuah
bilangan diambil secara acak dari S. Peluang bilangan yang terambil itu
merupakan kelipatan 2020 adalah ....

Pembahasan :
2020 = 22 x 5 x 101
Banyaknya faktor positif 20202 adalah (2 + 1) (1 + 1) (1 + 1) = 12

Faktor positif dari 20202 = 24 x 52 x 1012


Banyaknya faktor positif 20202 adalah (4 + 1) (2 + 1) (2 + 1) = 45

Dengan demikian peluang bilangan yang terambil merupakan kelipatan dari


2020, adalah :
12
𝑃=
45

166 | K O M B I N A T O R I K A
Olimpiade Matematika SMA 2020

TANTANGAN OLIMPIADE

TANTANGAN 1
1. Jika x dan y adalah bilangan bulat sedemikian sehingga 7x terletak diantara
208 dan 349 sedangkan 8y terletak diantara – 601 dan – 419, nilai terbesar xy =

1 𝑎√2+𝑏√3+𝑐√30
2. Jika disajikan dalam bentuk maka 𝑎 + 𝑏 + 𝑐 = ....
√2+√3+√5 12
Pembahasan :
1 1
=
√2+√3+√5 (√2+√3)+(√5)

1 (√2+√3)−(√5) (√2+√3)−(√5)
= 2
(√2+√3)+(√5) (√2+√3)−(√5) (√2+√3) −5

(√2+√3)−(√5) (√2+√3)−(√5)
2 =
(√2+√3) −5 2+3+2√6−5

1 √2+√3−√5
=
√2+√3+√5 2 √6

1 √2+√3−√5 √6
= ( 6)
√2+√3+√5 2 √6 √

1 √18+√12−√30
=
√2+√3+√5 12

1 3√2+2√3−√30
=
√2+√3+√5 12

1 𝑎√2+𝑏√3−𝑐 √30
=
√2+√3+√5 12

maka 𝑎 = 3, 𝑏 = 2 𝑑𝑎𝑛 𝑐 = −1
𝑎+𝑏+𝑐 =4

167 | T A N T A N G A N O L I M P I A D E
Olimpiade Matematika SMA 2020

3. Jika 𝑥 + 𝑎𝑦 = 𝑏 , 2𝑥 + 𝑏𝑦 = 𝑎, dan 3𝑥 + 𝑎𝑏𝑦 = 4 maka 𝑎 + 𝑏 = ....


Pembahasan :
Penjumlahan 𝑥 + 𝑎𝑦 = 𝑏 dan 2𝑥 + 𝑏𝑦 = 𝑎 menghasilkan 3𝑥 + (𝑎 + 𝑏)𝑦 = 𝑎 +
𝑏. Jika persamaan ini disubstitusi dengan persamaan ketiga,
3𝑥 + 𝑎𝑏𝑦 = 4 atau 3𝑥 = 4 − 𝑎𝑏𝑦 maka
(4 − 𝑎𝑏𝑦) + (𝑎 + 𝑏)𝑦 = 𝑎 + 𝑏

(𝑎 + 𝑏)𝑦 − (𝑎 + 𝑏) = 𝑎𝑏𝑦 − 4

(𝑎 + 𝑏)(𝑦 − 1) = 𝑎𝑏𝑦 − 4

𝑎𝑏𝑦 − 4
𝑎+𝑏 =
𝑦−1
agar diperoleh nilai a + b bulat maka nilai ab haruslah 4 sehingga
4𝑦−4
𝑎+𝑏 = dengan 𝑎𝑏 = 4
𝑦−1
4(𝑦 − 1)
𝑎+𝑏 = =4
𝑦−1

4. Bilangan 6 angka a1989b habis dibagi oleh 72. Maka nilai a = ...

5. Bilangan dua digit terbesar yang dapat membagi habis 248 − 1 adalah ....

6. Jika 𝑎1 , 𝑎2 , 𝑎3 , …. adalah bilangan-bilangan cacah berlainan, sehingga


memenuhi 2𝑎1 + 2𝑎1 + 2𝑎1 + 2𝑎1 + ⋯ + 2𝑎1 = 2020 , maka nilai dari 𝑎1 + 𝑎2 +
𝑎3 + … . +𝑎𝑛 adalah ....

7. Pak Budi memiliki sawah berbentuk huruf L. Jika diketahui bahwa sawahnya
pak Budi hanya memiliki sisi yang panjangnya 5 meter dan 10 meter dan
semua sudut sawahnya siku-siku, luas sawah pak Budi adalah ....... m2.

8. Jika 𝑎2 − 𝑎 + 7 = 0 maka nilai dari 2𝑎2 − 16𝑎 + 1 = ....

𝑎𝑥
9. Jika 𝑓(𝑥) = 𝑎𝑥 + maka 𝑓(𝑥) + 𝑓(1 − 𝑥)
√𝑎

10. Suatu barisan 𝑥1 , 𝑥2 , 𝑥3 , 𝑥4 , … . 𝑥𝑛 terdefinisi untuk 𝑥1 = 2


1 2
𝑥𝑛+1 = (1 + ) 𝑥𝑛 +
𝑛 𝑛
Untuk setiap bilangan asli 𝑛. Nilai 𝑥2020 adalah ...

168 | T A N T A N G A N O L I M P I A D E
Olimpiade Matematika SMA 2020

TANTANGAN 2
11. Agus mengukur tiga sisi suatu persegi panjang dan mendapatkan panjang
totalnya 68 cm. Iwan mengukur tiga sisi suatu persegi panjang yang sama
dan mendapatkan panjang totalnya 52 cm. Maka keliling persegi panjang
tersebut adalah ...cm
Pembahasan :
Pada persegi panjang hanya ada 2 sisi ukuran panjang dan 2 sisi ukuran
lebar. Menurut perhitungan Agus, 68 cm adalah 2 sisi panjang dan 1 sisi
lebar, sedangkan menurut Iwan, 52 cm adalah 2 sisi pendek dan 1 sisi
panjang. Jika a = sisi panjang dan b = sisi lebar, maka :
2a + b = 68
a + 2b = 52
Hasil penjumlahan kedua persamaan 3a + 3b = 120 atau a + b = 40
Keliling persegi panjang itu adalah = 2a + 2b = 80 cm

12. Panjang suatu persegi panjang adalah 2 cm lebih panjang dari lebarnya. Jika
lebarnya bertambah 2 cm dan panjangnya bertambah 3 cm maka luasnya
menjadi dua kali semula. Luar persegi panjang semula adalah .... cm2.

13. Diberikan persamaan :


√2𝑥 2 + 2𝑥 + 3 + √2𝑥 2 + 2 = √3𝑥 2 + 2𝑥 − 1 + √𝑥 2 + 6
Tentukan jumlah semua solusi yang mungkin dari persamaan diatas !
Pembahasan :

√2𝑥 2 + 2𝑥 + 3 + √2𝑥 2 + 2 = √3𝑥 2 + 2𝑥 − 1 + √𝑥 2 + 6


2 2
(√2𝑥 2 + 2𝑥 + 3 + √2𝑥 2 + 2) = ( √3𝑥 2 + 2𝑥 − 1 + √𝑥 2 + 6)

4𝑥 2 + 2𝑥 + 5 + 2√(2𝑥 2 + 2𝑥 + 3) (2𝑥 2 + 2) = 4𝑥 2 + 2𝑥 + 5 + 2√3𝑥 2 + 2𝑥 − 1) (𝑥 2 + 6)

2√(2𝑥 2 + 2𝑥 + 3) (2𝑥 2 + 2) = 2√(2𝑥 2 + 2𝑥 − 1) (𝑥 2 + 6)

(2𝑥 2 + 2𝑥 + 3) (2𝑥 2 + 2) = (2𝑥 2 + 2𝑥 − 1) (𝑥 2 + 6)


4𝑥 4 + 4𝑥 3 + ⋯ . . = 2𝑥 4 + 2𝑥 3 + ⋯.
2𝑥 4 + 2𝑥 3 + ⋯ … = 0
2
Jumlah semua solusi 𝑥1 + 𝑥2 + 𝑥3 + 𝑥4 = − 2 = −1

169 | T A N T A N G A N O L I M P I A D E
Olimpiade Matematika SMA 2020

14. Sembilan kartu bernomor dari 1 sampai 9 berturut-turut dibagikan kepada 4


anak yang masing-masing mendapat 2 kartu. Jumlah angka pada dua kartu
yang diberikan pada anak-anak adalah 7 untuk Wahyu, 10 untuk Budi, 11
untuk Nanang, dan 12 untuk Aries. Maka kartu yang tidak dibagikan adalah
kartu bernomor .....

𝑎 𝑐 𝑏 𝑑
15. Jika 𝑎, 𝑏, 𝑐 dan 𝑑 adalah bilangan real yang memenuhi 𝑏 + 𝑑 = 2 dan +𝑎=4
𝑐
𝑎 𝑏 𝑐 𝑑
Maka nilai dari + 𝑑 + 𝑎 + 𝑏 adalah ....
𝑐

16. Diberikan n adalah bilangan bulat positif yang mungkin dan d adalah 1 digit
dengan dasar 10 (basis 10). Tentukan n jika :
𝑛
= 0, 𝑑25𝑑25𝑑25 … …
810

17. Sisa dari 270 + 370 apabila dibagi 13 adalah ....

18. Jika 𝑓(𝑥) = 𝑥 4 + 𝑥 3 + 𝑥 2 + 𝑥 + 1, tentukan solusi dari (𝑥 − 1)𝑓(𝑥) = 31 !

19. Place the numbers 4, 5, 6, 7, 8 and 9 in the boxes


so that each side of the triangle add to 18 !

20. Aji melakukan percobaan pelemparan sekeping uang logam dengan


permukaan Gambar dan Angka. Pada Percobaan pertama 40 kali pelemparan,
munculnya Gambar dan Angka adalah 5 : 3. Kemudian Aji melanjutkan
percobaan kedua dengan melakukan 60 kali pelemparan keping mata uang,
ternyata hasilnya Gambar muncul 2 kali lebih banyak daripada Angka. Maka
dari kedua percobaan ini, banyaknya gambar yang muncul adalah .....

170 | T A N T A N G A N O L I M P I A D E
Olimpiade Matematika SMA 2020

TANTANGAN 3

√35 + √21 + √15 + 5


21. Bentuk sederhana dari adalah ...
√3 + 2√5 + √7
Pembahasan :

√35 + √21 + √15 + 5 √7 √5+√7 √3+√5 √3+√5 √5


=
√3 + 2√5 + √7 √3 + 2√5 + √7

√35 + √21 + √15 + 5 (√7 + √5) (√3 + √5)


=
√3 + 2√5 + √7 (√7 + √5)+(√3 + √5)

√3 + 2√5 + √7 (√7 + √5)+(√3 + √5)


=
√35 + √21 + √15 + 5 (√7 + √5) (√3 + √5)

√3 + 2√5 + √7 1 1
= +
√35 + √21 + √15 + 5 √7 + √5 √5 + √3

√3 + 2√5 + √7 √7− √5 √5− √3


= +
√35 + √21 + √15 + 5 2 2

√3 + 2√5 + √7 √7− √3
=
√35 + √21 + √15 + 5 2

√3 + 2√5 + √7 √7− √3 √7+√3


=( ) ( )
√35 + √21 + √15 + 5 2 √7+√3

√3 + 2√5 + √7 2
=
√35 + √21 + √15 + 5 √7+√3

√35 + √21 + √15 + 5 √7+√3


=
√3 + 2√5 + √7 2

√143+√165+√195+13
22. Bentuk sederhana dari adalah ....
√11+2√13+√15

23. Jika diketahui :


𝑏 + 3𝑎 = 302
𝑑 + 2𝑐 = 242
𝑎 + 3𝑏 + 2𝑑 = 832
Maka nilai dari 𝑎 + 𝑏 + 𝑐 + 𝑑 = .....

171 | T A N T A N G A N O L I M P I A D E
Olimpiade Matematika SMA 2020

24. Dengan 4 pekerja dapat dihasilkan 350 batako selama 10 hari. Banyak batako
yang dihasilkan oleh 8 pekerja selama 4 hari adalah …

25. Dari angka 3, 5, 6, 7 dan 8 dibuat nomor kupon yang terdiri dari 5 angka yang
berbeda. Kupon-kupon itu diurutkan dari nomor terendah sampai nomor
tertinggi. Kupon dengan urutan ke 75 adalah kupon dengan nomor seri ...

26. Dari 4 pria dan 3 wanita, mereka duduk di meja bundar. Peluang tidak ada
wanita duduk berdampingan adalah ...
Pembahasan :
Dari 4 pria dan 3 wanita, mereka duduk di meja bundar. Agar tidak ada wanita
duduk berdampingan, 4 pria duduk lebih dulu.
Banyaknya cara 4 pria duduk di meja bundar adalah 3 ! = 6 cara
Ketika 4 pria semua sudah duduk di meja bundar, ada 4 tempat duduk antar
dua pria berdekatan yang bisa ditempati 3 wanita.
Wanita pertama memiliki 4 pilihan untuk duduk
Wanita kedua memiliki 3 pilihan untuk duduk
Wanita ketiga memiliki 2 pilihan untuk duduk
Banyaknya cara 3 wanita duduk dengan syarat itu adalah 4 x 3 x 2 = 24
Jadi akan ada satu tempat kosong atau ada dua pria duduk bersebelahan

Jika A adalah benyaknya kejadian yang dinginkan, maka 𝑛(𝐴) = 6 𝑥 24 = 144


Banyaknya kemungkinan susunan 7 orang duduk di meja bundar = 6 ! = 720

Maka Peluang kejadian A dinyatakan dengan


𝑛(𝐴) 144 1
𝑃(𝐴) = = =
𝑛(𝑆) 720 5

27. Jika (𝑥, 𝑦, 𝑧) 𝐵𝑢𝑙𝑎𝑡 𝑝𝑜𝑠𝑖𝑡𝑖𝑓 sehingga 𝑥𝑦 = 6 , 𝑦𝑧 = 12 dan 𝑥𝑧 = 8 maka nilai
𝑥2+ 𝑦2+ 𝑧 2
dari adalah ...
𝑥𝑦𝑧

(𝑎+𝑏)(𝑏+𝑐)(𝑎+𝑐)
28. Jika 𝑎 + 𝑏 + 𝑐 = 0 maka = ....
𝑎𝑏𝑐

29. Pada kelompok bilangan yang disusun sebagai berikut :


(1) , (3 , 9, 27) , (81, 243, 729, 2187, 6561) , .....
Suku tengah kelompok ke-12 adalah 81𝑥 . Berapa nilai x ?

30. Misalkan 𝑎, 𝑏, 𝑐 membentuk barisan geometri. Jika 𝑎 + 𝑏 + 𝑐 = 26 dan 𝑎2 +


𝑏 2 + 𝑐 2 = 364 , maka 𝑏 =

172 | T A N T A N G A N O L I M P I A D E
Olimpiade Matematika SMA 2020

TANTANGAN 4

31. (𝑥 + 𝑦 + 𝑧)𝑥 = 4 , (𝑥 + 𝑦 + 𝑧)𝑦 = 2 dan (𝑥 + 𝑦 + 𝑧) 𝑧 = 32.


Maka dari 𝑧 − 𝑦 − 𝑥 =

1 1
32. Untuk 𝑥 real positif, jika 𝑥 2 + 𝑥 2 = 14 maka nilai dari 𝑥 5 + 𝑥 5 adalah ....
Pembahasan :
1 2 1 1
(𝑥 + ) = 𝑥 2 + 2 + 2 𝑥 .
𝑥 𝑥 𝑥
2
1 1
(𝑥 + ) = 𝑥 2 + 2 + 2 = 14 + 2 = 16
𝑥 𝑥
1
maka 𝑥 + = 4 , untuk x real positif
𝑥
1 3 1 1 1
(𝑥 + ) = 𝑥 3 + 3 + 3 𝑥 . (𝑥 + )
𝑥 𝑥 𝑥 𝑥
1
(4)3 = 𝑥 3 + + 3 (1) (4)
𝑥3
1
𝑥3 + = 64 − 12 = 52
𝑥3

5
1 3
1 2
1 1 2 1
𝑥 + 5 = (𝑥 + 3 ) (𝑥 + 2 ) − (𝑥 . ) (𝑥 + )
𝑥 𝑥 𝑥 𝑥 𝑥
1
𝑥5 + = (52)(14) − (1 )2 (4) = 724
𝑥5

33. Jika diketahui :


𝑎+𝑏+𝑐 =3
𝑎2 + 𝑏 2 + 𝑐 2 = 5
𝑎3 + 𝑏 3 + 𝑐 3 = 7
Tentukan nilai dari 𝑎4 + 𝑏 4 + 𝑐 4 !

34. Jika 𝑎 + 𝑏 = 10 , 𝑏 + 𝑐 = 20 dan 𝑎 + 𝑐 = 30, maka 𝑎2 + 𝑏 2 + 𝑐 2 = ....

35. Berapa jumlah digit bilangan pada 828 580

𝑛3 −1
36. Banyaknya bilangan positif 𝑛 sehingga merupakan bilangan prima ?
5

173 | T A N T A N G A N O L I M P I A D E
Olimpiade Matematika SMA 2020

37. Berapakah bilangan terakhir pada 4242 ?

38. Tentukan sisa pembagian 6𝑛 − 5𝑛 + 7 oleh 5 !


Pembahasan :
Untuk 𝑛 = 1, maka 6𝑛 − 5𝑛 + 7 = 61 − 5(1) + 7 = 8
Karena 8 ≡ 3 (𝑚𝑜𝑑 5) maka 6𝑛 − 5𝑛 + 7 3 (mod 5)
Jadi 6𝑛 − 5𝑛 + 7 oleh 5 akan bersisa 3

39. Tentukan solusi dari 43𝑥 + 5𝑦 = 250 jika (𝑥 , 𝑦) bilangan bulat positif
Pembahasan :
gcd(43 , 5) = 1 dan 1 membagi 250 maka :

250 − 5𝑦 215 + 35 − 5𝑦 35 − 5𝑦
𝑥= = =5+
43 43 43
35−5𝑦
Jika 𝑝 = , sehingga 𝑥 = 5 + 𝑝
43

250 − 43𝑥 250 − 43(5 + 𝑝) 35 − 43𝑝 3𝑝


𝑦= = = = 7 − 8𝑝 +
5 5 5 5
3𝑝 3𝑟 𝑟 2𝑞
Jika 𝑞 = = =𝑟+ dan 𝑟 = maka :
5 2 2 3
𝑝 = 𝑞 + 𝑟 dan dengan mengambil 𝑟 = 2𝑡
𝑞 = 3𝑡 dan 𝑝 = 5𝑡
dengan demikian : 𝑥 = 5 + 𝑝 = 5 + 5𝑡 dan 𝑦 = 7 − 43𝑡

40. Diketahui 𝑓(𝑥) = (1 + 𝑎𝑥)5 (1 + 𝑏𝑥)4 dengan 𝑎 dan 𝑏 bilangan positif


sedemikian sehingga koefisien 𝑥 2 adalah 62. Maka nilai 𝑎 + 𝑏 adalah ....
Pembahasan :
(1 + 𝑎𝑥)5 = 1 + 5𝑎𝑥 + 10𝑎𝑥 2 + ⋯
(1 + 𝑏𝑥)4 = 1 + 4𝑏𝑥 + 6𝑏𝑥 2 + ⋯
𝑓(𝑥) = (1 + 𝑎𝑥)5 (1 + 𝑏𝑥)5
𝑓(𝑥) = (1 + 5𝑎𝑥 + 10𝑎𝑥 2 + ⋯ )(1 + 4𝑏𝑥 + 6𝑏𝑥 2 + ⋯ )
𝑓(𝑥) = ⋯ + 10𝑎𝑥 2 + 6𝑏𝑥 2 + 20𝑎𝑏𝑥 2 + ⋯
maka :
10𝑎 + 6𝑏 + 20𝑎𝑏 = 62
5𝑎 + 3𝑏 + 10𝑎𝑏 = 31
Dengan cara mendaftar 𝑎 dan 𝑏 maka diperoleh nilai dengan 𝑎 = 1 dan 𝑏 = 2,
sehingga nilai 𝑎 + 𝑏 = 3

174 | T A N T A N G A N O L I M P I A D E
Olimpiade Matematika SMA 2020

KUNCI JAWABAN TANTANGAN OLIMPIDE

TANTANGAN 1 TANTANGAN 3
1. – 1508 √7+√3
21.
2. 4 2
1
3. 4 22. 2 (√15 + √11)
4. 3 23. 480
5. 91 24. 280
6. 47 25. 73658
7. 75 26. 5
1

8. 15 29
27. 24
9. 1
10. 8087 28. – 1
29. 33
30. 6
TANTANGAN 2
11. 80 TANTANGAN 4
12. 35 31. 1
13. – 1 32. 724
14. 5 33. 9
15. 8 34. 500
16. 750 35. 82
17. 0 36. 6
18. 2 37. 4
19. – 38. 3
20. 65 39. (5 + 5𝑡 , 7 − 43𝑡)
40. 3

175 | T A N T A N G A N O L I M P I A D E
Olimpiade Matematika SMA 2020

TANTANGAN OLIMPIADE 5
Mengenal Penyelenggaraan jenis kompetisi dan Olimpiade Matematika di dalam
dan di Luar negeri.

OSK Olimpiade Tingkat Kabupaten/Kotamadya


OSP Olimpiade Tingkat Propinsi
OSN Olimpiade Tingkat Nasional

AHSME American High School Mathematics Examination


AIME American Invitational Mathematics Examination
APMO Asia Pacific Mathematics Olympiad
ASUMO Olympics Mathematical Competitions of All the Soviet Union
BMO British Mathematical Olympiad
CHNMO China Mathematical Olympiad
CHNMOL China Mathematical Competition for Secondary Schools
CHINA China Mathematical Competitions for Secondary Schools
CMO Canada Mathematical Olympiad
HUNGARY Hungary Mathematical Competition
IMO International Mathematical Olympiad
JAPAN Japan Mathematical Olympiad
KIEV Kiev Mathematical Olympiad
MOSCOW Moscow Mathematical Olympiad
NORTH EUROPE North Europe Mathematical Olympiad
RUSMO All-Russia Olympics Mathematical Competitions
SSSMO Singapore Secondary Schools Mathematical Olympiads
SMO Singapore Mathematical Olympiads
SSSMO Singapore Secondary Schools Mathematical Olympiads for Junior
Section
UKJMO United Kingdom Junior Mathematical Olympiad
USAMO United States of American Mathematical Olympiad

176 | T A N T A N G A N O L I M P I A D E
Olimpiade Matematika SMA 2020

1. (OSP 2006) Luas sisi-sisi sebuah balok dalam cm2 adalah 486, 486, 243, 243,
162, 162. Volume balok tersebut adalah ....

2. (OSP 2002) Tinjau peramaan yang berbentuk 𝑥 2 + 𝑏𝑥 + 𝑐 = 0. Berapa


banyakkah persamaan demikian yang memiliki akar-akar real, jika koefisien
b dan c hanya boleh dipilih dari {1, 2, 3, 4, 5, 6}

3. (OSK 2006) Pada sebuah barisan aritmatika, nilai suku ke-25 tiga kali nilai
suku ke-5. Suku yang bernilai dua kali nilai suku pertama adalah suku ke ....

4. (OSK 2013) Misalkan bahwa 𝑓(𝑥) = 𝑥 5 + 𝑎𝑥 4 + 𝑏𝑥 3 + 𝑐𝑥 2 + 𝑑𝑥 + 𝑒


Dan 𝑓(1) = 𝑓(2) = 𝑓(3) = 𝑓(4) = 𝑓(5). Berapakah nilai 𝑎 ?

5. (OSP 2008) carilah semua pasangan bilangan asli (𝑥 , 𝑛) yang memenuhi


1 + 𝑥 + 𝑥 2 + 𝑥 3 + ⋯ . . + 𝑥 𝑛 = 40

6. (OSP 2006) Jika |𝑥| + 𝑥 + 𝑦 = 10 dan 𝑥 + |𝑦| − 𝑦 = 12, maka 𝑥 + 𝑦 = ....

7. (OSP 2007) Tujuh belas permen dikemas ke dalam kantong-kantong sehingga


banyaknya permen dalam setiap dua kantong berselisih paling banyak 1.
Banyak cara mengemas permen tersebut ke dalam paling sedikit dua kantong
adalah ...

8. (OSP 2007) Himpunan semua bilangan asli n sehingga 6𝑛 + 30 adalah


kelipatan 2𝑛 + 1 adalah ....

43
9. (OSP 2002) Berapakah sisa pembagian 4343 oleh 100 ?

10. (OSP 2003) Misalkan 𝑥, 𝑦, 𝑧 tiga bilangan asli berbeda. Faktor persekutuan
terbesar ketiganya adalah 12, sedangkan kelipatan persekutuan terkecil
ketiganya adalah 840. Berapakah nilai terbesar 𝑥 + 𝑦 + 𝑧 ?

11. (OSP 2004) Berapakah banyaknya barisan bilangan bulat tak negatif (𝑥 , 𝑦, 𝑧)
yang memenuhi persamaan 𝑥 + 𝑦 + 𝑧 = 99 ?

12. (OSP 2005) Banyaknya pasangan bilangan bulat (𝑥 , 𝑦) yang memenuhi


persamaan 2𝑥𝑦 − 5𝑥 + 𝑦 = 55 adalah ...

177 | T A N T A N G A N O L I M P I A D E
Olimpiade Matematika SMA 2020

13. (OSP 2009) Tiga dadu berwarna hitam, merah dan putih dilempar bersama-
sama. Banyaknya cara sehingga jumlah tiga mata dadu berjumlah 8 adalah ...

14. (OSK 2011) Bilangan bulat positif terkecil a sehingga 2a + 4a + 6a + ... + 200a
merupakan kuadrat sempurna adalah ⋅⋅⋅⋅⋅⋅⋅⋅

15. (OSK 2017) Diketahui x − y = 10 dan xy = 10. Nilai 𝑥 4 − 𝑦 4 adalah ....

16. (OSK 2007) Bila n adalah bilangan asli sehingga 3𝑛 adalah faktor dari 33!,
tentukan nilai terbesar dari n

17. (OSP 2018) Bilangan 1, 2, 3, ⋅⋅⋅, 9 disusun melingkar secara acak. Buktikan
bahwa ada tiga bilangan berdekatan yang jumlahnya lebih besar dari 15

18. (OSP 2018)Banyaknya pasangan terurut bilangan bulat (𝑎 , 𝑏) sehingga


𝑎2 + 𝑏 2 = 𝑎 + 𝑏 adalah ...

19. (AIME 2019) Akar-akar persamaan dapat ditulis


3 3
√𝑎+ √𝑏+1
dalam bentuk dengan 𝑎, 𝑏, 𝑐 bilangan bulat positif. Tentukan nilai
𝑐
𝑎+𝑏+𝑐!

20. (AIME 1986) (√5 + √6 + √7)(√5 + √6 − √7)(√5 − √6 + √7)(−√5 + √6 + √7)

21. (AHSME 1996) Barisan 1, 2, 1, 2, 2, 1, 2, 2, 2, 1, 2, 2, 2, 2, 1, 2, 2, 2, 2, 2, 1, 2, ⋅⋅⋅


memiliki blok angka 1 yang berisi n buah angka 2 pada blok ke-n. Tentukan
jumlah 1234 bilangan pertama.

22. (AIME 2012) Tentukan jumlah bilangan bulat positif dengan tiga angka yang
tidak harus berbeda, abc dengan a  0 , c  0 sehingga abc dan cba adalah
kelipatan dari 4.

23. (AIME 2012) Sembilan orang duduk untuk makan malam di mana ada tiga
pilihan makanan. Tiga orang memesan makan daging sapi, tiga memesan
makan ayam, dan tiga memesan makan ikan. Pelayan menyajikan sembilan
makanan dalam urutan acak. Temukan sejumlah cara di mana pelayan dapat
menyajikan jenis makanan untuk sembilan orang sehingga tepat satu orang
menerima jenis makanan yang dipesan oleh orang itu.

178 | T A N T A N G A N O L I M P I A D E
Olimpiade Matematika SMA 2020

24. (AHSME 1998) Misalkan fungsi 𝑓(𝑥) memenuhi sifat bahwa untuk sebarang
dua bilangan 𝑥 , 𝑦 sehingga 𝑓(𝑥 + 𝑦) = 𝑥 + 𝑓(𝑦). Jika 𝑓(0) = 2 , maka
tentukan nilai 𝑓(1998)

25. (AMC 2002) Untuk bilangan real tidak nol a, b, dan c, didefinisikan
𝑎𝑏𝑐
(𝑎, 𝑏, 𝑐) =
𝑎+𝑏+𝑐
Tentukan nilai dari (2, 4, 6)

26. (HSMC-USC, 2005) Misal 𝑓(𝑥) adalah fungsi, untuk setiap bilangan real 𝑥.
𝑓(𝑥) + 2𝑓(−𝑥) = sin 𝑥
𝜋
Berapakah nilai dari 𝑓 ( 2 ) ?

27. (IMO 2017) Misalkan R menyatakan himpunan bilangan real. Tentukan


fungsi 𝑓 ∶ 𝑅 → 𝑅 sehingga untuk semua bilangan real x dan y memenuhi
𝑓(𝑓(𝑥)𝑓(𝑦)) + 𝑓(𝑥 + 𝑦) = 𝑓(𝑥𝑦)

28. (IMO 2018) Tentukan semua bilangan bulat 𝑛 ≥ 3 sehingga terdapat bilangan
real 𝑎1 , 𝑎2 , 𝑎3 , … . 𝑎𝑛+2 , sehingga 𝑎𝑛+1 = 𝑎1 , 𝑎𝑛+2 = 𝑎2
𝑎𝑖 𝑎𝑖+1 + 1 = 𝑎𝑖+2
untuk setiap 𝑖 = 1, 2, 3, 4, … . . 𝑛

29. (OMITS 2012) Di suatu pagi yang cerah, Meyta mencari banyaknya bilangan
komposit dua digit yang habis dibagi digit-digitnya. Berapa banyak bilangan
yang akan didapatkan oleh Meyta ?

30. (OMITS 2012) Banyaknya bilangan yang tidak lebih dari 2012 dan jika dibagi
dengan 2, 3, 4, 5 dan 7 akan bersisa 1 adalah ...

31. (AIME 2019) Diketahui 𝑁 = 9 + 99 + 999 + ⋯ … + ⏟


999 … .999
321 𝑑𝑖𝑔𝑖𝑡
Hitung jumlah semua digit N !

32. (AIME 2018) Let S be the number of ordered pairs of integers (a , b) with
1 ≤ a ≤ 100 and b ≥ 0 such that the polynomial 𝑥 2 + 𝑎𝑥 + 𝑏 can be factored into
the product of two (not necessarily distinct) linier factors with integer
coefficients. Find thr remainder when S is divided by 1000

179 | T A N T A N G A N O L I M P I A D E
Olimpiade Matematika SMA 2020

33. (AIME 2017) When each of 702, 787 and 855 is divided by the positive
integer , the remainder is always the positive integer . When each
of 412, 722, and 815 is divided by the positive integer , the remainder is
always the positive integer 𝑠 ≠ 𝑟. Find 𝑚 + 𝑛 + 𝑟 + 𝑠

34. (AIME 1983) Diketahui . dan . adalah bilangan kompleks yang memenuhi
𝑤 2 + 𝑦 2 = 7 dan 𝑤 3 + 𝑧 3 = 10, maka nilai terbesar 𝑤 + 𝑧 adalah ....

35. (BULGARIA 1995) Carilah pasangan bilangan bulat (𝑥 , 𝑦) sehingga bentuk


𝑥2+ 𝑦2
adalah bilangan bulat yang habis dibagi 1995
𝑥−𝑦

36. (BULGARIA 1995) Find the number of integers 𝑛 > 1 for which the number
𝑎25 − 𝑎 divisible by 𝑛 for each integer 𝑎

37. (SLOVAKIA 1995) Tentukan semua nilai p real pada persamaan :


𝑥 3 − 2𝑝(𝑝 + 1)𝑥 2 + (𝑝4 + 4𝑝3 − 1)𝑥 − 3𝑝3 = 0
yang memiliki tiga akar yang merupakan panjang sisi-sisi segitiga siku-siku.

38. (YUNANI 1995) Tentukan semua bilangan bulat positif 𝑛 sehingga


−54 + 55 + 5𝑛 merupakan bilangan kuadrat sempurna

39. (CHINA 1995) Misalnya N = {1, 2, 3, 4, … . }. diketahui 𝑓(𝑛) = 1 untuk setiap 𝑛


dalam N. Jika :
(a) 3 𝑓(𝑛) 𝑓(2𝑛 + 1) = 𝑓(2𝑛) (1 + 3 𝑓(𝑛))
(b) 𝑓(2𝑛) < 6 𝑓(𝑛)
Carilah semua solusi persamaan 𝑓(𝑘) + 𝑓(𝑙) = 293 , 𝑘 < 𝑙

40. (HUNGGARIA 1995) Buktikan bahwa jika 𝑥, 𝑦, 𝑧 bilangan real berbeda yang
𝑥 𝑦 𝑧
memenuhi + + = 0,
𝑦−𝑧 𝑧−𝑥 𝑥−𝑦
𝑥 𝑦 𝑧
maka berlaku bahwa + + (𝑥−𝑦)2 = 0
(𝑦−𝑧)2 (𝑧−𝑥)2

41. (INDIA 1995) Tentukan semua bilangan positif (𝑥 , 𝑦) sehingga memenuhi


7𝑥 − 3𝑦 = 4

42. (INDIA 1995) Tentukan semua bilangan positif 𝑥, 𝑦, 𝑧, 𝑝 dengan 𝑝 bilangan


prima yang memenuhi persamaan 𝑥 𝑝 + 𝑦 𝑝 = 𝑝 𝑧

180 | T A N T A N G A N O L I M P I A D E
Olimpiade Matematika SMA 2020

43. (IRAN 1995) Determine with proof all real – valued functions 𝑓(𝑥) satisfying
the equation
𝑥 𝑓(𝑥) − 𝑦 𝑓(𝑦) = (𝑥 − 𝑦)𝑓(𝑥 + 𝑦)
for all real numbers 𝑥 , 𝑦.

44. (KOREA 1995) For a given positive 𝑚, find all pairs (𝑛, 𝑥, 𝑦) of positive
integers such that 𝑚 , 𝑛 are relatively prime and (𝑥 2 + 𝑦 2 )𝑚 = (𝑥𝑦)𝑛 , where
𝑛, 𝑥, 𝑦 can be represented in terms of 𝑚

45. (RUSIA 1995) Consider all quadratic functions 𝑓(𝑥) = 𝑎𝑥 2 + 𝑏𝑥 + 𝑐 such that
𝑎 < 𝑏 and 𝑓(𝑥) ≥ 0 for all 𝑥.
(𝑎+𝑏+𝑐)
Determine the minimal value of the expression
(𝑏−𝑎)

46. (JEPANG 1995) Find all real – valued functions 𝑓 defined on nonzero
1 1
numbers such that 𝑓(−𝑥) + 𝑓 (𝑥) = 𝑥 , 𝑥 ≠ 0
𝑥

47. (ITALY 1996) Prove that the equation 𝑎2 + 𝑏 2 = 𝑐 2 + 3 has infinitely many
integer solutions {𝑎, 𝑏, 𝑐}

48. (JEPANG 1996) Let 𝑚 and 𝑛 be positive integers with gcd(𝑚 , 𝑛) = 1.


Hitung gcd(5𝑚 + 7𝑚 , 5𝑛 + 7𝑛 ) !

49. (POLANDIA 1996) Tentukan semua pasangan (𝑛 , 𝑟) dengan 𝑛 bilangan bulat


positif dan 𝑟 adalah bilangan real, sehingga suku banyak (𝑥 + 1)𝑛 − 𝑟 habis
dibagi oleh 2𝑥 2 + 2𝑥 + 1

50. (RUSIA 1996) Let 𝑥, 𝑦, 𝑝, 𝑛, 𝑘 be natural numbers such that 𝑥 𝑛 + 𝑦 𝑛 = 𝑝𝑘


Prove that if 𝑛 > 1 is odd, and 𝑝 is an odd prime, then 𝑛 is a power of 𝑝.

51. (SPANYOL 1996) Tentukan solusi real persamaan √𝑥 2 − 𝑝 + 2√𝑥 2 − 1 = 𝑥


untuk setiap nilai 𝑝.

52. (INGGRIS 1996) Suatu fungsi didefinisikan sebagai 𝑓(1) = 1996


𝑓(1) + 𝑓(2) + 𝑓(3) + ⋯ . . +𝑓(1996) = 𝑛2 𝑓(𝑛) , (𝑛 > 1)
Tentukan 𝑓(1996) !

181 | T A N T A N G A N O L I M P I A D E
Olimpiade Matematika SMA 2020

53. (INGGRIS 1996) Misalkan 𝑎, 𝑏, 𝑐 adalah bilangan real positif


(a) Buktikan 4(𝑎3 + 𝑏 3 ) ≥ (𝑎 + 𝑏)3
(b) Buktikan 9(𝑎3 + 𝑏 3 + 𝑐 3 ) ≥ (𝑎 + 𝑏 + 𝑐)3

54. (VIETNAM 1996) Selesaikan sistem persamaan berikut :


1 1
√3𝑥 (1 + 𝑥+𝑦) = 2 dan √7𝑦 (1 − 𝑥+𝑦) = 4√2

55. (AIME 2010) Positive integers , , , and satisfy 𝑎 > 𝑏 > 𝑐 > 𝑑,
𝑎 + 𝑏 + 𝑐 + 𝑑 = 2010 , and 𝑎2 − 𝑏 2 + 𝑐 2 − 𝑑 2 = 2010 . Find the number of
possible values of .

56. (AIME 2001) Temukan jumlah semua bilangan bulat dua digit positif yang
dapat dibagi oleh masing-masing digit mereka.

57. (AIME 2019) Sebuah tim sepak bola memiliki 22 pemain yang tersedia. Satu
team utama 11 pemain memulai permainan, sementara 11 lainnya tersedia
sebagai pengganti. Selama pertandingan, pelatih melakukan sebanyak 3
pergantian pemain, di mana salah satu dari 11 pemain dalam permainan
digantikan oleh salah satu pemain pengganti. Tidak ada pemain yang
dikeluarkan dari permainan, mereka dapat memasuki kembali permainan
dan pemain yang menggatikan dapat di ganti kembali oleh pemain pengganti
yang lain. Tidak ada dua pergantian pemain yang bisa terjadi secara
bersamaan. Para pemain yang terlibat dan urutan masalah pergantian
pemain. Misalkan n adalah banyaknya cara pelatih dapat membuat
pergantian pemain selama pertandingan (termasuk kemungkinan tidak
membuat pergantian pemain). Temukan sisanya ketika n dibagi dengan 1000

58. (AIME 2001) Sebuah dadu yang seimbang ditos empat kali. Probabilitas
bahwa masing-masing dari tiga gulungan terakhir setidaknya sama besar
𝑚
dengan gulungan sebelumnya dapat dinyatakan dalam bentuk √ 𝑛 di mana
m dan n adalah bilangan bulat positif yang relatif prima. Tentukan dari
nilai m + n !

182 | T A N T A N G A N O L I M P I A D E
Olimpiade Matematika SMA 2020

59. (AIME 2001) Seorang pengantar surat mengantarkan surat ke sembilan belas
rumah di sisi timur Elm Street. Pengantar Surat memberitahu bahwa tidak
ada dua rumah yang berdekatan pernah mendapatkan surat pada hari yang
sama, tetapi tidak ada lebih dari dua rumah berturut-turut yang tidak
mendapat surat pada hari yang sama. Berapa banyak pola pengiriman surat
yang memungkinkan?

60. (AIME 2012) Jane berusia 25 tahun. Dick lebih tua dari Jane. Dalam n tahun,
di mana n adalah bilangan bulat positif, usia Dick dan usia Jane akan menjadi
dua digit angka dimana usia Jane diperoleh dengan menukar angka usia
Dick. Misalkan n menjadi usia Dick saat ini. Berapa banyak pasangan
bilangan bulat positif (d, n) yang mungkin?

61. (AIME 2002) Banyak negara menggunakan urutan tiga huruf diikuti oleh
urutan tiga digit sebagai pola plat nomor standar kendaraan mereka.
Mengingat bahwa masing-masing pengaturan tiga digit tiga huruf memiliki
peluang yang sama, probabilitas bahwa plat tersebut akan mengandung
setidaknya satu palindrom (pengaturan tiga huruf atau pengaturan tiga digit
yang terbaca dari kiri dan kanan sama, seperti KATAK dan DEIFIED) adalah
𝑚
, di mana m dan n adalah bilangan bulat positif yang relatif prima.
𝑛
Tentukan nilai 𝑚 + 𝑛

62. (AIME 2001) Tentukan jumlah semua akar-akar rean dan tidak real dari
1 2001
𝑥 2001 + (2 − 𝑥) =0

63. (AIME 2002) Solusi dari sistem persamaan :


225 64
log 𝑥 + log 𝑦 = 4
𝑥 𝑦
log 225 − log 64 = 1
30
adalah (𝑥1 , 𝑦1 ) dan (𝑥2 , 𝑦2 ). Tentukan nilai log(𝑥1 . 𝑥2 . 𝑦1 . 𝑦2 )

64. (AIME 2003) Diketahui bahwa :

Dengan 𝑘 dan 𝑛 adalah bilangan bulat positif dan nilai 𝑛 sebesar mungkin.
Tentukan nilai dari 𝑘 + 𝑛

183 | T A N T A N G A N O L I M P I A D E
Olimpiade Matematika SMA 2020

65. (AIME 2003) Dalam urutan meningkat empat bilangan bulat positif, tiga
bilangan pertama membentuk barisan aritmatika dan tiga bilangan terakhir
membentuk barisan geometrik. Bilangan pertama dan keempat berbeda
dengan 30. Temukan jumlah keempat bilangan tersebut !

66. (OSP 2019) Dalam kantong terdapat 7 bola merah dan 8 bola putih. Andi
mengambil dua bola sekaligus dari dalam kantong. Peluang terambilnya dua
bola yang berwarna sama adalah ...

67. (OSP 2019) Banyaknya pasangan bilangan asli (m, n) sehingga FPB(m, a) = 2
dan KPK(m. n) = 1000 adalah ...

68. (OSP 2019) Polinom 𝑝(𝑥) yang memenuhi persamaan :


𝑝(𝑥 2 ) = 𝑥 2019 (𝑥 + 1)𝑝(𝑥)
1
Dengan 𝑝 (2) = −1 adalah .....

69. (OSP 2019) Misalkan a, b, c, d adalah bilangan-bilangan bulat lebih besar dari
2019 yang merupakan empat suku berurutan dari barisan aritmetika dengan
a < b < c < d. Jika a dan d merupakan kuadrat dari dua bilangan asli yang
berurutan, maka nilai terkecil dari 𝑐 − 𝑏 adalah ...

70. (OSP 2019) Himpunan S terdiri dari n bilangan bulat dengan sifat berikut,
untuk setiap tiga anggota herbeda dari S ada dua di antaranya yang basil
penjumlahannya merupakan anggota S. Nilai terbesar dari n adalah ....

KUNCI JAWABAN BEBERAPA SOAL


23. 216 38. 8 56. 630 60. 25 64. 839
31. 342 41. (1 , 1) 57. 122 61. 59 65. 129
36. 31 45. 3 58. 79 62. 500
37. 1 , 2√2 , 3 55. 501 59. 351 63. 12

184 | T A N T A N G A N O L I M P I A D E
Olimpiade Matematika SMA 2020

Catatan :
..............................................................................................................................................
..............................................................................................................................................
..............................................................................................................................................
..............................................................................................................................................
..............................................................................................................................................
..............................................................................................................................................
..............................................................................................................................................
..............................................................................................................................................
..............................................................................................................................................
..............................................................................................................................................
..............................................................................................................................................
..............................................................................................................................................
..............................................................................................................................................
..............................................................................................................................................
..............................................................................................................................................
..............................................................................................................................................
..............................................................................................................................................
..............................................................................................................................................
..............................................................................................................................................
..............................................................................................................................................
..............................................................................................................................................
..............................................................................................................................................
..............................................................................................................................................
..............................................................................................................................................
..............................................................................................................................................
..............................................................................................................................................
..............................................................................................................................................
..............................................................................................................................................
..............................................................................................................................................
..............................................................................................................................................
..............................................................................................................................................
..............................................................................................................................................
..............................................................................................................................................
..............................................................................................................................................
..............................................................................................................................................

185 | T A N T A N G A N O L I M P I A D E
Olimpiade Matematika SMA 2020

Kepustakaan
Budhi, Wono Setya, Langkah Awal Menuju Olimpiade <atematika
SMA Nasional dan Internasional, Yogyakarta, 2009, PN
Pustaka Widyatama

Dynkin, E.B., Molchanov, C.A., Tolpygo, A.L., Rozental, A.K.,


Mathematicheskie Zadachi (Mathematical Problems), Nauka,
Moscow, 1965 (Rus- sian).

Faires, J., Douglas & David Wells. 2008. The Contest Problem Book VIII-
AMC 10-( 2000-2007). Amerika: MAA

Honsberger, R., In Po´lya’s Footsteps: Miscellaneous Problems and


Essays, Mathematical Association of America, Washington
DC, 1997.

Kessler, G & Lawrence Z. 2006. NYsML-ARML Contest 1983-1988.


USA:MAA.

Manfrino, Radmila B & J A G Ortega. 2009. Inequality: A Mathematical


Olympiad Approach. Berlin: Birkhauser.

Wiworo. 2009. Diklat Instruktur Pengembang Matematika SMA


Jenjang Lanjut : OSN Matematika SMA. Yogyakarta.

Soifer, A., Colorado Mathematical Olympiad: The First Ten Years


and Further Explorations, Center for Excellence in
Mathematical Education, Colorado Springs, 1994.

Soifer, A., Geometric Etudes in Combinatorial Mathematics, 2nd


edition, Springer, New York, 2009, to appear.

Tung, Khoe Yao, Ayo Raih Medali Emas Olimpiade Matematika


SMA, Yogyakarta, 2013, PN : Andi

186 | T A N T A N G A N O L I M P I A D E

Anda mungkin juga menyukai